Vous êtes sur la page 1sur 92

Downloaded From OutlineDepot.

com

Kannar Federal Courts, Spring 2011

JUDICIAL REVIEW
A. General Considerations: Article III of the United States Constitution creates the federal
judiciary and defines its powers. The language of the Article does five important things
(1) Creates a Federal Judicial System: First, the initial words of Article III the judicial
power of the United States shall be vested create a federal judicial system. Federal courts
were desired to effectively implement the powers of the national government; there as fear
that state courts might not fully enforce and implement federal policies, especially where
there was conflict between state and federal law. At a minimum, a federal judiciary could
help provide the uniform interpretation of the Constitution and federal laws; it could also
protect individual liberties.

How Do you structure any system: What are the ground rules? What are the bases
for the rules? All this doctrine is made by the S.C. S.C. makes a lot of changes that
nobody notices. S.C. can make a lot of changes & subject to further changes. In a
system of flux, its important to know where they came from, where they are, &
where they are going what the trends are, *whether there is something that you
can do to protect yourself/your client (highly technical aspect).
Nature of Justiciability
o Whether a matter is justicable (standing , mootness). Look at the history of
it. History of division of fed. & state cts. Extent of which states enjoy
immunity. Powers of Congress in defining fed. jurisdiction. Theme of
judicial restraint.
o Felix Frankfurter (keep it out of politics) was silent during ct packing
(FDR). Believed in independence, saw politicizing judiciary in a negative
way. During 1930s & 1940stryoing to protect judiciary from itself
(insulate institution from political attack).

(2) Creates Supreme Court and Permits Establishment of Lower Courts: Second, Article III
vests the judicial power of the United States in one supreme Court and in such inferior
courts as Congress may from time to time ordain and establish. Congress established lower
federal courts in its first judiciary act, and they have existed ever since.
(3) Insulates Federal Judges: Third, Article III assures the independence of the federal judiciary
by according all federal judges life tenure during good behavior, and salaries that cannot be
decreased during their time in office. This difference from state courts makes federal judges
uniquely suited for the protection of constitutional rights.
(4) Cases and Controversies Defined: Fourth, Article III defines the federal judicial power in
terms of nine categories of cases and controversies. These nine categories fall into two major
types of provisions. One set of clauses authorizes the federal courts to vindicate and enforce
the powers of the federal government. The other authorizes the federal courts to serve an
interstate umpiring function, resolving disputes between states and their citizens.
(5) Allocates Authority Between Supreme Court and Lower Courts: Fifth, Article III
allocates judicial power between the Supreme Court and the lower federal courts. Article III
states that the Supreme Court has original jurisdiction over cases affecting ambassadors, other
public ministers and consuls, and those in which a state shall be a party. In all other cases,

Downloaded From OutlineDepot.com

Kannar Federal Courts, Spring 2011

the Supreme Court is granted appellate jurisdiction, both as to law and fact, subject to such
exceptions and under such regulations as Congress shall make.
B. Judicial Review: Article III courts have the power of judicial review, which enables them to
determine the constitutionality of acts of the other two branches of the federal government and of
the states. However, this power is limited by the case and controversy requirement and by
justiciability doctrines.
Thomas Jeffersons letter to the justices: questions about treaty w/France. John Jay
writes back & says not w/in Courts duties, job is not advisory. (Pg. 53judiciary barred
from giving advisory opinions (dont give legal advice, only decide on cases that have
already arisen).
Steel Co. v. Citizens: can see temptation of lower cts of justicability. Scalia says fed. cts.
must resolve questions of subject matter jurisdiction. Cant give advisory opinions.
o Ct wouldnt take case if the result would be the same if there was an adequate &
independent state ground (ct imposed rule). If S.C. rendered an opinion, it would
be an advisory opinion.
Declaratory judgments: arent they on their face advisory opinions?
o Aetna Life v. Haworth: Ct is limited to an injunction or damages.
Marbury v. Madison (1803) (created judicial review for the federal courts): Marbury filed suit
in the United States Supreme Court seeking a writ of mandamus to compel Madison, as secretary
of state, to deliver his judicial commission. Marbury claimed that the Judiciary Act of 1789
authorized the Supreme Court to grant mandamus in a proceeding filed originally in the Supreme
Court. However, Madison claimed that the Constitution specifically limited the Courts original
jurisdiction to specific areas. Held: The Supreme Court has the power, under the Supremacy
Clause and Article III, 2 of the Constitution, to review acts of Congress which are repugnant to
the Constitution and find them constitutional. The Court ruled against Marbury and held that it
could not hear the case as a matter of original jurisdiction. The Court held that although the
Judiciary Act of 1789 authorized such jurisdiction, the statute was unconstitutional and hence
void.
Establishes where the jurisdiction of the S.C. lies. It was a case about judicial power.
Where the cts fit in the tri-part system.
Concept of political questions. Those that are political are not for the Ct to decide, they
differ from legal questions. There will be cases/disputes that are not w/in Art. III power.
Reaffirms separation of powers. Have an independent separation of powers. Derive
power from Constitution.
Appellate jurisdiction & what it means, he brings something over from the common law
system.
o Discretion, Prudence, & Judicial Function-MarshallCt wont take jurisdiction
if it shouldnt. Question of whether or not mandated by law to take the case..
o Ashwander v. TVA: says never make a rule constitutional broader. 2 ways of
disposing of the case, use the non-constitutional one.
1. Federal Courts are Courts of Limited Jurisdiction: Federal courts may not hear matters
unless there is constitutional authority, and Congress may not expand the jurisdiction granted
in Article III of the Constitution.
i)

Limited Original Jurisdiction: The Court in Marbury ruled that Article III creates the
ceiling on the Supreme Courts original jurisdiction. Congress cannot authorize original
jurisdiction greater than that provided for within Article III.

Downloaded From OutlineDepot.com

Kannar Federal Courts, Spring 2011

2. Role of Judiciary in Separations of Powers Doctrine: It is emphatically the province and


duty of the judicial department to say what the law is. The Constitution imposes limits on
government powers and those limits are meaningless unless subject to judicial enforcement.
C. Private v. Public Rights Models
1. Dispute Resolution or Private Rights Model: Some claim that the power of judicial review
exists only as a necessary incident of the power to decide cases. The definition of such
cases should be restricted to the kinds of disputes historically viewed as appropriate for
judicial resolution paradigmatically, those in which a defendants violation of a legal duty to
the plaintiff has caused a distinct and palpable injury to an economic of other legally
protected interest. Courts should avoid any role as a general overseer of government
conduct, and should especially avoid the award of remedies that invade traditional and
legislative prerogatives.
E.g. Frothingham
2. Public Rights Model: A more diffused conception of the function of courts in public law
matters, which depicts constitutional interpretation by the courts as other than an incident of
the power to resolve particular disputes between identified litigants, is called the public rights
model. Advocates for this model argue that the judiciary should not be viewed as a mere
settler of disputes, but rather as an institution with a distinctive capacity to declare and
explicate public values norms that transcend individual controversies.
E.g. Flast
3. Connection to Standing: The private v. public model factors into standing doctrine because
it determines how broad or narrow the claim needs to be.

JUSTICIABILITY: CONSTITUTIONAL &


PRUDENTIAL LIMITS ON FEDERAL JUDICIAL
POWER
A.

Justiciability Rules Generally: Article III courts are only authorized to hear
judicially cognizable disputes. In addition to the case and controversy limits set forth in Article
III, the federal courts have developed a set of sub-constitutional factors based on prudence
that dictate whether a dispute should be heard and/or decided.
1. Reasoning: Justiciability rules are based on separation of powers concerns and define what
the court may hear and what the court must defer to other branches of the government. They
also conserve judicial resources allowing the federal courts to focus their attention on the
matters most deserving of review. Finally, they improve judicial decision-making by
providing the federal courts with concrete controversies which will be zealously litigated for
judicial resolution.
2. State courts not required to follow federal standing requirements, even for federal
claims. (Fairchild: wanted to sue to keep the 19th amendment from being ratified. Suing the
Sec. of State and Atty general, to keep it from being entered and enforced.)

Downloaded From OutlineDepot.com

Kannar Federal Courts, Spring 2011

B.

ADVISORY OPINIONS: Federal courts will not issue advisory opinions.

Advisory
opinions go to judicial propriety/ethics. These raise some questions b/c extra judicial actions (ex.
Writing books). Notion of S.C. justices taking on other assignments (ethical issues).
Moral/ethical issue: when Kennedy was assassinated, Chief Justice Warren led the investigation.
1. Reasoning:
i) Separation of powers is maintained by keeping the courts out of the legislative process.
The judicial role is limited to deciding actual disputes; it does not include giving advice
to Congress or the President
ii) Judicial resources are conserved because advisory opinions might be requested in many
instances in which the law ultimately would not pass the legislature
iii) Helps ensure that cases will be presented to the Court in terns of specific disputes, not as
hypothetical legal questions (adverse parties will be more likely to present all the facts).
2. TEST: In order for a case to be justiciable and not be an advisory opinion, two criteria must
be met
i) First, there must be an ACTUAL DISPUTE between adverse litigants
a. Must be adverse, if Congress just says you can file to get things clarified, the parties
arent in dispute yet. (Muskrat v. U.S.)
ii) AND There must be a SUBSTANTIAL LIKELIHOOD that a federal court DECISION
in favor of a claimant will bring about some CHANGE or have SOME EFFECT.
a. Case where Court was asked to review Revolutionary War Vet benefits, but the
Secretary could ignore the courts recommendations, thats an advisory opinion.
(Hayburns Case)
3. State Courts May Offer Advisory Opinions about the constitutionality of pending
legislation or on constitutional questions referred to them by other branches of government.
These rulings can prevent unconstitutional laws and save legislature wasted effort
4. Declaratory Judgments Allowed: Congress adopted the Declaratory Judgment Act of
1934, authorizing a federal court to issue a declaratory judgment in a case or controversy
within its jurisdiction.
i) Must still meet requirement for judicial review. (Must be actual dispute b/w adverse
litigants & substantial likelihood that favorable decision will bring about some change.)
ii) Aetna Life Insurance Co. v. Haworth (1937): upheld the constitutionality of the DJA.
where there is such a concrete case admitting of an immediate and definitive
determination of the legal rights of the parties in an adversary proceeding upon the facts
alleged, the judicial function may be appropriately exercised although the adjudication of
the rights of the litigants may not require the award of process or the payment of
damages.

C.

FINALITY: A federal court will not decide a case if its decision is liable to be overturned
by one of the coordinate branches of the federal government.
Because such action would violate the principle of separation of powers. It would
interfere with the independence of the judicial branch by depriving its judgments of
finality.
i) Hayburns Case (1792)-Not a Supreme Court Case. (no decisions that can be
overturned by coordinate branches): Court was asked to determine Rev. War Vet
benefits. But, the secretary could refuse to follow the courts recommendation. The
assignment of these tasks was unconstitutional b/c of separation of powers.

Downloaded From OutlineDepot.com

Kannar Federal Courts, Spring 2011

Circuit ct judges in charge of determining if Revolutionary War Veterans


pension. When judges issue decisions, Sec. of War reviews & makes
decision to pay or not.
Judges decide fed. judiciary cant do this. There were supposed to be 3
branches.
Problem w/way Congress set this up was judicial branch was making
recommendations to executive branch. Sec. of War didnt have to follow,
it undermines judicial independence. Theres no finality.
Majority of judges said Act of Congress was void. This was the power
of judicial review.
Congress then fixed the system.

a. Class example: extradition proceedings, judge holds hearing, gives rec to Secretary who

decides whether to deliver the person to the country seeking extradition. There is adversarial
proceeding, but seems like same problem from Hayburn, b/c Exec may overrule the judge.
Argument for allowing it is the judges are acting as extradition officers. Maybe judge is
deciding different issue (is the person extraditable) vs the Secretary who is deciding whether
he will extradite or not.

Tutun v. US (84)naturalization proceeding. No case or controversy b/c


only 1 party (that is asking for a petition). Ct says that the US could
always be a party & challenge.
2. Exceptions to Finality Requirement:
i) Presidential Pardons: Although presidential pardons seems to wipe out judicial
decisions (thus compromises finality of criminal law decisions), this is a constitutionally
recognized exception (Article II) and only occur in the most minimal sense (very rare
exception).
ii) Immigration: Executive branch can rule on immigration issues i.e. deportation orders.
iii) Legislative Revision: Congress is forbidden from granting monetary relief to a party
which a federal court has ruled is entitled to none (retrospective = damages), BUT
Congress may require federal courts to revise their injunctions to be in compliance with
changes in federal law (prospective = injunctions).
a. (Congress cannot give relief which judiciary ruled party is not entitled to): Plaut v.
Spendthrift Farm (1995). In 1991, the Court ruled that actions brought under
securities laws had to be brought within one year of discovering the facts of the
violation and three years of the violation. Congress then amended the law to allow
cases to go forward that were filed before the decision if they could have been
brought under the prior law. Held: The statute was unconstitutional as a violation of
separation of powers. The Constitution gives the Federal judiciary the power not
merely to rule on cases but to decide them. The statute was unconstitutional because
it overturned a Supreme Court decision and gave relief to a party that the Court had
said was entitled to none.
S.C. decides its a much shorter statute of limitations than anyone
thought & a lot of cases were closed. Congress amended & extended the
statute of limitations & re-opened the closed cases. S.C. says Congress
can revise legislation but cant re-open a case by statute.
b. (Congress can require changes in injunctions to be in compliance with fed law):
Miller v. French (2000) Prospective relief under a continuing, executory decree
remains subject to alteration due to changes in the underlying law. Thus, unlike
Plaut, it is not the last word of the judicial department.
c. Legislative Revision: From Hayburns case to Plaut.

Downloaded From OutlineDepot.com

Kannar Federal Courts, Spring 2011

US v. Klein: said that Congressional motives cannot be overlooked.


Furthermore, Congress cannot dictate the result in a case where it is the
party to litigation. Cts had been making rulings. Congress thought
Andrew Jackson was giving too many pardons, so they passed legislation
to negate power of pardon. Klein & Boumediene cases are the only 2
cases that said Congress couldnt so something. In Kleina law about
judicial significance of a pardon. Congress passes a law that a
presidential pardon is not about proof of loyalty; the fact that you
received a pardon means youre disloyal & Ct should deny jurisdiction
for hearing the case. Ct says no, there are some things that are
essentially judicial. You cant direct us; this is a judicial function (on
how to take a certain piece of evidence). A proper Act of Congress was
flatly rejected.

iv) Claims Against the United States: Payment of any judgment against the United States
used to require a general or specific appropriation by Congress. The Ct has assumed that
it is permissible for Congress to employ non-Article III tribunals (judges lack life tenure
& whose powers are not subject to Art. III justiciability doctrines) either to adjudicate or
to recommend to Congress whether to pay such claims.
a. But, If Congress provides for review of the decisions of the tribunals by Article III
courts, justiciability rules apply to appeal.
b. Judicial Revision
US v. Johnson
Facts: The tenant brought suit against the landlord (rent control during
WWII), alleging violation of the Act and demanding treble damages and
reasonable attorney fees. The landlord challenged the constitutionality of
the Act via a motion to dismiss. The government intervened and filed a
brief in support of the constitutionality of the Act.
The landlord basically sued himself, & the Ct said it was a collusive suit,
NOT an actual dispute.
Holding: suit was collusive because it was not in any real sense
adversary. In so holding, the court noted that the tenant had no active
participation in the suit, exercised no control in the case, was only
nominally represented by counsel, and his counsel was selected by the
landlord's counsel.
Test cases: to see what the rights are. There are test cases framed by Congress: Muskrat v. USCt
refused to entertain a suit that Congress had specifically authorized. Congress cant set up an advisory
opinion situation.
Consent decrees: theres no adversity, how can Ct enter agreement? Some part of the deal has notion of
force of law. There is adversity until judges signature goes on the line. Could also avoid cases.
3. Nonjusticiable State Court Decision of Federal Right: If a question of federal law is
decided in a state court which does not have justiciability rules and thus the question would
not have been cognizable in federal court, the Supreme Court still has the power to hear its
appeal. This is based on reasoning that the state court decision itself satisfies the injury
requirement necessary to get into federal court.

Downloaded From OutlineDepot.com

Kannar Federal Courts, Spring 2011

D.

STANDING: Standing is the determination of whether a specific person is the proper party
to bring a matter to the court for adjudication. Standing doctrine promotes separation of powers
i.e. by restricting who may sue in court, standing limits what matters the judiciary will address
and minimizes judicial review of the actions of the other branches of government. Standing also
conserves judicial resources, ensures zealous litigation, and serves the value of fairness by
ensuring that people will raise only their own rights/concerns rather than intermeddling. Warren
says that standing is an iceberg wordmost of it is below water. The constitutional requirements
of standing are: live case & controversy.
1. Standing Constitutional Requirements (TEST): Requirements for standing are derived
from Courts interpretation of Article III, and cannot be overridden by statute b/c they are
based on Const. They must be satisfied whether they are challenged or not.
i) The requirements for Standing are Injury-In Fact, Causation, & Redressibility.
(1) INJURY-IN-FACT: The plaintiff must show that he has sustained or is in immediate
danger of sustaining some direct injury as the result of the challenged official conduct
and the injury or threat of injury must be both real and immediate, not conjectural or
hypothetical. (cant be too speculative or abstract, but different than asking whether there
is a cause of action)
(2) Flast v. Cohen: establishment clause its hard to determine who suffers when Congress
passes a law that violates the establishment clause. The Ct systematically moves away
from Flast (in Valley Forge & Hein). Hein reads Flast extremely narrowly.
a. Sierra Club v. Morton (1972) (organizational interest is insufficient injury):
The Sierra Club sought to prevent the construction of a ski resort in Mineral King
Valley in CA and asserted a special interest in the conservation and the sound
maintenance of the national parks and forces of the country. Held: No standing
because none of Sierra Clubs members had ever used Mineral King Valley. A
mere interest in a problem is not sufficient for standing. Challenged approval of
development of ski resort & had interest in national forest. Ct said they lacked
standing b/c NO INJURY. Aggrieved is more than it makes you upset.
b. US v. SCRAPbasically same case as Sierra ClubCt said they pled the right
stuff (it had to do w/use). s actually hiked there. Importance of injury in fact
requirement.
c. US v. Richardsonheld that the lacked standing to litigate whether the CIA
was violating Art. I 9 cl. 7.
d. Linda R.S. v. Richard Dmother bringing a case against the D.A. to force child
support payments. Marshall says shes making a speculative argument, he might
not be able to pay b/c of other possible factors (not being prosecuted is
speculative).
e. Simon v. Eastern Kentucky Welfare RightsIRS allegedly violated a statute.
Ct says its speculative & regulation of a requirement is not sufficiently definite.
Redressability & causation tied together.
f. Lujan v. National Wildlife Federation (1990) (members must use specific lands
damaged) Govt lessening the environmental protection of certain federal lands.
Two members of the National Wildlife Federation submitted affidavits that they
used land in the vicinity. Held: No standing: Too general to establish a
particular injury. P failed to demonstrate that they used specific federal land that
was being affected. Their plans to use the land were too conjectural (they didnt
have plane tickets to see the places)

Downloaded From OutlineDepot.com

Kannar Federal Courts, Spring 2011

g. Injuries to Statutory Rights Allowed: Violations of rights created by statute


also are sufficient for standing purposes. Congress may create a statutory right
or entitlement the alleged deprivation of which can confer standing to sue even if
the plaintiff would have suffered no cognizable injury absent the statute, but cant
be a generalized injury (still need injury in fact).
i. Trafficante v. Metropolitan Life Insurance (1972) (standing on Civil
Rights Act): Two white residents of an apartment complex were accorded
standing to challenge the owners discrimination against black applicants
in violation of the Civil Rights Act of 1968. Statute created right to be
free from discrimination
1. But See Lujan v. Defenders of Wildlife (1992) (no standing
w/re Take Care Clause/General Public Interest): The
Endangered Species Act provides that any person may
commence a civil suit to enjoin a violation of the Act. Held:
Congress could not create standing in this manner. Its a general
public interest. To permit Congress to convert the
undifferentiated public interest in executive officers compliance
with the law into an individual right is to permit Congress to
transfer from the president to the courts the Executives
constitutional duty, to take care than the laws be faithfully
executed.
2. Lujan: wildlife advocacy group sued for violation of 7(a)(2).
A consultation, believes that consultation wasnt done &
believes they were appropriate ppl to bring this complaint. s
lawyers were trying to claim an injury (lack of consultation
increases chances of endangered species). Trying to make a
personal injury claim (future enjoyment). Argument must be
more than a cognizable injury; it must be an injury-in-fact (the
ppl must actually be injured). 2 women traveled to 2 areas yrs
before (they either saw or did not see the endangered species) &
wanted them to survive & wanted to return one day & see the
species again. Ct said this argument is not concrete, theres not
plan. Must have a connection to the animals. In section III(B)
s did not show redressibility and part IV (pg 133)Congress
created a procedural injury & a citizen may bring a civil suit.
In the ESA, the Ct is looking at #1. The statute says any
person may sue any person for any possible violation.
Scalia said Congress cant get around cases &
controversy standard of Art. III. This is not a case that is
trying to enforce a procedural injury; this would be
allowed.
If Congress can write a statue like this & cts will go
along w/it, then judicial & legislative branches have
taken power away from the executive branchthis is
unconstitutional. It is the Cts duty to recognize
separation of powers.
It is okay to Congress to allow injuries (for aesthetic
injuries).

Downloaded From OutlineDepot.com

Kannar Federal Courts, Spring 2011

Kennedy & Soutertheyre on board w/Scaliastatute


is overbroad, Congress could be read (through the
statute) as saying no Art. III.
Stevensconcurring in the judgmentthere is standing,
they lose on the merits.
Blackmun & OConnor (dissent)issue w/redressibility.
In part II, Scalia says across the board standing in a noninstrumental complex & the dissenting judges think
there are some areas of procedural harms that could have
standing & Scalia is keeping these cases out. There are
procedural protections in the statute
When the case came down, questions about
environmental standing. It was a general/broad statute.
Get clarification on pg 142(2)
o Fed. Election Commn v. AkinsFEC
determine AIPAC was not a political committee
& they didnt have to disclose members.
wants to sue FEC for misapplication of its law.
Question of how can you sue an agency for not
following its own rules (Wright) this case
comes out the other way b/c of the statute. The
statute was carefully drafted (dismissing
complaint gives you standing)
What is the injury they have suffered?
They dont have information to make
decisions about candidates. Peoples
rights as voters are undermined this is
an injury & gives enough basis to file a
complaint & brings them in the suit.
This is a generalized grievance.
Scalias dissentunitary executive.
This is going too far, all trumping on
executive branch & is still a generalized
grievance.
3. How can Congress create standing:
Pass a statute creating standing
Pass a statute creating substantive rights, a violation
of these righs would create standing.
ii. Federal Elections Commission v. Atkins (1998) (statutory right to
information created standing): Court granted standing and concluded
that Congress had created a right to information about political
committees and that the plaintiffs were denied the information by virtue
of the FECs decision. Why is this not a generalized grievance like that
in Lujan? Court thought right to information was more like other
common, but cognizable injuries like mass torts or voting injuries, so its
common but still important.
h. Massachusetts v. EPA: If state doesnt regulate pollution, then greenhouse gases
go up, ice caps melt, and the home owners beach front property will be eroded.
5 justices said this gave standing. By regulating the cars the court says it reduces
the risk to the property. Refusal by EPA to enforce regulations. Massachusetts

Downloaded From OutlineDepot.com

Kannar Federal Courts, Spring 2011

wants to challenge EPAs failure to act. Massachusetts has to plead that they
have some interest. The dissent talks about redressibility problem (its a
globalized grievance). No redressibility, causation is very attenuated. Failure of
EPA to issue vehicle emissions rules b/c of cars in China. States harmsgases
will contribute to harm. Roberts response to Stevensthen states do have
special powers. It is probably limited to states.
i. Summers v. Earth Island Institute (Supplement)the s arguments is were so
big, our interests are so nationwide, s programs are nationwide, the statistical
probability is very high b/c of membership/interests that there will be a problem.
The Ct. says no, you have to prove that a concrete injury happened to at least 1
individual. Dissent says if there were a statute (parallel to Akin) let people part
of the rule-marking, it will give them standing.
j. Stigmatic injury doesnt count. Only counts if plaintiff personally suffered
injury. E.g., where parents claimed discrimination b/c IRS didn't deny tax
exempt status to schools, no injury b/c parents didn't allege that their children
applied/would apply to those schools (Allen v. Wright, 1984).
k. Right to receive benefits in a non-discriminatory manner counts, even if higher
benefits wouldn't have actually been received (Heckler v. Mathews)
l. Freight tax's adverse impact on cost of recycling counts. Court bought argument
that more expensive recycling would deplete natural resources around school
(SCRAP case).
m. Taxpayer Standing: Usually not enough, different in municipality (can feel
effects).
(3) CAUSATION: The plaintiff must allege that the injury is fairly traceable to the
defendants conduct.
a. Allen v. Wright (1984): No causation when IRS didnt follow law and deny taxexempt status to racially-discriminatory leading to black children being
stigmatized, denied integrated education. Real injury, but the injury is not fairly
traceable to the government conduct. The injury to respondents is highly
indirect and results from the independent action of some third party not before
the court.
i. Could maybe get around this by defining the injury as being denied the
right to receive economic benefits in a nondiscriminatory way.
ii. Common criticism is that standing is determined by how you define the
injury
iii. Facts: Parents of black public school children are suing the IRS b/c it did
not deny tax exempt status for desegregated private schools & that it was
harming their children b/c schools were not integrated. On pg. 103,
argument (a) sounds like a generalized grievance.
iv. Issue: whether s have standing to bring the suit.
v. In II(A)the Ct talks about justiciability in general. Core component is:
injury is fairly traceable & likely to be redressed by requested relief. All
of the doctrines come from Art. III. The s claim 2 injuries. The 1st
fails b/c there is no judicially cognizable injuryit is a generalized
grievance. The stigmatizing injury the Ct said is too abstract, it needs to
be concrete. Its too generic & undefined. The 2nd claim of injury was
the diminished opportunity for children to receive an education in a
racially integrated public school (Ct said the link of causation is far too

10

Downloaded From OutlineDepot.com

Kannar Federal Courts, Spring 2011

weak & sees s complaint as asking the Ct to fix IRSs policy


separation of powers argument.
vi. Holding: Respondents did not have standing to sue as their first basis for
standing failed because it did not constitute a judicially cognizable injury
and their second basis failed as the alleged injury was not fairly traceable
to government conduct that respondents challenged as unlawful.
vii. Rule: the requirement of standing has a core component derived directly
from the Constitution. A must allege personal injury fairly traceable
to the s allegedly unlawful conduct & likely to be redressed by the
requested relief.
viii. Stevens dissent: preferential tax treatmentwhy should it change now?
It is supposed to be about standing, not about separation of powers.
Dooming the s w/o a chance to be heard.
b. ASARCO v. Kadish: state taxpayers were suing school teachers over mineral
leases. Private leaseholders intervened as s. Once the lower ct ruled, a
controversy began.
(4) REDRESSABILITY: The plaintiff must allege that a favorable federal court decision is
likely to redress the injury.
a. Ex., if requested relief only cuts 10% of challenged projects funding, that
probably isnt redressable, if its 90%, thats probably redressable.
b. In case where student applying to med school sued b/c school has AA policy of
setting aside 16 out of 100 spots per year for minorities. Even if there was no
AA, he still might not get in so not redressable. But by defining it as the ability
to compete for all 100 spots, there was redressability/causation and standing.
(Bakke)
2. Prudential Limits: In addition to the constitutional limitations, the Court has also identified
3 prudential standing principles. Unlike constitutional requirements, Congress can override
these by statute (e.g. qui tam)
i) No third-party standing: A party may assert only his or her own rights and cannot raise
the claims of third parties not in court. (subject to a few limited exceptions, p. 90 in
Chimerinsky)
ii) No generalized grievances: Cant sue as citizen or taxpayer concerned w/ govt following
law
a. Taxpayer Status Not enough for Standing when suing about how taxes are spent
(Frothingham): interest too minute and indeterminable, must be particularized
injury to the party
i. Municipal tax payers usually do have standing, they can actually feel the effects
of how their taxes are spent.
ii. Sometimes no one can sue, then its up to the political process.
iii) Zone of Interest Rule: A party must be within the zone of interests protected by the
statute.
a. Applies when person is challenging an administrative agency regulation that does not
directly control persons actions. P must be part of the group intended to benefit from
the law.
3. Qui Tam Actions: Qui tam is a provision in the False Claims Act, which allows for a private
individual or whistleblower with knowledge of past or present fraud committed against the
U.S. government to bring suit on its behalf. Someone has defrauded the government, private

11

Downloaded From OutlineDepot.com

Kannar Federal Courts, Spring 2011

4.
5.
6.

7.
8.
9.
10.
11.

E.

party sues the other party for the government. If they win, then the private party gets some
money. They fed. government has assigned part of the rights to the governments claim.
Competitor Standingcaused grief by what an agency did for a 3rd party.
Pg. 157(2)the traditional rule was that parties to a lawsuit could only assert their own rights
or immunities.
Preliminary note on as-applied & facial challenges & problem of separabilitypg. 162
questions that arise when ppl litigate their own issue, 3 rd parties may or may not be covered
under a statute & it must be struck down on its face (as applied facial challenges). Pg 164(c)
series of cases in which Ct said some part of statute is discriminatory limitation on 1
gender is unconstitutional & extend rights to other gender. Pg. 165defn of crime or
defenseno possibility of constitutional application b/c its perfectly unconstitutional. 168174overbreadthbasic notion, a person can attack the application of a statute to them, the
fact that it is overbroad raises constitutional questions.
The Substantiality RequirementBroadrick v. Oklahoma (169)Cts should only apply
overbreadth analysis when there is a substantial overbreadth to a case in which merely
speech was involved.
Complicated fed. questions about state law. Its must easier for fed. cts when a challenge is
against a fed. statute b/c ct can give a narrowing construction.
Mass. v. OakesMass. S.C. reversed a conviction under a state statute. State amends the
statute. S.C. applies overbreadth doctrine & reverses state supreme ct (chilling effect).
Doe v. Reed (pg 8 Supp.)has to raise question about referendum. Deemed to have satisfied
the facial challenge.
As of 2006, the law begins to change (Ayotte & Gonzales). Ct is shifting from facial
challenge methodology to as-applied methodology.
Ayotteif issue of minors health comes into play, then apply as-applied
methodology. Groups were upset b/c it wont get to the Ct.
Gonzales v. CarhartCongress found no one would come into this medical
situation. s said there are people that need this procedure. The Ct found
uncertainty so a fail challenge must fail. It is proper to consider an as-applied
challenge.

MOOTNESS: A case is moot when events subsequent to the filing of the case resolve the
dispute. An actual controversy must exist at all stages of federal court proceedings, both at the
trial and appellate levels. This rule is derived from Article IIIs prohibition against federal
courts issuing advisory opinions. Ancient thoughts standing was about parties & mootness,
ripeness & political question were about issues. Mootness is the doctrine of standing set in a time
frame. The adversarial posture must exist at time of adjudication. For standing, you need a case
or controversy and for mootness, it allows adjudication of issues capable of repetition yet evading
review. If a case is moot, then judges are giving an advisory opinion. Standing requirements are
more constitutional than mootness. If standing fails it fails (Wright) & Ct must dismiss the case.
If there are questions about mootness, Ct will weigh policy concerns, its more flexible (its a
managerial flexibility).
1. Honig v. Doechallenge to way of enforcing a CA law about handicapped children.
Whether CA was managing system w/in the Constitutional limits. While case was litigated,
the children would turn 21, question arose whether the case could go forward. Rehnquist
wanted an overt policy holding that if a case get the S.C., at that point, no mootness
argument. It was the 3rd case that got the S.C. that was moot. In contrast, Scalia is much
more conservative & thought the case should be dismissed as moot (constitutionally

12

Downloaded From OutlineDepot.com

Kannar Federal Courts, Spring 2011

precluded from giving an advisory opinion if ct decides on a case thats moot b/c there no Art.
III case or controversy).
2. DeFunis v. Odegaard (exception) (1974) (3L (admitted via p. injunction pending case over
discrimination) seeking admission. Case was moot b/c school said hed be allowed to
graduate no matter what, controversy was over):
Facts: Petitioner student commenced action against respondent law school
contending that the procedures and criteria employed by respondent invidiously
discriminated against him on account of his race in violation of the Equal Protection
Clause. S.C. decides the case is moot after oral argument. The doctrinal reason for
dismissing the case as moot an actual controversy must exist (not just when filing),
no matter what Ct decides, he will get what he wants (the record states that school
will allow him to finish the semester).
Holding: controversy between the parties had clearly ceased to be "definite and
concrete" and no longer touched the legal relations of parties having adverse legal
interests because the petitioner would have completed his law school studies at the
end of the term for which he was registered regardless of any decision the court
reached on the merits of the litigation. The court found that it could not, consistently
with the limitations of U.S. Const. art. III, consider the substantive constitutional
issues tendered by the parties.
Brennan says there are many possible factors that the stipulation does not speak to
(what if he gets sick & drops out).
Pg 184DeFunis did not file a class action lawsuit, & it is important to the way the
ct treats the case.
Pg 185the stipulation depends upon their good faith, they have not changed their
policy. S.C. says it doesnt matter that case is moot b/c of stipulation.
3. Exceptions
i) Wrongs Capable of Repetition Yet Evading Review: When the injury is BOTH likely
to recur to plaintiff (not just anyone) AND is of an inherently short duration that would
make complete federal court review impossible, then case is still moot, but not dismissed
The relevant question is the possibility of recurrence with respect to the complaining
party, not another member at large.
Roe v. Wade, Court refused to dismiss the case on mootness grounds, explaining that
the duration of pregnancy was inherently likely to be shorter than the time required
for federal court litigation. Thus, the challenge to the state laws prohibiting abortions
could be capable of repetition yet evading review.
Prior restraints on speech commonly fall under this exception (likely to happen
again, over quickly)
ii) Voluntary Cessation: A case is not dismissed as moot if the defendant voluntarily ceases
the allegedly improper conduct but is free to return to it at any time. Only if there is no
reasonable chance that the defendant could resume the offending behavior is a case
deemed moot on the basis of voluntary cessation. Cts play institutional role. Cts
looking at s, if could moot out a case by not doing what is complained of while the
case is ongoing. would be manipulating jurisdiction of the ct & when case is
dismissed as moot, could do some activity the day after case is dismissed. Ct will not
allow s in bad faith to moot cases out. If DeFunis had been a class action &
Washington said that they changed policies, then apply voluntary cessation doctrine.
Ex. Roe v. Wadecapable of repetition claim, nature of gestation & judicial
process, it is almost impossible for a pregnant woman to go through cts to S.C. (it
will take more than 9 months). Impossible to obtain review & moot out by the

13

Downloaded From OutlineDepot.com

Kannar Federal Courts, Spring 2011

birth of the baby. Its different from voluntary cessation b/c repetition is a bad fit
to judicial fit. This also happens in elections cases (pg 190)
City of Erie v. Paps (voluntary cessation)highest state ct strikes down law.
Erie brings it to S.C. & Paps voluntarily goes out of business & argues that
theres no party so its moot (so the high cts ruling stands). Owner of Paps
opens another business. S.C. cuts through it & held that the case was not moot.
iii) Collateral Consequences: A case is not moot where a secondary or collateral injury
survives after the plaintiffs primary injury has been resolved (e.g. employee reinstated,
but back pay Q remains). So long as decision has some effect in the future, case
shouldnt be dismissed.
a. E.g. a challenge to a criminal conviction is not moot, even after the defendant has
completed the sentence and is released from custody, when the defendant continues
to face adverse consequences of the criminal conviction, including preventing certain
occupational licenses, voting, etc.
b. Class Actions: Properly certified class actions not mooted even if named partys
claims are mooted as long as some members of the class still have a live controversy.
iv) Mootness in Criminal Cases
What do you do on appeal?
In civil cases, Munsingwear go all the way down, remand & dismiss so
case never existed. But in Bankcorp, it is altered b/c the party settled.
Bankcorp wanted Ct to apply Munsingwear. Scalia said that once
mootness is gone, higher ct cant do anything. Bankcorp was trying to
do something like voluntary cessation. Munsingwear should be thought
of as equitable as something that may or may not be applied.
Fed. Criminal casesif someone dies, its moot; the record below was
dismissed under Durham (Dove).
4. Procedural Issue: Mootness can be raised by the court sua sponte. Usual practice when a
civil case becomes moot pending appeal is to vacate lower decision and remand with an order
to dismiss the case. This also clears the path for future litigation between the parties if
needed.
i) But Settlement: United States Bankcorp: after certiorari granted, settlement occurred,
losing party in court of appeals asked for that decision to be vacated. SCOTUS said
mootness by reason of settlement usually doesnt justify vacatur of judgment under
review. Precedent is still useful.
a. Most important for repeat players so they dont just keep settling until they get a
useful judgment. May no apply in a one-off case.
5. In mootness, there may be a drift to de-constitutionalize a fed. class action, make motion if
fed. ct., if certified, then proceed for the class. Potential problem w/class action:
Sosna v. Iowacould state require you to stay in the state for 1 yr to get a
divorce? It was class certified, moots out, could class move forward & get a
resolution close to capable of repetition.
Franksis more difficult. Whether the class can go forward when no party has
seniority. Look to see if its an adversary relationship.
Gerstein v. Pughpre-trial detention. People were complaining about 1 st 6 days
(waiting for bail) in jaileverybody will always move out. The problem is the
mooting out of the named parties happened before the class is certified.
Public defender absorbs role of named . There will always be
someone (the class will always exist).

14

Downloaded From OutlineDepot.com

Kannar Federal Courts, Spring 2011

F.

Geraghtyparole guidelines legitimacy in question. People kept getting out.


District Ct. refused to certify the class & s mooted outwhat happens?S.C.
separates the case in 2 look to see if certification was appropriate.
Issues: 1 is claim on the merits & other is claim that he is entitled to
represent a class.
Holding: action brought on behalf of a class doesnt become moot upon
expiration of named s substantive claim. Ct is thinking maximal
practicality.
Roper (197)not a public interest case. Named s are given what they want
from the banks & ct denies class certification. S.C. holds the same way as
Geraghty ( s are trying to moot cases out, like in Bankcorp). The issue was
whether it was proper to deny class certification.

RIPENESS: The ripeness doctrine seeks to separate matters that are premature for review
because the injury is speculative and may never occur, from those cases that are appropriate for
federal court decision. Dispute must have progressed beyond the point where a legal question is
merely hypothetical. It is designed to ensure that an actual, immediate, and concrete controversy
is presented to the court. It is closely related to standing (b/c no injury is often = injury hasnt
happened yet)
1. United Public Workers v. Mitchell
Facts: employees contended that 9(a) of the Hatch Act, which prohibited them
from taking an active part in political management or political campaigns,
violated their rights. The Hatch Act forbade executive officers & employees
from taking part in political management/campaigns. Workers wanted to
challenge the rules b/c they wanted to engage in political activities.
Holding: the federal employees who had not yet engaged in the activities
prohibited by 9(a) did not state a cognizable controversy because they sought
an advisory opinion on broad claims of constitutional rights. Therefore, the
district court erred in hearing the claims of those employees. Third, although the
federal employee who had been charged by the commission with political
activity presented matters appropriate for judicial determination, the Court held
that a breach of the Hatch Act and Civil Service Rule 1 could, without violating
the Constitution, be made the basis for disciplinary action. Section 9(a) was only
directed at partisan political activity and its application to all federal employees
was justified.
Ct decided that there were many employees that did not engage in political
activity, only 1 was involved. The rest of them, its not clear what they wanted to
do. For everyone other than Poole, the case is not ripe. Suit has not yet taken
shape for review. If it is deemed so indefinite, ripeness is the doctrine the Ct will
invoke b/c the controversy isnt definite enough.
2. Abbott Labs Test: Abbott Labs & Toilet Goods v. Gardner (companion cases). Regulation
in toilet goodshave to give fed. inspectors total access, if access is refused, commissioner
can suspend manufacturers need. In Abbott, the regulation was about writing on drugs. In
Toilet Goods, there were a bunch of contingencies & none occurred.
Abbott Labsnothing has happened but the regulation is self-enforcing & need to
know now if the law is good b/c of packaging (would have to change all labels if the
regulation is enforceable). Pre-enforcement challenge to an agency regulation.
Harlan2 matters that are important (1) whether they look like legal issues (fitness)
& (2) judgments as to the bottom line of fitness, if you wait (better fitted

15

Downloaded From OutlineDepot.com

Kannar Federal Courts, Spring 2011

3.
4.
5.
6.

controversy), sense if anyone will get hurt if you wait (for fitness) ex. could evolve
further & no one will get hurt. In Toilet, it could never be enforced, but Abbott, need
to know now.
Lugan v. National Wildlife Federationobjecting to change in management of forests.
Not an agency action (doesnt fall under APA), its a program, not an adjudication. Not ripe
b/c you dont know what they are going to do.
Reno v. Catholic Social Servicesnot ripe for review b/c havent shown class will be
injured b/c of statute they are challenging.
Buckley v. ValeoCt thinks its time to give a ruling before the next big election.
Duke Power v. Carolina Environmental Study Group (210)a challenge to provisions of
the Price-Anderson Act which limits liability of employers dealing w/nuclear material. The
ruling on Price-Anderson was RIPE (contingencies have to happen for cap to matter but it is
deemed ripe b/c people that want to build these plants are exposed to liability w/o the cap,
they would never want to build b/c they would want to know the liability in the future today
(Like Abbott Labs, its a legal issue also). Being a legal issue makes it more likely that its
ripe).

7. Anticipation of injury is not sufficient: (OShea v. Littleton) (1974) Not ripe: Black Ps alleged that
magistrate and a judge discriminated against blacks in setting bail and imposing sentences. None of
the plaintiffs currently faced proceedings in the defendants courtrooms so the threat of injury form the
alleged course of conduct was too remote to satisfy the case-or-controversy requirement. Attempting
to anticipate when and if injury will occur is forbidden speculation and conjecture.
i) ***This decision could be placed either under the label of standing no injury was alleged or
ripeness the type of injury was adequate but had not yet occurred.***
ii) Damage claims never go moot. Future claims may become pure speculations.
iii) Facts: Respondents commenced a civil rights action against petitioners alleging they intentionally
engaged in various patterns of conduct in the administration of the criminal justice system that
deprived respondents of their constitutional rights and their rights. A lot of the claims are in
general terms, none of the s at the time of the complaint suffered any injury in the manner
alleged. It was institutional racism.
iv) Holding: issuance of an injunction was not forbidden and that directed the lower court to fashion
appropriate injunctive relief. The Supreme Court held that the complaint failed to satisfy the
threshold requirement imposed by U.S. Const. art. III that those who seek to invoke the power of
federal courts must allege an actual case or controversy. None of respondents claimed they
suffered any injury in the manner specified, and the case or controversy requirement was not
satisfied by general assertions that respondents would be prosecuted for violating valid criminal
laws. Moreover, respondents did not establish likelihood of substantial and immediate irreparable
injury and inadequacy of remedies at law.
v) Ripeness: a specific party has to be injured. There is something too amorphous about the

claim. What is the shapelessness in the case?No injury, its about future injury. Its too
speculative that those people will become subject to the criminal justice process generally
or subject to what they allege. Claims are too generic, general, & it hasnt assumed a
concrete form.
vi) Under II (215)no basis for equitable relief. Downside of entering relief in cases like this.
Problem w/ remedy is its too difficult to enforce & would amount to a fed. judicial takeover of a
state criminal justice system.
vii) The big question after OSheais there any way to construct a justicable case or controversy
when involving the government?

Rizzo v. Goode-discrimination. Ct. said its antidotal, will have a problem


showing it will happen again.

8. Substantial Likelihood TEST: In order for a person to have standing to seek an


injunction, the individual must allege a substantial likelihood that he or she, personally, will

16

Downloaded From OutlineDepot.com

Kannar Federal Courts, Spring 2011

be subjected to or injured by the allegedly illegal policy in the future. Absent proof of such a
substantial likelihood, the case will be dismissed on standing and ripeness grounds.
i) City of Los Angeles v. Lyons (1983) (not ripe if no substantial likelihood that plaintiff would be
subjected to illegal chokehold in future): Lyons was subjected to such a chokehold. Lyons did
not have standing to seek injunctive relief. Although Lyons could bring a suit seeking damages for
his injuries, he did not have standing to enjoin the police because he could not demonstrate a
substantial likelihood that he, personally, would be choked again in the future. Lyons has been
subjected to unjustifiably using chokeholds on black arrestees or suspects. He alleged damages
& sought an injunction. He had a damage claim, not abstract, his complaint suggested he had
been subject to this treatment. S.C. distinguishes b/w damage claim & injunction damage claim
can be decided below but talks about injunction dont know if he will be subject to this
treatment again. Not demonstrated that it will happen again. Ct treated Lyons as a standing case.
Why isnt it a mootness case?The injury already happened, & Ct doesnt know if it will happen
again.
ii) Note 5 (220)Ct is as friendly to class actions as it is opposed to institutional injunctions. Ct
seems to like class actions that are filed early. If you find a plaintiff that has standing once the
class is certified, then its a stronger case.
iii) Gratz v. Bollingerits outcome-centric. When the Ct wants to reach something, these

doctrines are malleable the other way (unlike Allen v. Wright).


iv) For standing & mootness, certify class action w/complaint, have an actual injury. For ripeness
narrowly tailor the remedies. Hardship of delaying adjudication helps w/issues of standing &
ripeness. In ripeness, they may have to wait until the situation takes on a concrete shape.
v) Lujan v. Defenders of Wildlife (1992) (not ripe w/out concrete plans): Suing to save endangered
species overseas. The fact that plaintiffs had visited the areas in the past proved nothing and their
desire to return in the future some day is insufficient for standing without any description of
some concrete plans or any specification concerning when that some day will be.

G.

POLITICAL QUESTION DOCTRINE: Certain allegations of


unconstitutional government conduct should not be ruled on by the federal courts even though all
of the jurisdictional and other justiciability requirements are met.
In other words, the political question doctrine refers to subject matter that the Court deems to
be inappropriate for judicial review.
Although there is an allegation that the Constitution has been violated, the federal courts
refuse to rule and instead dismiss the case, leaving the constitutional question to be resolved
by the political process.
It remains about issues mostly & stands apart from the other 3 (standing, mootness &
ripeness). Some are non-justicable b/c they are non-amendable to the shaping of the
judiciary.
1. Political Question Factors TEST: The Court, in Baker v. Carr (1962), stated, prominent on
the surface of any case held to involve a political question is found
i) A TEXTUALLY DEMONSTRABLE COMMITMENT of the issue to a coordinate political
department

ii) A LACK of judicially DISCOVERABLE and MANAGEABLE STANDARDS for resolving it


iii) The impossibility of deciding without an INITIAL POLICY DETERMINATION of a kind
clearly for nonjudicial discretion
iv) The impossibility of a courts undertaking independent resolution without EXPRESSING LACK
OF THE RESPECT DUE coordinate branches of the government
v) Any unusual need for UNQUESTIONING ADHERENCE to a POLITICAL DECISION
already made
vi) The potentialality of embarrassment from MULTIFARIOUS PRONOUNCEMENTS by various
departments on one question.

17

Downloaded From OutlineDepot.com

Kannar Federal Courts, Spring 2011

vii) Pg. 247the mere fact that something involves politics doesnt mean its a political question.
2. Nixon v. US
Facts: federal judge was convicted of lying to a grand jury & he refused to resign
& still collected his salary. He challenged the impeachment & claimed that going
before the Senate was not a trial.
The Senate claimed that the judiciary couldnt review the claim.
Nixons lawyers try to blow by a good Senate argument (entrusted to Senate, not
reviewable Senate shall have SOLE power).
The Ct says sole only appears in 1 other part of the Constitution. The sole
place you can try this case is the Senate. At this point how should Nixons
lawyers respond?
o Theres no check on Congress. It focuses on what try means. Try or
trial is that clause is a substantive word & he is allowed to have a
substantive trial, even if Senate can try the case Nixon has the right to
have the judiciary review it.
o S.C. responds that theyre hesitant b/c of sole word. Hesitant to 2 nd
guess the Senate giving a trial.
o On 230, Justice White characterized the majority as having said that they
dont know what a trial is.
o 231Souter concurselement of discretion in political question. Want
to reserve some judicial power. There is a resemblance to a trial.
3. Examples of Test Applied: The Court has considered and adhered to the political question
doctrine in the following areas: the republican form of government clause and the electoral
process; foreign affairs; Congress ability to regulate its internal processes; the process for
ratifying constitutional amendments; instances where the federal could cannot shape effective
equitable relief; and the impeachment process.
4. Textual Demonstrable Commitment to Another Branch
Vieth v. Jubelirer (2004): deliberate partisan gerrymandering comes close to Ct
throwing up their hands. (Note pg. 238)
Luther v. Bordenthe Guarantee clause. Ct declines to get involved, its up to
Congress to decide.
Constitutional Amendmentstreat w/deference.
Foreign AffairsGoldwater v. Carter takes Senate to enter a treaty; it must take
Senate to get out. Ct says it takes great deference in foreign affairs & will not tell
president what he can/cant do.

18

Downloaded From OutlineDepot.com

Kannar Federal Courts, Spring 2011

CONGRESSIONAL REGULATION OF JURISDICTION


A.

CONGRESSIONAL RESTRICTION OF LOWER FEDERAL COURT


JURISDICTION
1. Madisonian Compromise: Congress has authority to determine/limit the jurisdiction of the
federal courts because Congress had the discretion as to whether to establish those tribunals
in the first place.
2. Establishment of non Art. III Cts (ex. territorial, military cts). There is a legitimate place for
them. In the necessary & proper clauseCongress is to determine the exclusivity of
jurisdiction. The Ct has fought to preserve jurisdiction (278). 292-93Congress wanted to
limit Cts power from intervening in labor disputes. Theres a distinction of judicial
limitation statutes. Onestrip Ct of jurisdiction. Alternative formconcedes jurisdiction
but precludes certain remedies.
3. Parity Debatearises from perception of Civil Rights lawyers. Evaluative argument. Lock
horns about where there is parity among state & fed. systems regarding less popular groups
(State Ctsjudges have to be re-elected).
i) Sheldon v. Sill (1850) Lender assigned debt to out of state person to obtain diversity jurisdiction.
Under the Judiciary Act of 1789, federal courts could not hear cases where diversity was created
by assignment. Sill contended that b/c Article III authorizes diversity jurisdiction and does not
contain a limitation, this section of the Judiciary Act was unconstitutional. Held: Congress may
withhold from any court of its creation jurisdiction of any of the enumerated controversies.
Courts created by statute can have no jurisdiction but such as the statute confers.
a. Critique: Others claim that lower federal courts created by Congress must have the full
judicial power described in Article III, as the text states the judicial power of the U.S. shall
be vested in one Supreme Court and such inferior courts as Congress shall establish.
i. Congress had a choice regarding whether to establish lower federal courts, but once they
were created they were required to have the judicial power to decide all matters
described in Article III. All attempts to restrict jurisdiction are unconstitutional.
b. Issue of bond & mortgage & if circuit Ct had jurisdiction. Judiciary Act precludes Ct from
hearing cases involving a promissory note. Art. IIIdiversity jurisdiction. It is possible to
argue that the judiciary act runs afoul to the constitution.
c. The Ct said may for jurisdiction Congress has discretion to limit structure of jurisdiction of
cts. Judiciary Act doesnt violate the Constitution; therefore, Circuit Ct doesnt have
jurisdiction.
4. Exclusive Jurisdiction Critique: In Martin v. Hunters Lessee, Justice Story urged that the full
judicial power must be vested in some federal court, b/c the text of Article III states shall be vested
and not may be vested. If then, it is the duty of Congress to vest the judicial power, it is the duty to
vest the whole judicial power, and as a result, lower federal courts must exist and are exclusively
vested with jurisdiction of cases over which the Supreme Court could not take original cognizance.
(every claim has to have somewhere to be heard)
i) Why we like fed courts:
a. Lifetime tenure, salary, more competent
b. We dont trust state courts, theyre elected, might resent federal regulation, etc
5. Diversity Jx and D.C.---Tidewater case: statute authorized federal district courts to hear suits
between citizens of DC and citizens of other states (similar to diversity suits) even though no federal
question was presented. Plurality upheld statute, holding that DC is not a state, but Article I confers
power on legislature to set jurisdiction of lower courts. 7 justices voted to overrule old case that said
DC doesnt count for diversity jurisdiction.
i) Concurrence: DC is a state, court should've overruled precedent, but Article III sets jurisdictional
bounds (agreed w/ upholding of statute).

19

Downloaded From OutlineDepot.com

Kannar Federal Courts, Spring 2011

ii) Dissent 1: DC is NOT a state, and Article III sets jurisdictional bounds. Statute invalid.
iii) Dissent 2: DC is NOT a state, and Article III sets jurisdictional bounds. Statute invalid.
B.

CONGRESSIONAL RESTRICTION OF U.S. SUPREME COURT


APPELLATE JURISDICTION:
1. Supreme Court shall have appellate jurisdiction, both as to Law and Fact, with such
Exceptions, and under such Regulations as the Congress shall make.
i) Stripping is Rare: Because Congress rarely has attempted such jurisdiction stripping
and never in a manner that has been interpreted as precluding all Supreme Court
review the constitutionality of jurisdiction stripping remains uncertain.
2. Textual Reasoning in Favor of Stripping: The Framers of the Constitution intended this
type of congressional control as a check on the judiciarys power. Evidence of this intent is
found in the fact that the first Congress did not vest the Supreme Court with appellate
jurisdiction over all of the types of cases and controversies enumerated in Article III.
3. To be Constitutional, Supreme Court Must Retain Some Form of Appellate Review: Any
continuing basis for Supreme Court review no matter how unlikely is sufficient to make
a restriction on jurisdictional constitutional.
4. Ex Parte McCardle: Although the Supreme Court derives its appellate jurisdiction from
the Constitution, the Constitution also gives Congress the express power to make
exceptions to that appellate jurisdiction: Critic of reconstruction was arrested by feds, filed
habeas petition. SCOTUS had app JX over state habeas petitions pursuant to 1867 law. But
while the case was pending, Congress amended the law to strip SCOTUS of appellate jx over
habeas petitions, w/ intent to remove McCardle case from docket. SCOTUS said this was
permissible.
i) Habeas petition. McCardle was an editor & was arrested by fed. military authorities.
appeals to S.C. & gets oral argument. Congress passes a bill that strips S.C.s power to
hear habeas petitions. Ct says they dont have jurisdiction to hear the appeal but preserve
their power of original jurisdiction.
ii) Judge Story argued that Sheldon would be wrong if it ever came down.
iii) Rebuttal: although SCOTUS lost app jx over habeas petitions, Court could have heard
original habeas petition. McCardle was also a federal prisoner, meaning Court could've
heard claim under 1789 Judiciary Act.
iv) BUT see KLEIN, below (separation of powers)
5. Habeas jurisdiction stripping constitutional b/c retained original jurisdiction over habeas
(Felker v. Turpin) (1996) : AEDPA prohibited prisoners from bringing successive habeas
petitions unless pre-approved by a U.S. Court of Appeals. The Court emphasized that the law
did not preclude all Supreme Court review because the law did not repeal the Courts
authority to entertain original habeas petitions (even though thats SUPER unlikely)
(297) Statute was phrased very broadly. Some thought it presented the question of
whether McCardle was still good law.
Ct found a way of reading the statute to say they didnt have to confront the question
of whether congress could strip judiciary of habeas power. What if Congress takes
away all fed. jurisdiction (Sheldon power to the extreme & McCardle to the extreme).
6. Hamdan v. Rumsfeld (2006) (Supreme Court review of procedural issues in DTA, avoids
jurisdiction stripping question): Involved jurisdiction-stripping provision in the Detainee Treatment
Act of 2005. Although the majority found it unnecessary to reach the argument about whether
jurisdiction was stripped, Justice Scalia in dissent argued that there could be no doubt the statute in
question stripped jurisdiction as it gave the D.C. Circuit exclusive but limited jurisdiction to review
final decisions of military tribunals with an opportunity for review of the decisions in the Supreme

Court.

20

Downloaded From OutlineDepot.com

Kannar Federal Courts, Spring 2011

7. Troy Davis case: Court granted original jurisdiction on habeas claim (b/c no appellate jx
available), then relegated the case to a district court, ordering it to examine Davis's claim of
"actual innocence" based on new evidence.
C.

CONGRESSIONAL POWER TO WITHDRAW ALL FEDERAL


JURISDICTION:
1. At least when dealing with Indian Tribes, Congress does have the power to pass a federal
law which withdraws all federal jurisdiction and vests such jurisdiction in tribal courts.
i) Santa Clara Pueblo v. Martinez (1978): The Supreme Court held that federal courts possess no
jurisdiction over suits to enforce the federal Indian Civil Rights Act, the express purpose of which
is to protect individual Indians from arbitrary and unjust actions of tribal governments by
imposing limitations on an Indian tribe in the exercise of its powers of self-government.
Although the suits arise under federal law, enforcement actions can be filed only in tribal courts,
and there is no possibility of Supreme Court review.

ii) U.S. v. Klein: Disguising Impermissible Separation of Powers Violation as Stripping:


Congress passes law after Civil War that gave fed cts jurisdiction to hear Claims to give
back property to people the President Pardons. Then pass law taking away jurisdiction.
Really they were just using jurisdiction as a cover for the goal to do something
unconstitutional, denying the Presidents pardon power from its proper effect, which is a
separation of power violation. Had to protect the constitutional power of the Executive.
2. Professor Amar: distinguishes b/w all cases and controversies, Congress can limit
jurisdiction over controversies. Congress has consistently limited federal jurisdiction (amt in
controversy, defendants cant remove if sued in their home state, requiring complete
jurisdiction) Class Action Fairness Act: Congress used diversity jurisdiction, says in state
removal bar doesnt apply, only requires minimal diversity not complete. Its not that
convincing that stuff falling under cases is more pressing than the controversies stuff.
3. Professor Sagar: Constitution requires original or appellate federal jurisdiction of
constitutional claims: where state judges are elected, there is a danger that populism will put
someone on the bench that wont protect Constitutional rights.
D.

CONGRESSIONAL PRECLUSION OF BOTH STATE AND


FEDERAL COURT JURISDICTION:
1. The Court will go out of its way to read statutes so that they do not foreclose all judicial
review.
i) Due Process Requires Review: If a statute cannot be interpreted as such, the Court will
strike it down as an unconstitutional restriction preventing all courts from hearing
claims. Due Process requires that some court state or federal must be available to
hear claims.
a. Battaglia v. General Motors Corp. (2d. Cir. 1948) (Congress cannot limit jurisdiction as a
means of violating Due Process): The Supreme Court had held that employees were allowed
to consider as part of their work week time spent walking to their work stations, washing after
work, and changing clothes. Congress responded by adopting a statute, the Portal-to-Portal
Act, specifying that time spend on such activities did not could as part of the work week. Act
also provided that no court in the United States, or any State, shall have jurisdiction to
enforce liability for failure of the employer to pay for work time spent on such activities.
Congress passed a bill that stripped cts of jurisdiction (fed & states) to enforce liabilities f a
Fair Labor statute. 2nd Cir. Held: The Second Circuit indicated that Congress could not
restrict jurisdiction in a manner that prevented all courts from hearing claims. While
Congress has the ability to restrict jurisdiction, it may not do so as to deprive life, liberty, or
property without due process of law or to take private property without just compensation.
Ct. says the action of Congress was constitutional b/c of its Commerce Power.

21

Downloaded From OutlineDepot.com

Kannar Federal Courts, Spring 2011

b. Hart (310)Congress wont strip all remedies, there will always be something left.
Congress never has & never can close all of the cts. At the end of the day, every citizen could
manipulate (get arrested) have it tested by the S.C. (challenge all these things by habeas
corpus).
c. Suspension Clause (314)note 3 (315) St. Cyr CaseCongress suspension clause power
allies under executive power.
d. Rusal v. Bushdoes jurisdiction extend to Guantanamo? Yes b/c status of enclave &

scope of American Legal system. Legislature then writes DTA & MCAto try to
remove Art. III review of an enemy combatant by a non-Art. III agent. Cant get a
review of the system that is doing the detaining (get executive review).
e. Boumediene v. Bush: Suspension clause. Invalidate parts of the statue for infringing
on some of their jurisdiction.
ii) Constitutional Avoidance: If one acceptable construction of a statute would raise serious
constitutional problems i.e. foreclose all federal and state judicial review and if an
alternative interpretation of the statute is fairly possible, the Court will construe the
statute as to avoid such problems.
a. Webster v. Doe (1988) (preclusion of constitutional claim review must be explicitly
provided): Fired CIA employee who alleged that his termination was the result of
discrimination based on sexual orientation in violation of the Administrative Procedures Act
& Const. Held: National Security Act precluded review under the APA. However, the Court
held that the Act did not preclude review of constitutional claims (as opposed to the
procedural claims). The Court reasoned that Congress should not be taken to have intended to
preclude constitutional claims unless it has explicitly so provided.
b. Immigration and Naturalization Service v. St. Cyr (2001) (statutory jurisdiction
preclusion did not bar habeas filing): The express statutory preclusion of judicial review of
deportation proceedings did not bar a challenge from being brought through a writ of habeas
corpus. Preclusion of direct judicial review of INS deportation orders ok but statute did not
bar habeas corpus proceedings. Have to clearly state they are precluding review of a
Constitutional Right to take away that jurisdiction.
i. But SeeReal ID of 2005: Took habeas jurisdiction away from federal courts as to
immigration cases but provided alternative limited review of removal decisions (remedial
measures) including constitutional claims and questions of law.
c. Kucana v. Holder: congress said court cant review stuff thats up to the AG's discretion per
the statute. Then AG passes regulation that gives him discretion. SCOTUS says the lack of
jurisdiction is ok.

iii) Intersection of Sovereign Immunity:


a. Post-payment remedy may be denied if pre-payment remedy available, and vice versa
b. Denial of recovery of taxes obtained through compulsion violates 14A if promise a
post-deprivation remedy, due process requires the state to provide it (Reich v. Collins)
must give taxpayer a remedy at some point.
c. May be out of court if seek the wrong remedy; as long as some remedy, may not
matter if its not the remedy you want
E.

CONGRESSIONAL APPORTIONMENT OF JURISDICTION


AMONG FEDERAL COURTS:
1. Congress has unquestioned power to allocate jurisdiction over particular kinds of cases
among various federal courts. Furthermore, Congress may divide up the original and
appellate jurisdiction of the inferior federal courts in whatever way it deems best.
i) Specialized Courts
Emergency price control act of 1942pushed things to the limit of federal
jurisdiction. The Act involved price controls during WWII, & set prices (some
are bound to be unhappy). A price administrator set the prices for the country &
Congress set up Emergency Ct of Appeals (only ct ppl could go to after the claim

22

Downloaded From OutlineDepot.com

Kannar Federal Courts, Spring 2011

was rejected by the administrator. Other than this ct, there was no other judicial
review & no interlocutory relief.
ii) Lockerty v. Phillips (1943) (constitutional allocation of injunctive relief under Federal Price
Control Act): During WWII, Congress created an Article III Emergency Court with exclusive
power to provide injunctive relief against the enforcement of regulations promulgated under the
Federal Price Control Act. The Supreme Court held that the Emergency Court had exclusive
jurisdiction to grant injunctive relief to restrain enforcement of regulations under the Act or of the
Act itself. Lockerty was a meat dealer & sued in district ct, the case was dismissed b/c of the
Emergency Price Control Act. S.C. said it was allowed b/c it was not a constitutional violation
limiting jurisdiction. The Ct affirmed the dismissal.
iii) Yakas v. United States (1944) (constitutional allocation of a certain type of claim challenging
the Federal Price Control Act to an Emergency Court): A criminal prosecution was brought in
federal district court for violation of a price control regulation. The Price Control Act provided
that the legality of a regulation could not be raised in the district court as a defense in a criminal
prosecution. Held: The Court sustained this restriction on the district courts jurisdiction on the
ground that the defendant could have challenged the regulation by brining an earlier and separate
suit in the Emergency Court. Having failed to avail himself of an adequate separate procedure,
defendant may be barred from raising the issue as a defense in the district court criminal
procedure.

If you are prosecuted under the Price Control Act, can you raise it as a defense
that its unconstitutional?No, youre obligated to follow the rule, cant violate
the rule. 6th A rights are still present in a criminal case. You can challenge the
Act.
Dissentdispute as to whether Congress can provide jurisdiction in enforcement
proceedings. Issue w/separation of powers.

iv) Falbo & Estep (violation of the Act is a crime resulting in incarceration). Falbodoesnt show
up for the draft. Estep gets convicted for rejecting the decision of the draft bd. Hart said you go to
jail & bring a habeas claim.
v) US v. Mendoza-Lopez

Issue: whether is allowed to challenge the validity of a deportation order. Can


he re-litigate the defects of the order?
Ct talks about Yakus against & says he was allowed to challenge the order
(unlike Yakus). It doesnt overturn Yakus b/c it shouldnt be read as a generative
case b/c Yakus took place during the war.

F. Fed. Authority & State Ct Jurisdiction (383)


Tafflin v. Levitt (384)
o Facts: After the failure of a savings and loan association and the collapse of a
state-chartered nonprofit corporation created to insure the savings and loan
accounts, petitioners brought a civil action against respondents in federal court
alleging violations of the Racketeer Influenced and Corrupt Organizations Act
(RICO).

o
o

Issue: whether a state ct has jurisdiction over a civil RICO claim


The opinion gives the black letter law. How do we know if jurisdiction
is concurrent, exclusively state or exclusively fed.?
States have concurrent jurisdiction to hear fed. claims unless
Congress says otherwise. Congress has the power to structure &
target jurisdiction.
Evidence of intent of Congress to disturb concurrent jurisdiction:
(1) explicit statutory directive; (2) legislative history; & (3) clear
incompatibility b/w state ct jurisdiction & fed. interests.

23

Downloaded From OutlineDepot.com

Kannar Federal Courts, Spring 2011

o
o
o
o
o

(1) Ct says mere grant of jurisdiction doesnt say fed. jurisdiction, it does
nothing to displace state jurisdiction.
(2) Review legislative historyCongress never really discussed it.
Petitioner said language in statute is from the Clayton Act which
exclusively gave jurisdiction.
Ct said language doesnt mean anything. Not an unmistakable part of the
legislative history.
(3) Look to see how complex RICO claims are. S.C. said many RICO
claims have state claims, state judges have the ability to handle these
claims.
Holding: a RICO action could be instituted in a state court and that the state's
comprehensive scheme for the rehabilitation and liquidation of insolvent statechartered savings and loan associations provided a proper basis for the federal
court to abstain. The Supreme Court held that state courts had concurrent
jurisdiction over civil actions brought under RICO.

Scalia & Kennedy Concurringdisputes state ct jurisdiction conferred


by Congress. No one confers sovereignty on the states, the states just
have it & have jurisdiction. Legislative history test is unreliable &
shouldnt be used. The real test should be clear legislative directive. The
3rd test has no foundation. #2 is on the books but ct never uses it.

Congressional Policy: possible choices that Congress may have: could exclude fed. cts; concurrent;
concurrent but some right for to remove to fed. ct; & exclusive fed. jurisdiction.

Exclusive State Original Jurisdictionno Art. III power.


Concurrent Jurisdiction w/rights of removal: has the option
Exclusive Fed. jurisdiction: what the RICO case was NOT about.

Tennessee v. Davis
o Facts: The defendant was a deputy tax collector whose duties included
seizing illicit distilleries. While seizing one of the stills, the defendant
was fired upon by several men. The defendant fired back and killed one
of the men. The defendant was arrested and indicted for murder. The
defendant sought to have his state criminal case removed to a federal
court.
o Issue: whether the defendants case was transferable to the Supreme Ct.
o Holding: Constitution authorized the removal of civil and criminal cases
from state court to federal courts, and that upon removal of a case from a
state court to federal court, the federal court administered state laws to
determine the outcome of the case.
Note of 396-97 IMPORTANT!!Orients to removal
o Mesa v. CA (398)fed. official doesnt automatically get case moved to
fed. forum. Need a fed. question, diversity jurisdiction or US being a
party.
Tarbles Case (398)
o Facts: The soldier apparently enlisted in the United States Army under a
different name when he was under the age of 18. The soldier was being
held in custody and confinement by an Army lieutenant under charges of

24

Downloaded From OutlineDepot.com

Kannar Federal Courts, Spring 2011

desertion. The father filed a petition for habeas corpus. State Ct grants
the writ.
o Holding: the commissioner lacked jurisdiction to issue the writ of habeas
corpus because the soldier was held by an officer of the United States,
under the authority of the United States. The Court found that within the
territorial limits of each state, there were two spheres of government, the
state government and the federal government. Both were separate and
distinct, except that the United States was supreme when any conflict
arose. There was no jurisdiction.
o Chief Justice Chase dissentsdenying the states the right to issue writs
would deny the right to protect the citizens by habeas corpus against
arbitrary imprisonment.
Testa v. Katt (408)
o Facts: Respondent car dealer sold petitioner buyer an automobile at a
price above the ceiling price. Petitioner filed an action against respondent
for violation of 205(e) of the Emergency Price Control Act. The
highest state ct said they didnt have to enforce the Act.
o Justices Blacks response: US is NOT a foreign country as compared
to the state & under sovereign immunity clauseit forces states to
enforce the Act. When its concurrent it concurrent, but there could be
pre-existing procedural things in a state that might need to be recognized
by fed. as a departure (valid excuse doctrine).

Non-Discrimination Principlehave to take the fed w/the state. Fed. Civil Rights claims seem to have
a more favorable status. Exception to non-discriminationmay have a valid excuse to decline
jurisdiction (ex. if there is a valid state procedure). May encourage forum shopping b/c cases may be
treated differently in state vs. fed. ct.
Herb v. Pitcairnfile fed. claim. 2 yr statute of limitation. Neutral application
of transfer of venue. Statute expired & ct affirmed the dismissal.
o But see Haywood v. Drown (pg. 18 Supp.)--1983 claim, its not a
validly excused difference.
o Felder v. Casey--1983 claim (public official committed a tort). Notice
of claim requirement. Ct held it s not a valid excuse.
Dice v. Akron, Canton & YoungstownFed. employee liability. 3rd point
disparity on how parts of a case are treated in OH system (would be treated
differently in fed. system). Question of whether judge or jury should hear the
case (its answered differently in fed. & OH state cts). Frankfurter black letter
law. Cant refuse fed. claims under this statute. But not obliged to treat it the
same as would fed. system. Question of fraud must be answered by a jury. The
issue was whether states may follow their procedural roles in entertaining fed.
rights.
Johnson v. Fankell: 1983 case. Can you get an interlocutory appeal?Fed. ct
does, but does state have to?NO.
Jinks v. Richland Country (429)the statute of limitations expired. Congress
passed a law tolling (extending the claim in state ct). Ct dismissed the claim but
extends the state law. Ct used the necessary & proper clause to uphold the law.

NON-ARTICLE III TRIBUNALS (Ch 4 Sec. 2)


25

Downloaded From OutlineDepot.com

Kannar Federal Courts, Spring 2011

A. Article III Court = subject matter authorized by Article III, independent judges, judgments are
final.
B. CONGRESSIONAL POWER TO CREATE LEGISLATIVE COURTS :
Throughout history, Congress has created tribunals in which the judges do not have life tenure
and protected salary to decide cases and controversies enumerated in Article III. These tribunals
are termed legislative courts or Article I courts.
1. Reasoning: Congress might want to avoid establishing a large number of additional
judgeships to deal with the countless matters handled in administrative agencies and in
specialized tribunals like bankruptcy courts. Congress might want to allow agencies that also
possess rule-making and investigative powers to decide particular controversies within their
expertise.
i) Public Rights vs. Private Rights: public rights claims (which, at minimum, feature govt
as one of the parties) can be assigned to non-art III judge. Wrt private rights claims, cong
cannot wholly preclude judicial consideration
C. Current Law Summarized: there are four situations in which legislative courts are
permissible
1. for United States possessions and territories;
2. for military matters
3. for civil disputes between the United States and private citizens (e.g. Court of Federal
Claims, Tax Court, Court of Veterans Affairs); and
4. Non-Article III Courts as Adjuncts in criminal matters or private disputes, where fed court
can review the legislative courts decisions. To be an adjunct, cant enforce its own judgment
and there must be de novo review by Art. III courts.
i) Mandatory (but Limited) Article III De Novo Review: Article III courts must be able
to decide de novo all questions of law, constitutional facts those facts that are the basis
for a claim of a constitutional violation and jurisdictional facts those facts that are the
basis for the agencys legal authority to hear the matter.
a. Crowell v. Benson (1932) (private law disputes can be decided in non-Article III
courts only if there is de novo review by an Article III court): Claim under workers
comp act Held: The Court held that in private law matters, ultimate decision-making
authority must rest in Article III courts. Legislative courts, such as the Employees
Compensation Commission, could resolve private law disputes only if there was
substantial oversight by an Article III court. The Court said that Article III courts
must be able to decide de novo all questions of law, constitutional facts, and
jurisdictional facts. All may be re-litigated de novo in Article III ct.
Is the scope of or limitation on the powers of Congress to establish these cts
(under their Art. I power). Crowelldeputy commissioner ordered Benson
to pay Knudsen. Benson says some commissioner cant tell him to pay; only
ct can tell him to pay. Is Benson right?
Ct says due process cant guarantee Art. III process from beginning to end.
Pg 326public rights vs. private rights: public rights involve interest of the
gov. & private rights involve going after another party. Congress ability to
interfere w/Art. III rights is much more limited.
Its not a public rights case. Ct says its okay for Congress to set up a scheme
to go to a factfinder to determine liability. (Brennan says thats not fair b/c
he didnt get a trial). Hughes says you dont get a trial for liability but could
get a trial if the employee worked for him (its a statutory question).
Brandeis dissents: he suggests Congress can displace Art. III power more
than Hughes did. Congress should give more power to regulatory agencies.

26

Downloaded From OutlineDepot.com

Kannar Federal Courts, Spring 2011

ii) Pg. 332 note 4(a) &(b) Public Rights & Judicial Review: another route Benson could
have used . In practice, there might be more options to Benson than the black letter
suggests. 334-5: cts develop what is going to be a fact. Gradually reducing scope of Art.
III power to these administrative agencies. Gradually reducing scope of judicial review.
Pg. 342(4)(b)basis for distinguishing the enforceability of judgments.
iii) Findings of Fact Conclusive: Administrative findings of non-constitutional and nonjurisdictional facts may be made conclusive upon the courts, if not infected with any error
or law, as a basis for judicial enforcement of a money liability of one private person to
another.
iv) Jurisdictional Fact Doctrine Abandoned: The jurisdictional fact doctrine is no longer
followed and has seldom been mentioned since the Courts decision in Crowell.
Independent judicial fact-finding rather than redetermination of facts on the
administrative record virtually never occurs.
v) Chevron Deference to Administrative Decisions of Law: A two-part analysis was born
from the Chevron decision where a reviewing court determines (1) whether a statute
permits or forbids an agency's interpretation, and (2) if a statute is not clear on step (1),
then the court decides whether the agency's interpretation of a statute is reasonable or
permissible. If an agency's interpretation is reasonable, then the court will defer to the
agency's reading of the statute.
vi) Categorical Approach of Northern Pipeline: The only case invalidating congressional
employment of legislative courts is Northern Pipeline, which struck down the jurisdiction
vent to non-Article III federal bankruptcy judges after the Bankruptcy Act of 1978 which
gave bankruptcy judges broad discretion to decide private civil disputes (a contract claim
in this case). The Supreme Court held this authority violated Article III.
a. Involved new bankruptcy rules, law set up a bankruptcy ct. Ct declares the law
unconstitutional (Burger helped draft the law). Brennan determines some aspects of
bankruptcy as private rights (some have to come before an Art. III ct). The law took
all power away from Art. III cts. Congress then redrafted the law. Brennan takes a
hard-line approach.
b. Congressional Response: Following Northern Pipeline, Congress amended the
Bankruptcy Act to make the bankruptcy courts adjuncts to Article III courts when
they adjudicate state law matters.
c. There was a change in the cts makeup (pg. 357)by the time you get to Thomas &
Schor, ct comes around to a view different from Brennans the proper view is
balancing test that comes to govern these things going forward.
D. Current Law Balancing Approach : Where the benefits of using legislative court in
terms of efficiency and expertise outweigh concerns about fairness to litigants and separation of
powers, legislative courts may be constitutionally used.
i) Legislative Courts allowed when Private Law Disputes Closely Related to
Government Regulatory Programs: (Thomas v. Union Carbide Agricultural
Products (1985)) (private rights is created and integrated into a public regulatory
scheme): In determining cost sharing for environmental study data, Congress shifted the
task of valuation from EPA to a system of negotiations and binding arbitrations. Judicial
review was limited to instances of fraud, misrepresentation, or misconduct. Held: The
public nature of the regulatory scheme and the public interest served by the arbitration
procedure was key. Matter handled by a government agency and was not a substitute for
any existing common law proceeding.
a. Reasoning: Congress, may create a seemingly private right that is so closely
integrated into a public regulatory scheme as to be a matter appropriate for agency
resolution.

27

Downloaded From OutlineDepot.com

Kannar Federal Courts, Spring 2011

b. Appellate Review Avoided Separation of Powers Concerns: appellate review,


albeit limited in scope, was provided and there was no indication that the arbitration
system threatened Article III courts or implicated separation of powers concerns.
ii) Balancing in Favor of Non-Article III Court:
a. Waiver to Proceedings and Fairness to Litigant: If a litigant consented to
proceedings of a non-Article III court as an alternative to federal court litigation, he
cannot claim the adjudication is inherently fair.
b. Separation of Powers: The Court has declined to adopt formalistic and unbending
rules; instead it focuses on several factors none of which is likely to be
determinative of constitutionality including:
i. The extent to which the essential attributes of judicial power are reserved to
Article III courts, and conversely, the extent to which the Non-Article III forum
EXERCISES the range of jurisdiction and POWERS NORMALLY VESTED
only in ARTICLE III COURTS,
ii. The ORIGINS AND IMPORTANCE of the RIGHT TO BE ADJUDICATED,
and
iii. The CONCERNS that drove Congress to DEPART from the requirement of
ARTICLE III.
iii) Commodity Futures Trading Commission v. Schor (1986): Commission had the
authority to provide reparations to individuals who are injured by fraudulent or illegally
manipulative conduct by brokers. Commission promulgated regulations that enabled it
to hear all counterclaims arising out of the same allegedly impermissible transactions.
Held: The commissions authority is quite similar to that traditionally exercised by
agencies, except for the jurisdiction to hear counterclaims. Any intrusion on the Judicial
Branch was de minimus.
a. Because the commission could not enforce its own orders it served as an adjunct to
the federal court
b. As to fairness, the Court said that the defendant had consented to the admin
proceedings as an alternative to federal court proceedings.
E. Seventh Amendment Right to Jury Trial: A jury trial must be provided in a non-Article III
tribunal when the relief sought is of a legal nature, such as money damages, and the matter
involves a private right. But, the Supreme Court has never addressed the question whether it
violates Article III for Congress to authorize Article I courts to conduct jury trials.
1. Limitation Equitable Relief: In NLRB v. Jones & Laughlin Steel (1937), the Court stated
that the Seventh Amendment has no application to cases where recovery of money damages
is an incident to equitable relief even though damages might have been recovered in an action
at law.

MILITARY TRIBUNALS (Ch 4 Sec. 2)


A. Military Tribunals Generally: Distinct from the courts-martial employed to try members of the
U.S. military are military tribunals or commissions that have been constituted from time to time
by the Executive Branch, typically pursuant to express or tacit congressional authorization, to
deal with exigencies associated with war.
1. The judges of military tribunals are typically military officers.
2. Where military commissions are permitted, the full safeguards of the Fourth, Fifth, and
Sixth Amendments apparently do not apply of their own force.
3. Typically Used Abroad: Though most used abroad in connection with military occupations
of foreign territory military commissions have sometimes been used domestically as well.
4. Authority for Military Tribunals: Supreme Court precedent clearly establishes that the
Constitution permits use of military tribunals, under at least some circumstances, to conduct

28

Downloaded From OutlineDepot.com

Kannar Federal Courts, Spring 2011

trials for alleged violations of the law of war and for offenses in territory under military
occupation or subject to martial law.
5. Determining Enemy Combatant Status: Military commissions have also been used for the
distinct, non-criminal purpose of ascertaining whether a detainee is an enemy combatant in a
war against the United States who, under the laws of war, may be lawfully detained until
cessation of hostilities
B. Congress Can Prevent Pres from using military tribunals : Although it is not
clear to what extent the constitutional basis for employing military tribunals resided in the
Presidents Article II commander-in-chief power and to what extent in the conjunction of that
power with congressional powers under Article I
1. Court has held squarely that the President may not employ military tribunals in the face of an
express or implied congressional prohibition (see Hamdan v. Rumsfeld (2006)).
2. Military Tribunals in the United States: The leading decision involving the constitutionally
permissible use of military tribunals in the United States are not easily reconciled.
C. Cant try private citizens: Ex Parte Milligan (1866) (military commissions lacked
jurisdiction to try a U.S. citizen): The Court stressed that military tribunals couldnt try private citizen
because federal courts were always open to hear criminal accusations and redress grievances Military
authorities in Ind. Wanted to lock ppl (confederates) up for making trouble. The Ct upheld the decision that
a military tribunal lacked jurisdiction to try a US citizen.

D.

Can try unlawful combatants: Ex Parte Quirin (1942) (military commissions used
in U.S.; distinction between lawful and unlawful combatants): Eight German saboteurs came on shore,
took off their uniforms, tried by military commission. Held: The Court noted that Congress by statute
and the President by proclamation had authorized the use of military tribunals. Constitutional b/c the
Court drew a distinction between lawful and unlawful combatants.
1. Lawful combatants are subject to capture and detention as prisoners of war, but unlawful combatants
can be subjected to trial and punishment by military tribunals for acts which render their belligerency
unlawful
2. German service members spying (w/o uniforms) were picked up & tried before military tribunal &
S.C. upheld the military tribunals decision. Ct distinguished Milligan b/c petitioner (claimed
American citizenship) the law of war can never be applied to citizens of the state which have upheld
the authority of gov. where cts are open & their process unobstructed.
3. In times of war, the law is silent.

E.

U.S. citizens are not immune from enemy combatant designation and
therefore trial before a military tribunal, but some due process applies (Hamdi).
i) Hamdi v. Rumsfeld (2004) (U.S. citizens enjoy no greater immunity than foreigners if captured
as an enemy combatants): Quirin postdated and clarified Milligan by establishing that an
American citizen enjoys no immunity from punishment or detention as an enemy combatant that
would otherwise be permissible under the laws and usages of war.
US citizen on US soil being held as an enemy combatant. Cts narrow holdingan
American citizen enjoys no immunity from punishment or detention as an enemy
combatant that would otherwise be permissible under laws & usages of war.
Question if he was properly put into that matter is an Art. III matter.
Scalia & Stevens dissent thinking the case was like Milligan.

F.

Military Tribunals Abroad to Try Defendants Abroad: Military tribunals were


used abroad, especially during WWII. The use of such tribunals appears to have been predicated
on one of two bases:
1. (1) an occupying power could use military tribunals to try ordinary criminal offenses until
domestic civil government was restored; or
2. (2) military tribunals were appropriate to try alleged violations of laws of wars, even after
termination of hostilities.

29

Downloaded From OutlineDepot.com

Kannar Federal Courts, Spring 2011

G. Judicial Review and Habeas Corpus Relief of Military Commissions: Historically, Article III
courts have not typically had statutory jurisdiction to engage in appellate review of the decisions
of military tribunals.
1. However, the federal courts, in the exercise of their habeas corpus jurisdiction, have been
able to inquire whether the Constitution or laws of the United States withhold authority to
proceed with a trial.
i) U.S. Citizen w/in United States: Insofar as a petitioner whether a citizen or a noncitizen seeks to challenge the use of a military tribunal within the United States, habeas
corpus review is protected by the Constitution itself.
a. The President lacks unilateral authority to suspend habeas corpus w/in the United
States.
ii) Foreign Citizens on Foreign Soil: When military tribunals are used on foreign soil to try
foreign citizens who are not subsequently imprisoned in the United States, there is no
guarantee of habeas corpus relief as the federal courts lack jurisdiction. However, the
D.C. Circuit court does have jurisdiction where the prisoner or custodian is on foreign
soil which is the plenary and exclusive jurisdiction of the U.S.
a. Prisoner or Custodian Jurisdiction Required: The statutory grant of the habeas
statute gives judges the power to issue writs within their respective jurisdictions.
The Court has understood this to mean that habeas relief is limited to either where a
prisoner or his custodian is within the jurisdiction of the court.
b. Braden v. 30th Judicial Circuit (1973): Held that a court cannot exercise habeas jurisdiction
without territorial jurisdiction over the petitioner. However, the habeas statute requires only
jurisdiction over the custodian.
i. E.g. The Court has assumed that federal courts in the District of Columbia may inquire
into foreign detention and trials by court martial of U.S. service members based on their
jurisdiction over the Defense Department officials within the District.
a. But See Rasul v. Bush (2004): The Court upheld the jurisdiction of the district
court for the District of Columbia to consider challenges to the legality of the
detention of foreign nationals captured abroad and held for a protracted period
of time at the United States Naval Base in Guantanamo Bay, Cuba, over which
the United States Exercises plenary and exclusive jurisdiction, but not
ultimate sovereignty.

H. Military Commissions and the War on Terror: In Hamdan, the Supreme Court ruled that the
military commissions set up by the Bush administration to try detainees at Guantanamo Bay
violated both the Uniform Code of Military Justice and the four Geneva Conventions.
1. Hamdan v. Rumsfeld (2006): Hamdan designated him an enemy combatant. A few months later, the
district court granted Hamdan's habeas petition, ruling that he must first be given a hearing to
determine whether he was a prisoner of war under the Geneva Convention before he could be tried by
a military commission. Held: The military commission convened to try Hamdan lacks power to
proceed because of its structure and procedures violate both the Uniform Code of Military Justice and
the Geneva Conventions.
i) Neither an act of Congress nor the inherent powers of the Executive laid out in the Constitution
expressly authorized the sort of military commission at issue in this case. Absent that express
authorization, the commission had to comply with the ordinary laws of the United States and the
laws of war. The Geneva Convention, as a part of the ordinary laws of war, could therefore be
enforced by the Supreme Court, along with the statutory Uniform Code of Military Justice.
Hamdan's exclusion from certain parts of his trial deemed classified by the military commission
violated both of these, and the trial was therefore illegal.

ii) Jurisdiction and the DTA: The Court said that the Detainee Treatment Act of 2005
which gave the DC Circuit exclusive jurisdiction to review decision of cases being
tried before military commissions did not preclude its jurisdiction. This is because the
effective date of the DTA did not apply to pending cases (they punted here).

30

Downloaded From OutlineDepot.com

Kannar Federal Courts, Spring 2011

iii) Authority for Commissions: Court didnt decide whether President had Constitutional
power to convene military commissions like the ones create to try Hamdan b/c Congress
had given authority under various laws. These laws acknowledge that the President has
the power to convene military commissions for the exigencies of war, but such tribunals
must operate within the laws of war including UCMJ and Geneva Convention.
iv) Violation of Laws Applied: The Court found that the procedures of the military
commissions violated the laws of both the UCMJ (e.g. defendant was forbidden from
viewing certain evidence; evidence with any probative value admitted regardless whether
hearsay) and the Geneva Conventions (not a regularly constituted court).
2. Military Commissions Act of 2006: After Hamdan, MCA established procedures governing
the use of military commissions to try alien enemy combatants for violations of the law of
war and other offenses triable by military commission. There was controversy whether it
effects habeas corpus for United States citizens.
i) Procedures: MCA provides some due process rights e.g. right to be present but it had
procedures that differed from traditional criminal due process e.g. admissibility of
hearsay evidence, confidential security information, degree of guilt, etc.
ii) Limited Application of Geneva Conventions: MCA limits an enemy combatants
ability to invoke Geneva Conventions i.e. no alien unlawful enemy combatant may
invoke the Geneva Conventions as a source of rights. Also, the MCA states explicitly
that a military commission is a regularly constituted court as required by Geneva
Convention Common Article 3. Finally, the Act gives the President broader discretion to
determine and/or interpret the United States obligations under various treaties.
iii) Very Narrow and Limited Appellate Review: DC Circuit had exclusive appellate
jurisdiction only to review whether the final decision was consistent with the standards
and procedures of the MCA or, to the extent applicable, the Constitution and laws of the
United States. Also, combatant status is foreclosed issues of fact.
3. Boumediene: Held that the MCA was an Unconstitutional Suspension of the writ, aliens held
at Guantanamo still get habeas as it was defined in 1789, and Guantanamo is de facto U.S.
territory where U.S. law still applies, dont want the executive to avoid Habeas just by
moving bases around.

Law Applied in District Courts

539-547historical info
557-565 Federal courts lack power to make federal law in the absence of explicit or implicit
authorization by Congress or the Constitution. In Erie v. Tompkins, the US Supreme Court
held that federal common law can exist in areas governed by the federal constitution or by acts of
Congress. In other areas, though, Congress should not displace state law because that would
encourage forum shopping.
o The Erie holding overturned Swift v. Tyson which held that there was a transcendental
omnipresence common-law scheme where the federal courts could displace state
common law (except where the state had a statute or a local (property) law).
In Swiftfed. cts when exercising jurisdiction in diversity cases, cts dont have
to look to high state ct, can apply general principles of law to create law. Ct look
at general principles. This creates forum shopping concerns.
o The Erie holding was based on the ideas that: (1) there was a new interpretation of
Section 34 of the Judiciary Act of 1789; (2) this would create greater uniformity than
Swift; (3) this would ensure equality between citizens and non-citizens (in terms of
removal rights); (4) this would equalize the differences between plaintiffs and defendants.

31

Downloaded From OutlineDepot.com

Kannar Federal Courts, Spring 2011

It is the current law. The theory is that fed. cts are becoming hostile to state law, Brandies
has a political agenda (right time to overturn Swift). It was a personal injury case for a
RR accident. S.C. said Swift was wrong & unconstitutional. Congress establishes the
laws but not substantive riles, common law of states applicable (cts can create substantive
law, they must look at applicable state law had the case been brought in the state). Also
can have manipulation of forum shopping, no uniformity, Swift was based on a poor
reading of the statute.

STATUTORY LIMITATIONS ON FEDERAL COURT


JURISDICTION
A.

CONGRESS CAN STATUTORILY REGULATE


LIMITATIONS ON FEDERAL JURISDICTION:
1. Congress has broad powers to determine the authority of the federal courts, especially in
relation to state judiciaries. It would violate separation of powers for the courts to disregard
the statutes unless they were unconstitutional.
i) Federal Courts:
a. May enjoin proceedings in another federal court
b. May sometimes enjoin proceedings in a state court. But, possible hurdles include:
i. Anti-Injunction Act
ii. Tax Injunction Act (re taxes under state law)
iii. Johnson Act
iv. Abstention Doctrines
ii) A state court
a. May enjoin proceedings in another state court (no fed statute preventing it)
b. May not enjoin proceedings in a federal court (except under the res exception)

B.

THE ANTI-INJUNCTION ACT 28 U.S.C. 2283:


1. Anti-Injunction Act (AIA) provides that A court of the United States may NOT grant an
injunction to stay proceedings in a State court EXCEPT as expressly authorized by Act of
Congress, or where necessary in aid of its jurisdiction, or to protect or effectuate its
judgments.
i) Purpose: prevent conflict between federal and state courts. The underlying idea is
that a federal injunction of ongoing state proceedings is likely to breed resentment and
hostility in the state judiciaries and even risk disobedience of the federal courts order.
ii) Prohibits Federal Injunctions: The AIA prohibits federal courts from enjoining state
proceedings:
a. directly by enjoining state courts or
b. indirectly by enjoining the parties from proceeding with litigation in state courts.
2. Proceedings Requirement:
i) AIA only applies if there are proceedings actually pending in state courts;
ii) Does not prevent federal courts from issuing injunctions in the absence of ongoing state
court litigation.
iii) The Meaning of Proceedings: The AIA is not applicable if state proceedings have yet
to be instituted. For example, in Lynch v. Household Finance (1972), the Court held that
a prejudgment garnishment was not a proceeding in state court and hence could be
enjoined by a federal court, even though the garnishment might be necessary to obtain
satisfaction of any subsequent judgment obtained by the creditor.
iv) Prentis v. Atlantic Coast Line (1908) (AIA does not apply to injunctions against state
nonjudicial proceedings): Federal circuit court enjoined enforcement of a rate order of the

32

Downloaded From OutlineDepot.com

Kannar Federal Courts, Spring 2011

Virginia State Corporation Commission. Applicants argued that under state law the commission
had the characteristics and powers of a court and that the Anti-Injunction Act forbade a federal
injunction. Held: The Court held that whatever the status of the commission in other types of
proceedings, the establishment of a rate is the making of a rule for the future, and therefore is an
act legislative and not judicial in kind, to which the AIA did not apply.

C.

Younger Adds a Hurdle: To get an Injunction, case must fit under an AIA exception

AND an exception to the Younger Doctrine : The importance of the AIA has been substantially
lessened by the Supreme Courts creation of a parallel abstention doctrine Younger Abstention
based on concerns for equity and comity. Although in Younger the Court held that federal courts
may not enjoin pending state court criminal prosecutions, this has been extended to a variety of
other situations.
D. Exceptions to Anti-Injunction Act: The exceptions contained in the AIA are
exclusive, and the Court may not create additional situations in which injunctions may be issued.
REMEMBER AN EXCEPTION TO YOUNGER IS ALSO NECESSARY TO GET AN
INJUNCTION!!
1. Injunctions Expressly Authorized by Statute (1983 Claims): Because Congress created
the bar against injunctions contained in the Anti-Injunction Act, Congress may override its
own limitations and expressly authorize stays of state court proceedings.
i) No Need for Specific Authorization: To be an express authorization of an injunction, a
statute does not have to specifically state that it constitutes an exception to 2283.
a. No prescribed formula is required; an authorization need not expressly refer to
2283.
b. Test: A statute need not even mention the possibility of injunctions of state
proceedings if the PURPOSES OF THE STATUTE would be FRUSTRATED if
injunctions were not allowed.
ii) Section 1983 comes within expressly authorized exception, thus allows injunctions Other
Statutory Exceptions: Injunctions under Bankruptcy Laws, Removal, Admiralty
limitation of liability, federal interpleader, habeas.
iii) (Clayton Act DOES NOT come within expressly authorized exception) (Vendo Co. v. LektroVend Corp.(1977)) : There was no indication that Congress was concerned with the possibility
that state court proceedings would be used to violate the Clayton Acts. To rule otherwise would
eviscerate 2238 since the logic of that position could mean that virtually all federal statutes
authorizing injunctive relief are exceptions under 2283.

2. Injunctions Necessary In Aid of Federal Courts Jurisdiction: Two circumstances:


Removal and Res.
i) Removal Exception: Case is removed from state court to federal court; and
a. If a case is removed to federal court and the state court does not relinquish
jurisdiction, the federal court may enjoin further state judicial proceedings.
i. However, there was little need for a separate exception for removal cases because
the Court has always regarded the removal statute as an express authorization for
staying state proceedings.
ii) In Rem Jurisdiction: Res Exception: Federal court first acquires jurisdiction over a
case involving the disposition of real property.
a. Whichever court first gains jurisdiction in a case concerning the disposition of
property has exclusive jurisdiction to decide claims to that property. Thus, whatever
court initially acquires in rem or quasi in rem jurisdiction over a matter involving
property can enjoin all other courts from hearing the matter.
3. Injunctions to Prevent Relitigation of an Issue: when injunctions are necessary to protect
or effectuate an earlier judgment by a federal court.
i) When a federal court decides an issue, it can prevent that same issue from being
relitigated in state court.

33

Downloaded From OutlineDepot.com

Kannar Federal Courts, Spring 2011

a. Prevents harassment of federal court litigants by repetitive state court proceedings


and ensures finality of the federal courts proceedings.
ii) Earlier Federal Decision Must have been on the Merits: A federal court may not
enjoin state court proceedings under the relitigation exception if the earlier federal court
ruling was based on federal court procedures and not on the merits of the case.
E. Other Anti-Injunction Act Coverage Issues:
1. AIA is NOT Applicable to Strangers to Original Lawsuit: In County of Imperial v. Munoz
(1980), the Supreme Court held that unless the AIA was not applicable to federal plaintiffs
who were strangers to the original proceeding.
2. Declaratory Judgments / State Permits: If the state suit is likely to turn on a question of
federal law with which a federal court is likely to be more familiar and experienced than the
state court, and if the state court abstains from ruling pending federal decision on that
question, the Third Circuit has held that 2283 does not preclude the federal court from issuing
a declaratory judgment on the common federal question. (Thiokol Chemical v. Burlington
Industries)
3. US a party: The AIA does not apply when the US is a party. This means that Administrative
Agencies arent blocked by the AIA from getting injunctions, watch out for Younger though.

FEDERAL AUTHORITY AND STATE-COURT


JURISDICTION
A. Art III structure premised on the existence of state cts & assumption that they would be first-line
interpreters of fed law (Art VI Supremacy cl.) States have the right and obligation to hear fed
law. Fed cts play role in policing state cts.
1. State courts also lack Art III protections.
2. Raises issues of parity and beliefs about competency of state vs. fed cts.
B. Federalist 82: states should retain all existing auth, unless expressly taken away or
incompatible to have jx in both.
1. Const leaves in place power of states as it existed before const, but can be changed.
2. Federal and state are part of whole, not right for state to regard fed as foreign
3. Natl asspt at time: courts hear claims that walk in door, doesnt matter if under law of diff
sov
C. Generally State Courts Have Concurrent Jurisdiction: Deeply-rooted
presumption in favor of concurrent state court jurisdiction.
1. Therefore, state courts have the authority to hear all cases including ones arising under
federal law or even foreign law Unless Congress has vested exclusive jurisdiction in the
federal courts over a matter.
a. Reasoning: Concurrent jurisdiction offers the benefit of convenience when it is
easier for litigants to appear in a state than in a federal court and a relatively free
choice of forum. Finally, any erroneous decisions could be appealed through the
state court decision and then ultimately resolved by the U.S. Supreme Court.
D. Exclusive State Court Jurisdiction: State jurisdiction is exclusive in cases not within
the jurisdictional headings of Article III, as it is in cases in which Congress has not seen fit to
confer federal jurisdiction.
1. But, No Concurrent Jurisdiction for Indian Tribal Courts: Tribal Courts are not afforded
the same presumption of concurrent jurisdiction as state courts.
i) Nevada v. Hicks (2001): Tribal courts, unlike state courts, are not courts of general jurisdiction;
rather a tribes inherent adjudicative jurisdiction over nonmembers is at most only as broad as its
legislative jurisdiction.

34

Downloaded From OutlineDepot.com

Kannar Federal Courts, Spring 2011

E.

Congressional Exclusion of State Court Jurisdiction Allowed: Congress is


free to exclude certain cases from state courts and vest exclusive jurisdiction in Article III courts.
This is usually done when legal issues are technically difficult or when Congress distrusts state
judges and wants to avoid their interference or bias.
1. Tafflin v. Levitt (1990) (no exclusion of RICO claims from state courts; establishes test):
Concurrent state court jurisdiction over civil RICO claims would be denied only by an explicit
statutory directive, by unmistakable implication from legislative history, or by clear incompatibility
between state-court jurisdiction and federal interests. Here, nothing in the language, structure,
legislative history, or underlying policies of RICO precluded state court jurisdiction.
a. Concur (Scalia): Congress must take away jurisdiction explicitly not merely by implication
or legislation history

F.

Concurrent Jurisdiction Presumed Unless TEST: State courts have concurrent


jurisdiction over federal claims, unless it is denied
1. By an EXPLICIT STATUTORY DIRECTIVE,
2. By UNMISTAKABLE IMPLICATION from LEGISLATIVE HISTORY,
i) Congressional Silence is Evidence of Concurrent Jurisdiction: Omission of any
express provision making federal jurisdiction exclusive is strong and arguably sufficient
evidence that Congress had no such intent.
a. Yellow Freight System v. Donnelly (1990) (omission of jurisdiction-stripping language is
evidence of concurrent jurisdiction for state courts): The Court concluded that Title VII
contains no language that expressly confines jurisdiction to the federal courts or ousts state
courts of presumptive jurisdiction. The omission of any such provision is strong, and
arguably sufficient, evidence that Congress had no such intent.

3. By clear INCOMPATIBILITY between state-court jurisdiction and FEDERAL


INTERESTS
i) Incompatible if either field preemption or undermine congressional intent in passing
statute]
G. Obligation of State Courts to Enforce Federal Law: Congress has the power not
merely to authorize but to require state courts to hear cases based on federal causes of action.
(Based on Madisonian compromised which anticipated concurrent jurisdiction) Also, State courts
may not discriminate against rights arising under federal laws.
1. State Courts May Decline to Hear Federal Law Claims IF they have
a VALID excuse: Refusal to hear a federal case may not be based on a hostility to
federal law.
i) TEST: A state court may not deny a federal right when the parties and controversy are
properly before it in the absence of VALID EXCUSE. A valid excuse does not include
a states DISAGREEMENT with the content of a federal law or REFUSAL to
recognize the superior authority of its source. Rather, it must be a NEUTRAL state rule
regarding the ADMINISTRATION of the COURTS.
a. Testa v. Katt (1947) (state courts must not refuse to enforce federal rights where similar
state claims could be heard):

b. Successful Refusals to Hear Federal Cases: It is important to note that their refusal
was based on grounds equally applicable to state and federal cases.
i. Sovereign Immunity: If a sovereign immunity defense would preclude a case
from being brought in federal court, a state court may dismiss the case under
similar grounds in its own courts. (Alden v. Maine)
1. Limitation No Sovereign Immunity Defense if Not Available in
Federal Court: A state court cannot allow a sovereign immunity defense
in state court if that defense would not be available in federal court.

35

Downloaded From OutlineDepot.com

Kannar Federal Courts, Spring 2011

i.

Howlett v. Rose (1990) (state court could not apply state sovereign
immunity defense to defeat a federal claim): Illegal Search claim against
school brought in state court under 1983. State court said suit was barred
by state law sovereign immunity. SCOTUS reversed, because state law
sovereign immunity could not be used by a state court to deny recovery
under 1983. A state court must not deny a federal right, when the parties
and controversy are properly before it, in absence of a valid excuse.

ii. State Court of Limited Jurisdiction: A municipal court of limited jurisdiction

can decline jurisdiction over a claim under the Federal Employers Liability Act
when under state law it is without jurisdiction over this kind of law suit (see
Herb v. Pitcairn (1945)).
iii. Forum Non Conveniens: A state court may dismiss a suit brought under the
Federal Employer Liability Act because of the doctrine of forum non conveniens,
provided that the doctrine is applied as a general local practice to all causes of
action begun in its courts (see Southern Railway v. Mayfield (1950)).
H. Procedure: State Courts May have to Follow Federal Procedures : State
courts generally need not follow federal procedures when hearing federal law claims. However
state courts must do so if
i) Congress SPECIFIES the procedure for a particular matter OR
ii) The application of state procedures would be OUTCOME DETERMINATIVE or
SIGNIFICANTLY BURDEN the exercise of federal rights.
I. Forbidden State Court Proceedings Against Federal Officials: Even though
state courts are obliged to follow federal law, they have limited power to enter orders directly
against federal officers. The underlying reasoning for these limitations is that when the state and
federal governments come into conflict, the Supremacy Clause says that the federal government
reigns supreme.
1. No Habeas Corpus: State courts do not have the power to grant writes of habeas corpus
against federal officers alleged to be holding prisoners in violation of federal law.
i) Tarbles Case (1872) (state courts cannot issue writs of habeas corpus for people held by the
federal government): Tarble enlisted in the Army during Civil War, although he was allegedly a
minor and had failed to obtain his fathers consent to enlist. A state court has no jurisdiction to
issue a writ of habeas corpus releasing a person held by the United States or one of its officers.
a. State and local government conflict in some spheres, but the Constitution provides that when
jurisdictional confrontations occur, it is the federal government that reigns supreme. The
federal government has plenary authority to maintain and regulate the military, and if state
judicial officers had the power to secure the release of soldiers, the effectiveness of the Army
would be drastically undermined. Furthermore, the federal government equally protects
individual rights as states do.

2. No Injunction Against Federal Judicial Proceedings: State courts cannot enjoin federal
court proceedings, even to prevent relitigation of matters already fully decided by the state
court. (see Donovan v. City of Dallas (1964)).
i) Except: In rem: in cases where jurisdiction is based on possession of a res
3. No Writs of Mandamus: State courts cannot grant mandamus against federal officers
compelling performance (McClung v. Silliman (1821)).
4. Damages allowed: against fed officers, but removal and immunity will usually prevent it.

SUPREME COURT REVIEW OF STATE COURT


DECISIONS

36

Downloaded From OutlineDepot.com

Kannar Federal Courts, Spring 2011

A.

Authority for Supreme Court Review of State Court Decisions: Article III
provides that the Supreme Court shall have appellate jurisdiction, both as to law and fact, with
such exceptions and under such regulations as Congress shall make. Section 25 of the Judiciary
Act of 1789 then explicitly provided for Supreme Court review of state court judgments where
the state court had decided against a claimed federal right.
1. Mandatory v. Certiorari: Before 1988, the Supreme Courts appellate jurisdiction over state
court decisions was divided between mandatory appeals and discretionary writs of certiorari.
i) However, 28 U.S.C. 1257 eliminated appeals as of right and made all state court
judgments reviewable only by writ of certiorari.
2. Two Key Cases Establishing Boundaries of Supreme Court Review: Martin affirmed the
Courts power to review federal issues decided in state court, while Murdock established
limits on the Courts power to review non-federal issues decided in state court.
i) Congress has the power to authorize the Supreme Court to exercise appellate jurisdiction over
questions of federal law decided in state courts: (Martin v. Hunters Lessee) (1816)

a. Textual Reasoning: The language of Article III extends judicial power to cases and
not to courts. It is plain that the Framers of the Constitution did contemplate that
cases within the judicial cognizance of the United States not only might but would
arise in the state courts, in the exercising of their original jurisdiction. This view
can be seen by the wording of the Supremacy Clause which makes the Constitution,
treaties, and laws of the United States applicable in every state.
ii) Supreme Courts power to review state courts is limited to decisions about to federal questions,
A states highest court is the authoritative interpreter of state law. (Murdock v. City of
Memphis) (1875): The Supreme Court held that the issue presented did not involve federal law,
but instead was a state law matter concerning whether Murdock retained a reversionary interest
based on the original conveyance instrument. The Court held that it lacked any authority to
review state court rulings as to state law, based on Congresss unmistakable intent in revising the
Judiciary Act of 1789 and the assumption that its jurisdiction was limited to the correction of
errors relating solely to federal law and its role of protecting the Constitution and federal laws did
not extend to reviewing other questions not of federal character.

a. Congressional Intent and Judiciary Act of 1789 vs. 1867: Judiciary Act of 1789
explicitly prevented the Supreme Court review of sate court decision on state law
questions. Although the revision of the Judiciary Act of 1867 omitted this provision,
the Court concluded that Congresss unmistakable intent was to prevent Supreme
Court review of state court interpretations of state law, stating that the Congress
would have used plain, unmistakable language if it meant to grant such broad
jurisdiction to the court.
3. After Review, Remanded for State Adjudication: When reversing a state court judgment,
the Court will then remand the case for proceedings not inconsistent with the Courts
opinion. Thus, the state court is free to resolve any undecided questions or even to alter its
determination of underlying state law. The reversal may not, therefore, be decisive of the
final judgment.
4. Remember that SCOTUS cant review state law in an appeal from state court. It can
review state law in an appeal from lower fed courts (where there was supplemental jx over
state claims)
B.

Independent and Adequate State Grounds Doctrine: If


states highest courts decision is supported by a state law rationale that is independent of
federal law and adequate to sustain the result, SCOTUS wont hear the appeal b/c reversal of the
state courts federal law ruling will not change the outcome of the case.
1. Independent: To be independent, a decision must be based on state law that is independent
of whatever federal law might require. If state law is interpreted to make it consistent with

37

Downloaded From OutlineDepot.com

Kannar Federal Courts, Spring 2011

2.

3.

4.

5.

the supposed command of federal law, or if state law incorporations federal law, the state law
is not independent.
Adequate: An adequate state law ground exists where the state law basis for the decision is
sufficient by itself to support the judgment, regardless of whether the Supreme Court would
affirm or reverse the federal issue.
a. Rationales: No advisory opinions. Avoid unnecessary constitutional rulings,
promotes harmony between the federal and state systems by minimizing review, and
conserves judicial resources to cases most deserving of federal attention. In addition
to creating consistency in the doctrine, the Court wanted to avoid examining state
laws with which it was unfamiliar and from making potentially advisory opinions.
b. Criticisms: The doctrine permits inconsistent and incorrect interpretations of federal
law unreviewed. A state court decision wrongly interpreting federal law will remain
on the books, potentially influencing other courts around the country, if there is an
independent and adequate state ground. Also, it invites state courts to try to
immunize their decision from Supreme Court review by manufacturing a state basis
for the decision.
Ambiguous or Unclear Reliance on Federal Law: The Supreme Court will PRESUME
that there is NOT an independent and adequate state law basis for a decision UNLESS the
states highest court provides a CLEAR STATEMENT that its decision was grounded on
state law.
i) Presumption of Federal Law Reliance: When a state court decision appears to rest
primarily on federal law, or to be interwoven with the federal law, and when the adequacy
and independence of any possible state law ground is not clear Court assumes the state
court decided the case because it believed that federal law required it to do so
Procedural Bars As an Independent and Adequate State Ground: Failure to comply with
valid state or federal procedural requirement for raising an issue will prevent a litigant from
presenting a federal issue to the Supreme Court on appeal. However, the Supreme Court will
scrutinize the asserted state procedural bar to ensure that it has been appropriately invoked.
i) Test: when procedural ground, ask whether it is adequate. Inadequate if:
a. State law violates fed due process (ex: cant ask D to object to composition of jury
prior to being indicted, where D isnt even assigned atty until indictment. Reese v.
GA)
b. State law ground is novel or inconsistently applied (some overlap with d/p concept)
c. State rule discriminates against federally-protected rights
d. State rule unduly burdens the federal right (state court cant have lengthy rules on the
type of paper to use or call your brief too long if its only 2 pages, etc. See Staub v.
City of Baxley)
e. Stupid judge tricks
Unconstitutional State Laws are Inadequate: State law obviously is not adequate to
support the result when there is a clam that the state law itself violates the United States
Constitution. An unconstitutional state law cannot support the state courts holding.
i) Staub v. City of Baxley (1958) (unconstitutional state law is not an adequate state ground to
deny appeal): A city law made it an offense to solicit membership in any organization without a
permit. The Court held that the state law was not adequate to support he judgment because it was
unconstitutional.

6. Due Process Violations Create Independent Federal Issue for Review: The validity of the
state procedural rule under the Due Process Clause raises an independent federal question that
the Court has jurisdiction to review apart from any other federal issues in the case.
7. Federal Claims MUST be Raised in State Court: Federal constitutional issues must be
raised and decided in the state court before the federal courts may rule on the issue. This is
based both on statutory 28 U.S.C. 1257 grounds as well as prudential reasoning.

38

Downloaded From OutlineDepot.com

Kannar Federal Courts, Spring 2011

i)

Cardinale v. Louisiana (1969): Fed question had never been raised, preserved, or passed upon in
state courts. The record is likely to be inadequate and the state should be given the first
opportunity to consider the applicability of state statutes to determine whether they can be
interpreted in order to save constitutionality or whether they establish an adequate and
independent state ground for the decision thereby blocking appellate review. Furthermore, the
Court highlighted the fact that there is an alternative remedy habeas if there is no state
procedure available to raise the issue.

8. New Arguments Allowed If a federal claim was properly raised in state court, a party can
raise before any argument in support of that claim before SCOTUS, even if the argument was
not raised in state court (see Yee v. City of Escondido (1992)).
i) Must be Very Specific in Invocation: The Court held that a litigant who had complained
in a state court custody suit about a failure to give full faith and credit to a prior
judgment, but who had not mentioned the Full Faith and Credit Clause in particular, had
presented only a state law issue under, and thus could not raise the federal constitutional
issue in Supreme Court (see Webb v. Webb (1981)).
ii) But, Wood v. Georgia: Employees lawyer was conflicted (employee suing employer was
represented by lawyer paid for by employer). Didnt raise due process objection below,
but SCOTUS still reviewed b/c its not that the lawyer just screwed up below, its the
lawyer himself that the party is complaining about (the lawyer wasnt going to attack
himself below). This is an extremely narrow situation.
C.

Final Judgments That Arent Final and the Highest


State Court
1. The Supreme Court can only review state court decisions that are final judgments or
decrees
i) A final judgment terminates the litigation btw the parties on the merits of the case and
nothing remains to be done but require the lower court to enter the judgment. (dismissal
= final)
2. Cox Relaxed the final judgment rule: Cox Broadcasting v. Cohn: At least 4 categories of cases in
which Court has treated decision on fed issue as a original judgment for s. 1257 purposes w/o
awaiting completion of additional proceedings anticipated in lower state courts.
i) Practical Finality: states highest court may remand to lower court, but it is clear that the
federal issue is conclusive or outcome of further proceedings is preordained.
ii) Federal Issue will survive: Federal issue, finally decided by state highest court, will survive
and will require decision regardless of the outcome of future state court proceedings.
iii) Federal Issue will not survive: Federal claim has been finally decided, with further
proceedings on the merits in the state courts to come, but in which later review of the federal
issue cannot be had, whatever the ultimate outcome of the case (like where party seeking
interim review ultimately prevails, fed issue mooted; or if he were to lose, state law would
prohibit him from again presenting his federal claims for review.
iv) Serious Erosion of federal policy: federal issue finally decided in state courts w/ further
proceedings pending in which party seeking review might prevail on merits on nonfederal
grounds, thus rendering unnecessary review of the fed issue by SCOTUS, and where reversal
of state court on fed issue would be preclusive of any further litigation on relevant cause of
action rather than merely controlling nature and character of, or determining admissibility of
evidence in, state proceedings still to come. If refusal to immediately review state ct decision
might seriously erode fed policy, Court may take Jx.
a. Ex: Could win on 1st Amendment, but might win on something else after losing valid 1st
Amendment claim. Court may take case to make sure the 1st Amendment is protected.

39

Downloaded From OutlineDepot.com

Kannar Federal Courts, Spring 2011

FEDERAL QUESTION JURISDICTION OF THE


DISTRICT COURTS
A.

Federal Question Jurisdiction Generally: Federal question jurisdiction is authorized

by Article III and is specifically conferred on the district courts by statute in 1331. The scope
of the constitutionally authorized federal question jurisdiction is broader than that conferred by
statute. There is a fed. question jurisdiction statute thats also an Art. III provision. Arisingunder comes to mean 2 things: (1) constitutional language and (2) statutory language.
Until 1980, there had been 2 separate jurisdiction provisions pertaining to 1331 (amt in
controversy did not apply to civil rights cases). Now no amt is necessary so no exception & you
dont have to figure out civil rights matters.
1. Article III: Article III of the Constitution states that the judicial power of the United States
shall extend to all Cases, in Law and Equity, arising under this Constitution, the Laws of the
United States, and Treaties made, or which shall be made, under their Authority.
2. Statute: As now codified in 18 U.S.C. 1331, (first passed in 1875, removed amt in
controversy req in 1981) the law provides that [t]he district courts hall have original
jurisdiction of all civil actions arising under the Constitution, laws, or treaties of the United
States.
3. We dont trust state judges: b/c theyre elected, might discriminate against federal law, we
trust Art. III judges, we want a uniform system of federal law.
B. Arising Under Federal Law for Article III Jurisdiction (Broad): Under
the Courts broad definition, most matters will fit within the within the scope of Article IIIs
authorization of jurisdiction.
1. Federal Law is an Ingredient of the Original Cause: Under the Constitution (Article
III), a case arises under federal law whenever federal law forms an ingredient of the original
cause even though other questions of fact or law may be involved in it.
i) Osborn v. Bank of the United States (1824) (federal governments creation of national bank
created ingredient of federal law to grant federal jurisdiction): Ohio Unconstitutionally levied
taxes on each branch of the United States Bank in the state. Because the Bank of the United States
was created by federal law, any legal action brought by in arose under federal law. Notion that the
judicial power must be co-extensive with the legislative power.

Facts: OH w/holding funds under state law. Bank sues under general
jurisdiction. Osborns objection is a pure state law claim being adjudicated, it
should be adjudicated in state ct. Bank says that the statute says it can be heard
in a fed. forum.
Marshall looks at the statutepure jurisdiction grant. Now the question is
whether this case ARISES UNDER the law of the US? (750)
(751)if you understood arising under in every aspect, there wont be anything
to arise there. Talks about S.C.s jurisdiction under Art. III (original jurisdiction
could exist in fed. cts/S.C. where it is not precluded). Congress has the power to
grant Cir. Cts. original jurisdiction.
The real issue (751)can the ct address this specific issue (look at the charter of
Bank of US).
If you have cases when theres a mixture, it is precluded unless just fed., can ct
only hear parts that are just fed?NO, state issues dont disable the fed.
judiciary. Ct can take ANY CASE so long as it is deemed to be arising under the
Constitution.
So long as any case where fed. law is an ingredient, Congress can act w/o
exceeding power to grant original jurisdiction.

40

Downloaded From OutlineDepot.com

Kannar Federal Courts, Spring 2011

Marshallscope of arising under power under Art. IIIif fed. judicial power
(752) is an ingredient of original clause, its power of Congress to give Cir. Cts
jurisdiction of that cause.
In cases involving a bank, there is always the potential for fed. issues.
Justice Johnsons dissentgiving jurisdiction in a case like this will create a
landslide effect, wont be able to keep anything out. It becomes a gigantic flood
gates issue.

ii) 2 Qs to be answered: Does the statute grant federal court jurisdiction, and is the statute
constitutional?

2. Expansive Jurisdictional Grant: The Constitution permits Congress to create federal court
jurisdiction whenever federal law is a potential ingredient of a case. Not only does this
jurisdiction extend beyond situations where there are federal causes of action, but also it
includes instances where a case might turn on/be a dispute about a question of federal law,
no matter how unlikely it is that federal law will be a basis for the decision.
i) Protective Jurisdiction: Scholars argue that the decision in Osborn can be best
understood as authorizing Congress to create protective jurisdiction that is, Congress
may authorize federal court jurisdiction where it believes that that federal court
availability is necessary to protect important federal interests. By this view, in Osborn,
Congress created federal court jurisdiction, even as to state law claims litigated by the
Bank of the United States, to protect the bank from potential state court hostility.
a. Textile Workers Union v. Lincoln Mills (1957) (protective jurisdiction creating federal
question jurisdiction debated): A labor union brought suit under 301(a) of the Taft-Hartley
Act, which confers jurisdiction on the federal courts over actions for violation of labormanagement contracts in industries affecting commerce. Held: Majority held federal court
jurisdiction was appropriate because Congress intended for the federal courts to create a
federal common law of labor-management contracts.
i. Congress sets up fed. collective bargaining agreements. S.C. says the right way to
understand Congress is for someone to set up a fed. law to look at how to read K
law/interpretation.
ii. Concur: Justices Harlan and Burton concurred in the result; they disagreed that federal
law was to be applied, but argued that jurisdiction was appropriate based on the theory of
protective jurisdiction. They contended that state law was to be applied in contract cases
brought to federal court under the THA. However, they argued that courts should take
jurisdiction out of a desire to protect the federal interest in labor cases when Congress has
authority to make a rule to govern the disposition of the controversy. Court has never
adopted protective jurisdiction. (just a theory)
iii. Dissent: Justice Frankfurter dissented and strongly argued against the concept of
protective jurisdiction. He concluded that it could not be justified under any allowable
view of the scope of Article III and that the theory must have as its sole justification a
belief in the inadequacy of state tribunals in determining state law. However, the
Constitution reflects such a belief in the specific situation within which the diversity
clause was confined, thus the problem was already fixed. Congress doesnt enact fed.
substantive law. If premise is case doesnt arise under fed/ law is it consistent
w/Congress Art. I & III powers to establish cts? Frankfurters position is against the
notion of protective jurisdiction (if Congress could have power to proscribe fed. rules
(substantive law bit has elected not to) but w/o doing so enacts a jurisdictional statute).
Something deeply disturbing about this federally. You can strip a state law of collective
bargaining power. Wont do this unless you dont trust states. He thinks there should be
substantive law & protective jurisdiction doesnt trust the states. Hes in profound
disagreement w/everyone. Bankruptcyauthority is fed. & everything happens in 1
circumstance (includes state claims).
iv. Pg 762even if there were a situation in which to change forum to fed. would

still need a palpable substantive question. Note 2 on 763complication b/w

41

Downloaded From OutlineDepot.com

Kannar Federal Courts, Spring 2011

b.

c.
d.
e.
f.
C.

original & appellate jurisdiction . If case is decided in state Ct w/o a question of


fed law ever decided, can a fed. ct. hear the case?
Verlinden v. Central Bank (test case)is it rightfully decided?FSIA confers fed.
forum for any issue involving a foreign country. State law case but can be a fed. case
b/c whether theyre immune or not is always an ingredient. Theres a fed.
substantive law here (sovereign immunity is a major interest that other countries have
to know).
Footnote 8 (770)detailed & obscure. Case if filed in state ct. Question of
certification of AG question of scope of employment. Ct says even if certification
were set aside, the fed. ct would still have subject matter jurisdiction.
Pg. 772Diplomatic Relations ActDiplomats were immune from suits, which
make it fed. forum, parallel to Verlindendiplomats need to know what the rules are
even though auto cases are state law.
Clean Air Actfed. statute allows standards to be set by states incorporated into fed.
law. Fed. law delegated to states.
Air Transportation Safety after 9/11gave jurisdiction of claims to fed. districts in
S.D.N.Y. Substantive lawliability cap.

Arising Under Federal Law for 28 U.S.C. 1331 Jurisdiction


(Limited) The Court has held that 1331 is much narrower in its jurisdictional grant than the
corresponding language in Article III.
1. Test: A case arises under federal law if it is APPARENT from the FACE of the plaintiffs
COMPLAINT if either Cause of Action created by Fed. Law or Fed Law creates cause of
action essential to state law claim (broader than old test that only allowed federal causes of
action):
i) The plaintiffs CAUSE OF ACTION was CREATED by FEDERAL LAW;
a. Cause of Action Created by Federal Law: A case arises under federal law if P states
a claim under a federal law that provides a legal entitlement to a remedy. This is so
even when the only dispute between the parties is about the facts, and regardless
whether the claim is explicitly created, is implied by statute or the Constitution, or is
federal common law.
i.

American Well Works Co. v. Layne & Bowler Co. (1916) (cause of action arising
from federal law grants federal question jurisdiction): A suit for damages caused to
ones business by a threat to sue for patent infringement is not a suit under the patent
laws, and therefore may not be maintained in federal court.

ii. 2 companies manufacture pumps. 1 said the other made bad pumps & #2 sued.

claimed that libel involved some question of who holds the patent (fed.
question). Holmes said you have to look at the central claim of the entire suit (in
this case its libel which is a state issue, the patent issue is only a small issue of
the suit).
He gives further defn. of arising underwhat do you look for in the s
side of the picture? Look for where injury came from. Here, it is libel
(& the cause of action is state law).
Holmes rule: look only at s side of the complaint then look at the law
that creates the cause of action.
iii. Must be non-frivolous claim (Amer. Well Works Co. v. Layne & Bowler Co, 1916)
iv. Whether a complaint states a COA on which relief could be granted must be
decided after court has assumed jx. If no, dismissal on merits, not jxal. (Bell v.
Hood, 1946).
1. But when fed claim is clearly immaterial and solely for purpose of
obtaining jx, can dismiss on jurisdictional grounds.

42

Downloaded From OutlineDepot.com

Kannar Federal Courts, Spring 2011

Even if federal claim is not meritorious but also not insubstantial (under Bell v.
Hood), it could support supp jx over the state law claim even if dismissed on the
merits under Rule 12(b)(6). Must pass the raised eyebrow test to be not
frivolous, can lose on claim and still get jx. May want to raise that claim so that
jx can be maintained throughout appeals. Also, if there are supplemental state
law claims, the fed. ct could keep the state law claims
In Bell v. Hood, the Ct was generating law for cases involving causes of
action generating from the Constitution. Rehnquist read the case as a
frivolous claim & falls outside & requires a 3 tiered analysis.
ii) OR If a federal law creates a cause of action that is an ESSENTIAL COMPONENT of
the plaintiffs STATE LAW cause of action
a. Federal Questions as Essential Components of State Causes of Action: Even if P
does not allege a cause of action based on federal law, there is a federal question if it
is clear from the face of the plaintiffs complaint that a federal law creates a cause of
action that is an essential component of plaintiffs state claim.
v.

i.

Smith v. Kansas City Title & Trust (1921) (general rule for federal jurisdiction of
state law claims): Where it appears from Ps complaint that the right to relief depends
upon the construction or application of the Constitution or Federal Law, and Fed. claim is
not merely colorable, and rests upon a reasonable foundation, the District Court has
jurisdiction under the provision.
Think of paradigm situation. It was a shareholders suit. The basis of the
complaint was is raising a constitutional argument but its a state cause of
action. The Bank is investing in invalid bondsstate law cause of action.
The general disposition turns on fed. question of whether this law is valid or not.
The case remains in fed. ct b/c if cause arises under state law but if the right to
relief is determinative on Fed. Law, stay on the fed. side.
In what kind of procedural types of cases does this arise: (1) sues in fed. ct.
for a state claim and (2) wants to be in state ct & wants to remove to fed.
ct (could s case been brought in fed. ct?If yes, then it is removal).

b. Current Balancing Test: The Grable Test: Court set forth its most recent test for
federal question jurisdiction in suits based on state causes of action. Court will ask
three questions
i. Is there an ESSENTIAL ELEMENT of the state law claim that involves a
question of federal law? If yes
1. Grable brought a state law claim that involved, as an essential element, a
question of federal law (asserted violation of a federal scheme), to which
there is no private right of action. Under the reasoning of Merrell,
therefore, because there was no private right of action then there should
be no federal jurisdiction here either.
ii. Is it a SUBSTANTIAL and DISPUTED QUESTION OF FEDERAL LAW? If
yes
1. In Grable, whether service by mail had been proper under federal law or
not would determine the outcome of the case.
iii. Would recognizing federal question jurisdiction over this claim DISTURB the
CONTEMPLATED BALANCE OF WORKLOAD between the federal and
state courts?
1. However, the Court viewed the question of federal law (the legitimacy of
the IRS sales) was an important one because of the vast magnitude of
property sold from IRS seizures.
i. But, having federal jurisdiction here would not upset the balance
between the federal and state courts. Unlike in Merrell Dow where

43

Downloaded From OutlineDepot.com

Kannar Federal Courts, Spring 2011

the case would implicate thousands of state law tort cases flooding
the federal courts, most quiet title actions dont involve this type of
disputed federal law.
2.

Grable & Sons Metal Products v. Darue Engineering (2005) (established


balancing test): The IRS seized property owned by Grable and gave Grable
notice by certified mail (instead of personal service as required by fed law)
before selling the property to Darue. The issue was whether a case involving the
interpretation of federal tax law belongs in federal court and not the state court
where it was filed. Held: Federal jurisdiction was upheld. The Court held that
the case involved a federal question and could thus be removed to federal court.
Federal-questions jurisdiction, the Court reasoned, lay over some state-law
claims that implicated significant federal issues. In this case, the national interest
in providing a federal forum for federal tax litigation warranted moving the case
to federal court
Grable owned back taxed & IRS sent letter to seize the property & did.
Grable claims that Darues title is a bad b/c seizure was illegal (defect
in how he was servednot personal service).
Cause of action: quiet title claim, which is a state law claim.
Souterclassic ex. is Smith v. Kansas City. Case warrants fed.
jurisdiction b/c whether notice was given w/in the meaning of fed.
statute (its an essential element of quiet title claim, & meaning of fed.
statute is actually in dispute).
The only real issue in this case is a fed. issue. It is also a purely legal
issue, fed. gov has a fed. issue. Its an issue that gets resolved & will
know whether mail service is adequate & then state law cases will go
forward. In (B), he explains Merrell Dowit made perfect sense, must
be read as a whole & is consistent w/what he just did. Merrell Dow
didnt adopt a bright-line test. This case is completely different from
Merrell Dow.
Thomas concurs: jurisdictional rules should be clear. If American Well
Words rule works in 90% of cases, then it should be used.

iv. Empire Health v. McVeigh (798)


The case tells what Grable said. It involved a fed. employee, healthcare for the
fed. employeegov. had an interest. Insurance Co. trying to get $ from estate
of enrollee for tort claim. Estate had a tort action against someone & insurance
Co. was trying to get some of that settlement. Insurance Co wants it in fed. ct
b/c everyone is fed. They were arguing that Grable allows the case in fed. ct.
Ct categorized Grable, Grable was a pure question of law, and this case involves
questions of fact & is distinguishable.
v. Net Effect Gives Federal Courts Power to Decide Their Own Jurisdiction:

By giving the federal courts this power under Grable, it gives the federal courts
the power to determine whether they have the power to hear a particular case (or
string of cases) or not. Is this a good thing? Should a federal court have the
discretion to shape the limits of its jurisdiction? Is it even worth the trouble to
minimally expand the jurisdiction?
c. Old Test Federal Statute Must Itself Create a Cause of Action: It is not enough
for a federal law to be an essential component of a state law cause of action; federal
question jurisdiction exists only if the federal law itself creates a cause of action,
albeit not one relied by the plaintiff.
i.

Merrell Dow Pharmaceuticals v. Thompson (1986) (no federal jurisdiction unless


independent federal cause of action) : Birth defects allegedly caused by its drug.
Violated federal law by failing to include proper warnings on the labels. This violation of
federal law, if proved, would have constituted a presumption of negligence under state

44

Downloaded From OutlineDepot.com

Kannar Federal Courts, Spring 2011

law. Held: No federal question jurisdiction. B/c Congress has determined that there
should be no independent private federal cause of action for the violation, does not state a
claim arising under federal law.
Tort case. Under OH law, a violation under a fed. statute created negligence.
State cause of action (negligence), but compliance is under fed. law. Merrell
tries to remove it to fed. ct.
Justice Stevens: wants to analyze under congressional intent. Upon examining
the legislative history, Stevens concludes it doesnt matter if theres a conflict.
Brennans dissent: if Holmes isnt stating the rule then Ct wouldnt go through
analysis, what other factors are necessary to examine. Ct is setting up ad hoc
jurisdictional test.
Majority saysfed. interest isnt that great, you dont know how many cases
you would be opening the door to. It would federalize all tort claims & ct will
not do that.
ii. Shoshone (weird exception, rarely going to win on it): Federal law grants mining patents,
to be determined based state law, court says no fed jx cuz its just a factual Q. Maybe just
wanted to keep all these cases out of fed court.

B.

Well-Pleaded Complaint Rule: It must be clear from the face of the plaintiffs
complaint that there is a federal question. Federal court jurisdiction cannot be based on a federal
law defense or on the plaintiffs anticipation of a federal law defense.
1. Louisville & Nashville R.R. Co. v. Mottley (1908) (well pleaded complaint rule): Mottleys were
injured in trains crash, agreed to release the railroad from liability in exchange for free passes.
Congress then precluded common carriers from granting free transportation. They sued railroad in
federal court alleging that their property was deprived without due process of law. A federal court may
not exercise jurisdiction over a case merely because an anticipated defense or the response thereto will
involve a federal question. The plaintiffs complaint presented a state law claim for breach of contract.
The federal issue arose only from the plaintiffs anticipation of a defense based on the federal

statute.
Settlement agreement b/w RR & couple for free passes for life. Law passed & no
more free passes. Couple sues RR for specific performance.
S.C. says you need original jurisdiction under 1331 to be part of the well-pleaded
complaint rule (for ). Need to file in state ct. If fed. piece is there for s sign
under the statute, then fed. jurisdiction (abides by well-pleaded complaint rule).
Constrains day-to-day fed. jurisdiction.
2. If you are a or almostif you know you cant get a fed. forum, dont be the , be the (its your
complaint) & you can get control of it. Have to establish justiciablity. Note 4 (780)even
compulsory fed. counterclaim doesnt establish arising under jurisidciton.

Vaden v. Discover Bank (5-4) pg. 39 Supp.1 party files claim to compel arbitration.
District Ct lacked jurisdiction. Case turns on statutory interpretation.

3. Federal questions cannot be frivolous. To be a fed Q, just has to be non-frivolous, doesnt


necessarily have to be a winner.

4. Cases going from state court directly to SCOTUS: dont have to pass test b/c the inherently
already did b/c the federal Q must have been decided on the merits in state court in order to
get SCOTUS review.
C.

Also cannot use DJ action to use defense to get Fed Jurisdiction (Skelly
Oil): prevents plaintiffs from gaining federal question jurisdiction despite the well-pleaded
complaint rule by pleading an anticipated federal defense as a declaratory judgment claim
1. Court will look to see what the claim would look like if it was asking for damages or
injunctive relief instead of just a DJ. If the federal Q would only be available as a defense, it
fails the well pleaded complaint rule.

45

Downloaded From OutlineDepot.com

Kannar Federal Courts, Spring 2011

a. Company was trying to build pipelines across state lines. To do this, you need a
certificate & need to contract w/another co. If fed. certificate is not granted, seller
can terminate K. After the party tries to pull out, they try to sue under the
Declaratory Judgment Act. Skelly makes a K w/ Phillips & tried to get out but
Phillips sued in fed. district ct for a declaration that the certificates are valid under the
Act.
b. Ct doesnt make the declaration b/c no jurisdiction over a case like this (analogy to
Mottley)Phillips is making a pre-emptive suit. Its a state law case for anticipation
of a breach. The real complaint is Phillips ought to be suing for specific performance
which is a state claim.
c. You have to be aware of the artful pleading & figure out what the real situation is.
Fed. cts obligation extends to determine whether either party would have a state
claim (making it exceedingly difficult to go forward).
d. If you put Kansas City back into the picture (mixed state & fed. issues) & must
reconstruct to determine if it should have been brought in fed. ct.
e. Lower cts have held that if either party has a fed. action & uses the Declaratory
Judgment Act, cts can see if anyone has a claim under 1331 original jurisdiction.
f. Can you enjoin state laws?The answer is all over the place. What if theres a
statute that doesnt mention a cause of action?Supremacy Clause (can sue to enjoin
the operation of state law by virtue of pre-emption).
2. Franchise Tax Board (cant use claim of fed preemption to remove DJ action from state
court): Dispute brought in state court. Trust doesnt want to do what tax board says, claims
ERISA preempts state law and it doesnt have to pay taxes to board. State sues for state DJ.
Trust tries to remove to fed court saying ERISA DJ Q is a fed question. Two claims by state
tax board in state DJ action:
i) enforcement for damages for failing to comply with tax laws, and
a. no fed q jx (state law claim with federal defense see motley)
ii) state DJ action to say that ERISA doesnt apply.
a. ERISA is one of 3 statutory schemes with complete preemption. But court says Avco
doesnt apply, ERISA preemption doesnt extend this far.
b. Court says state DJ actions treated the same was as fed dj actions. Dont want ppl
making end run around Skelly Oil rule. (itd be like allowing an anticipated defense
in, which is not allowed under well pleaded complaint rule)
c. Issue: whether ct can move forward in making a fed. judgment action from a state
declaratory action?NO
d. If state couldnt have brought it in fed. ct, then no fed. jurisdiction & D cant remove
to fed. ct. ERISAs statutory provisions show Congress wanted to allow some parties
to bring fed. action.
1983 Creates a cause of action. But what is the bearing of injunctions on declaratory judgments if
you cant meet the bar of getting an injunction you still may be able to get a declaratory judgment (if you
could have a fed. injunction, you could elect to have a fed. declaratory judgment.
Federal Question Removal: 1441: where theres a fed. officer involved. What is the question
raised (pg 812(2-4)). Nature of fed. act is so that it displaces/obliterates the state cause of action. It
has preempted substantive matter. Under statutes, there is no state law to apply. Until 2003, its
unclear what fed. statute has to say. Pg. 819another provision 1443Civil Rights Removal.
1960s cases opened the door to widespread renewal (Georgia v. Rachel), the door was closed
quickly( City of Greenwood v. Peacock).

Peacock: seeing floodgates (1443 & Civil rights Act) suggested that it federalized all civil
rights cases. State prosecution must be based upon a state law that on its face violates civil

46

Downloaded From OutlineDepot.com

Kannar Federal Courts, Spring 2011

rights. State law itself would have to be a violation of fed. statute. Other options: (1) get an
injunction, (2) sue for damages, or (3) fed. habeas).

D. Supplemental (Pendent Jurisdiction)


United Mine Workers of America v. Gibbs
o Facts: The employee was awarded compensatory and punitive damages in an action

o
o

against petitioner union for alleged violations of federal law governing unfair labor
practices. The employee brought his action in federal court in connection with a state
court action for unlawful conspiracy and unlawful boycott. The suit stemmed from
the conduct of union members, who, through violent means, forcibly prevented the
opening of a mine operation supervised by the employee.
Ct explains supplemental jurisdictionfacts support both fed. & state claims &
doesnt make sense to break them up.
NEW TEST: if state & fed. claims arise from the same nucleus of fact, if Ps claims
are such that he would have them heard in a fed. ct, then decide them both. Power to
do it doesnt mean its recommended & shouldnt be used in every case. Can go
forward assuming if its fair, convenient, even if the fed. claims drops from the case.
After fed claim has dropped out then must question the convenience, time, economy,
etc. It the fed. claims fall out before trial, it is a major fact to sent to state ct.
Holding: even assuming pendent jurisdiction was proper in the case; reversal was
required due to the employee's failure to meet special proof requirements imposed by
federal law, which required proof that the union ratified the acts of its members. The
Court held that while the union members acted in a way that was reprehensible, there
was no proof that the union approved of the violent methods. The mere fact of
continued picketing at the mine site was not properly relied upon to show
ratification. Actual proof existed that the union put a stop to it as soon as it became
aware.
Pg. 829think constitutional avoidance. What if you have novel fed. issue?

Try to avoid & resolve case in state ct or fed statutory. Siler v. Louisville
& Nashville RRreach out & ignore fed constitutional question
Finley v. US: ct rejected pendent party jurisdiction. Ct articulates some boundaries.
Theres no ancillary jurisdiction over a Ps claim against a non-diverse 3 rd party.
o After Finley, Congress acted. In 1990, Congress enacts 1367: Supplemental
jurisdiction. It replaces Finley. Under 1367, district cts have supplemental
jurisdiction over all claims that are so related to claims in the action w/in
such original jurisdiction that they form part of the same case or controversy
under Art. III.
o May decline to hear cases if they fall under 1 of the 4 subsections (this
codifies Gibbs factors).
Jinks v. Richland Country: questioned whether tolling was valid (is it fed
overreaching). Congress can adjust jurisdiction under Art. III & 1367 is an ex. of that
power. Ct said its ok b/c it is necessary & proper part of Congress power to
establish inferior cts in a fair & efficient manner.
Raygor v. Regents (834): decided the yr before Jinks. 1367(d)s tolling provision
doesnt extend to fed ct suits against non-consenting state.
E. Similar Rules for Removal (D cannot remove if P didnt raise Fed Claim): A
defendant may not remove a case from state to federal court unless it is clear from the face of the
plaintiffs complaint that there is a federal question. Also, if a plaintiff chooses not to present a
federal claim, even though one is potentially available, the defendant may not remove the case
form state to federal court.
1. However, a plaintiff may not defeat removal by omitting to plead necessary federal questions
in a complaint.

47

Downloaded From OutlineDepot.com

Kannar Federal Courts, Spring 2011

F.

Counterclaims: A federal counterclaim, even when compulsory, does not establish arising

under jurisdiction. A contrary rule would (i) permit the defendant to defeat the plaintiffs forum
choice by raising federal counterclaims, (ii) radically expand the class of removable cases, and
(iii) undermine administrative simplicity by making jurisdictional determinations depend on the
content not only of the complaint but also of responsive pleadings (see Homes Group v. Vornado
Air Circulation Sys (2002))
1. .
G. PREEMPTION: Only defense that would allow for removal is when federal law provides
for complete preemption of state law claims and provides an exclusive federal right of action.
Court will be hesitant to find federal preemption of state law without a clear statutory
prescription, a direct conflict between federal and state law, or uniquely federal interests (see
Boyle v. UT)
1. Presumption against finding preemption in area of tradl state concern could interfere with
states expertise and work, federalism concerns. Ex: property, inheritance, family, tort
2. Two Circumstances where courts will find preemption:
i) When Congress intends federal law to occupy the field (field preemption) or
ii) When state law conflicts with a federal statute (would undermine purposes of fed law or
impossible to fully comply with state and fed law, conflict preemption) (Crosby v. Natl
Foreign Trade Council, 2000)
i. may want to lead with conflict preemption because field preemption can be more
narrowly applied
ii. Federal law generally permits parallel or supplemental state law to co-exist (CA
v. ARC America); State law will not be superseded unless Congress made this
purpose very clear
3. 3 cases where SCOTUS has found complete preemption:
i) Avco: bringing state law breach of K claim, but claim was covered by Taft Hartley Act.
They were just disguising a federal claim as a pure state law claim, no artful pleading
allowed.
ii) Met Life: Erisa preemption
iii) Beneficial National Life: court says in cases where complete preemption has been
found, these are situations where federal law not only preempts state law, but also
provides a substitute remedy. Congress probably intended this to be the exclusive
remedy. When thats the case, Fed jx will be found (obvi).
4. Examples of Counterclaims Rules effect:
i) Discover Bank (must look through claim to see if there is fed jurisdiction): Discover
sues cardholder under state law, cardholder counterclaims, Discover sues in fed court
under fed law to compel arbitration of counterclaims. Discovers claim doesnt arise
under federal law, its like a defense to the state law claim. Court looked thru the
arbitration act to see what it arose under.
ii) Examples: A state law claim against B, B counterclaims w/ fed claim: no fed jx
a. B can just bring claim separately. But this raises compulsory counterclaim problems.
If the federal claim was a compulsory counterclaim, failure to assert it in first case
(A v. B) will preempt Bs later federal claim.
b. A state law & fed law claim against B.fed Q jx depending on importance of fed
claim
c. Parties may use declaratory judgments if they think they are going to be sued in state
court, but no artful pleading (Skelly Oil)
i. a plaintiff may not use a declaratory judgment action to gain federal question
jurisdiction over an action that would not otherwise create 1331 jurisdiction
because of the well-pleaded complaint rule. Frankfurter's view in Skelly Oil is

48

Downloaded From OutlineDepot.com

Kannar Federal Courts, Spring 2011

supported by the text of 2201, which allows courts to grant relief to "actual
controversies within (their) jurisdiction." For declaratory judgments, federal
jurisdiction lies if either party could get federal jurisdiction over some underlying
coercive action.
D. Provokes Race to the Courthouse: This test often provokes a race to the courthouse
because if one party files (or fails to file) first, then a federal cause of action could be created or
be precluded. E.g. filing a federal counterclaim of patent infringement does not create federal
jurisdiction, while originally filing for patent infringement would create federal question
jurisdiction.

Suits Challenging Official Action (Ch 9)


Section 1. Suits Challenging Fed Official Action: Congress straightens out suits challenging fed
official action. Notion of sovereign immunity goes back to the king (the kind couldnt violate the
lawsthe cts worked for the king). Comes over to America in some fashion. Notion of petitions of
right. Notion of habeas corpus comes into play. Immunity is not though to extend to every officer
(Marburyagainst an officer for an action). 858in some ways sovereign immunity is abstract. A
gov cant insist on immunity all the time (b/c no one would do business w/you w/o having the right to
sue). Can sue gov under APA, statues, Tucker Act, Contractual Actions, fed. torts claims. What if the
gov sues you & you have an argument that they owe you $ too?Ability to assert counter-claims that
you would end up w/$ in your pocket vs. claims to offset claim (gov doesnt write you a check, you
either pay the difference or its a wash).
Section 2: Suits Challenging State Official Action: if you have 2 sovereigns how does everything
sort out? Fed. & State sovereignty
A.

Sovereign Immunity Generally: Eleventh Amendment: The Judicial power of

the United States shall not be construed to extend to any suit in law or equity, commenced or
prosecuted against one of the United States by Citizens of another State, or by Citizens or
Subjects of any foreign state.
1. Modern Interpretation of Eleventh Amendment: The Eleventh Amendment is a
jurisdictional bar to unconsented suits by all private citizens against a state in either federal or
state court. B/c it is jurisdictional, the defense can be raised at any time (so state could lose
on the merits, then raise SI defense on appeal, so long as they never waived SI)
i) Three exceptions to this general rule:
a. Suits against state officers under the Ex parte Young doctrine;
b. Suits pursuant to federal laws enacted under 5 of the Fourteenth Amendment; and
c. Waiver or consent by a state.
B. Competing Theories of Sovereign Immunity :
1. Majority Restriction of Subject Matter Jurisdiction: A current majority of the Supreme
Court interprets the Eleventh Amendment as a restriction of the subject matter jurisdiction of
the federal courts that bars ALL suits against state governments.
2. Minority Only Limits Diversity Suits: A minority of the Supreme Court interprets the
Eleventh Amendment as restricting only the diversity jurisdiction of the federal courts. These
judges rely on the specific language of the Amendment, against one of the United States by
Citizens of another State, and the purpose of the enactment of the Amendment in the first
place i.e. in response to Chisholm.
i) Chisholm v. Georgia (1793): Involved an attempt by a South Carolina citizen to recover money
owed to him by the state of Georgia. The state did not appear to argue the case, believing that

49

Downloaded From OutlineDepot.com

Kannar Federal Courts, Spring 2011

federal courts had no jurisdiction over it unless it consented to be sued. Held: The Court held that
there was subject matter jurisdiction. Article III authorized the federal courts to hear suits against
a state by citizens of another state. The State-Citizen Diversity Clause conferred jurisdiction, and
the state sovereign immunity was abrogated in suits brought under that clause. This led to the 11th
Amendments adoption.
State claimed sovereign immunity. Ct had jurisdiction to hear the case. Allowed a
citizen of 1 state to sue another state. There was an uproar after the case & Congress
passed the 11th A (invalidated Chisholm), Amendment didnt allow a citizen of 1 state to
sue another state. Jay had a good constitutional claim (his Art. III argument),
ii) Hans v. Louisiana (1890) (bars suit against state by state citizen): Hans, a citizen of Louisiana,
sued LA in federal court b/c legislature amended the state constitution to keep state from making
interest payments on the bonds he had bought. Under the Eleventh Amendment, the judicial
authority of the federal court does not extend to a suit against a state by one of its citizens. States
may only be sued with their consent and LA did not consent. Would seem to violate intent of
amendment otherwise.
not covered by 11th A. Hans was suing state over bonds
Issue: whether a citizen can sue his own state?NO!
The ct also noted that a state cant be sued by a citizen of another state. Although 11th A
doesnt mention a citizen not being permitted to sue their own state, you have to go back
to the time it was enacted. Way to understand 11th A was to correct some ambiguity in
Art. III. Ct said they dont have to explain it b/c its what it is. Ct says that what
Marshall said in Cohens v. VA was dicta.
Diversity theory of 11th ABrennan says that 11th A should be understood as textual.
Read it as having to do w/diversity jurisdiction only (Hans was decided incorrectly).
Note 6 (882)possibility of waiver & partial waiver, constructive waiver. Difficult how
to conceptualize what sovereign immunity is. States can waive it. Is it better to think
about sovereign immunity as jurisdictional or a defense.

C.

Application of the Eleventh Amendment: Whats Barred and Whats


Allowed:
1. Suits Barred: The Eleventh Amendment precludes:
i) suits against a state government by citizens of another state or foreign country (see
language of the Amendment).
ii) suits against an unconsenting state by its own citizens (see Hans);
iii) suits in admiralty by private citizens;
iv) suits by foreign nations or Native American tribes (see Seminole Tribe);
v) suits against the in state court (see Alden);
vi) suits in federal administrative agency proceedings (see Federal Maritime Commission);
vii) or statewide agencies.
2. Suits Allowed: The Eleventh Amendment does not bar:
i) Suits Against State Officers (Ex-Parte Young): b/c its technically against the officer, not
the state
ii) Suits allowed by Congress under Section 5 of 14th Amendment
iii) Suits against municipalities or political subdivisions of a state (e.g. schools, fire/police,
sanitation, etc.).
a. Exception: where there is so much state involvement in the municipalities action
that the relief, in essence, runs against the state (see Pennhurst).
iv) Appellate Jurisdiction Always Allowed: The Eleventh Amendment does not prevent the
United States Supreme Court from hearing claims against the state as part of its appellate
jurisdiction.
a. 11A does not apply to SCs app jx historically, writ of error was not a suit under
11A (Cohens v. VA, 1821; McKesson Corp v. Division of ABT, 1990))

50

Downloaded From OutlineDepot.com

Kannar Federal Courts, Spring 2011

v) Suits by US/State against an unconsenting state


D.

Ex Part Young Suits Against State Officers for Injunctive Relief


1. Injunctions Only: The Eleventh Amendment does not preclude suits against STATE
OFFICERS for INJUNCTIVE RELIEF, even when the remedy will enjoin the
implementation of an official state policy.
i) State officers have no authority to violate the Constitution and the laws of the United
States.
ii) Therefore, their illegal actions are stripped of state authority, and SI is not an issue.
a. Ex Parte Young (1908): On the merits, the Court ruled that the legislation was facially
unconstitutional. As a result, the Court also held that there was no Eleventh Amendment
sovereign immunity defense. If an act that a state attorney general seeks to enforce is a
violation of the federal Constitution, the officer, in proceeding under the act, comes into
conflict with the superior authority of the Constitution and is stripped of his official character
and is subject, in his personal capacity, to the consequences of his individual conduct.
b. Derivative action in fed. cts alleging that state legislation regulating RR rates was
confiscatory & violated 14th A. Prosecution said test the rates to see if they are illegal. Like a
modern test case but no one would want to challenge it b/c penalties are too harsh (it almost
has a chilling effect). If no challenge to it then everyone will comply w/an illegal law.
c. Ct said if it is unconstitutional, it will be deemed as AG Youngs idea. Hes divested of being
on the state to get around 11th A & Hans, but acting under color of the law.14th A.
d. Fed. cts cant interfere in a case where proceedings were already pending. Habeas is brought
forward. An absolute constitutional barrier (there must be some exception to it).
e. Can sue an officer for injunctive relief on grounds that the law violates the constitution.

2. NO Suits Against State Officers for Damages/Monetary Relief: Concern about federal
court relief that had the effect of forcing state governments to pay money damages. Eleventh
Amendment prevents an award of monetary relief even when the individual officer is named
defendant in the lawsuit.
i) Costly Prospective Compliance Permitted: Expensive injunctions are ok. (see Milliken
II, which held that a state paying half the costs of a school integration program was
prospective-compliance rather than retrospective relief).
3. Attorneys Fees Allowed: b/c they are ancillary to the injunctive relief ordered (Hutto v.
Finney).
4. Form of the Action is equitable

Edelman v. Jordan: an equitable action is brought for state to comply w/the law (ex parte
Young) & wanted restitution NOT damages. Ct doesnt believe that monetary damages would
be from Edelman, it would be from the state & thats not allowed. Complying w/injunction
can cost $ but is doesnt go to Ps, it is prospective injunction.
Note 7 (896)state has to pay Ps attorneys fees pursuant to Sec. 1988. Purpose of statute is
to ensure that the law/right will get enforced. Its ancillary to injunctive remedy. Its a basic
civil right & shouldnt have to sue to get it enforcedwhy attorneys fees are allowed.
Note 5 (894)appears that theory of ex Parte Young might be weakened. A very narrow
exception by 5 justices (sovereign control of submerged lands). But in 2002, Verizon case
upheld ex parte Young.

5. Exceptions to Ex parte Young Doctrine: No injunctive relief in 3 cases


i) Ex Parte Young legal fiction applies only to Federal Laws, not State Law: The
Eleventh Amendment bars federal courts from enjoining state officers from violating
state laws. No suits against state officers on supplemental state law claims in federal
court.
a. Pennhurst State School v. Halderman (1984) (no ex parte young suits in state courts): it is
difficult to think of a greater intrusion on state sovereignty than when a federal court instructs

51

Downloaded From OutlineDepot.com

Kannar Federal Courts, Spring 2011

state officials on how to conform their conduct to state law. Such a result conflicts with the
principles of federalism that underlie Eleventh Amendment.
b. Implications note (899)if you have a fed. & state claimwhat do you do?Can bring 2
cases but can get conflicting rulings, if 1 moves faster, it can tangle the case. No one has an
answer to this.
ii) Congressional power to Abrogate State Immunity from Suit
Parden v. Terminal Ry: looks as though constructive waiver will have some substance
to it.
Fitzpatrick v. Bitzer: there can be lines when times when an act of Congress can simply
abrogate states immunity. Congress deemed to have the power to abrogate states
sovereign immunity (under Sec. 5 of 14th A). 11th A did not par an award of retroactive
retirement benefits.
What was the basis on sovereign immunity under Hans? (see footnote 11 on 927). In the
Welsh case in 1987, 4 votes that Hans was wrongly decided, 4 said it was constitutionally
correct & Scalia wasnt ready to make a decision & overrule Hans. By 1989, in PA v.
Union Gas, 5 votes that Hans was incorrect.

iii) No Suits Enforcing Federal Laws with Comprehensive Enforcement: State officers
cannot be sued to enforce federal statutes that contain comprehensive enforcement
mechanisms such as complex remedial schemes because Young relief make Congresss
scheme redundant and possibly subject officers to increased liability for noncompliance
(e.g. contempt orders).
a. Seminole Tribe v. Florida (1996) (statute created comprehensive enforcement relief which
precluded ex parte young remedy): The Indian Gaming Regulatory Act contained a detailed
remedial scheme in that it provided for court ordered negotiations and the submission of
claims to a mediator. Held: Young relief would make the remedial scheme under the Act
superfluous, for it is difficult to see why an Indian tribe would suffer through an intricate
scheme when more immediate relief could be available. Further, the fact that congress chose
to impose upon the State a liability which is significantly more limited than would be the
liability imposed upon the state officer under Ex parte Young strongly indicates that Congress
had no wish to create the latter under the Act.
Indian Gaming Act that compels that state to act in good faith w/the tribes. Gov.
refuses to negotiate in good faith, & says immune from s suit under 11th A. This is an
injunction case. If the state is named (even if injunction) there is another interest
the indignity of subjecting a state to coercive process of judicial tribunals. Union
Gas mistakenly relied on Fitzpatrick.
Where an Art. I complaint would be put to rest.
11th A prevents congressional authorization of suits of private parties against
unconsenting states. What should Indian Tribe do now?use Ex Parte Young b/c
its a pure injunction. In III, the ct holds Ex Parte Young is inapplicable & the suit is
barred by the 11th A. The statute is unconstitutional, but since Congress passed it, it
is valid
Justice Stevens dissent: 14th A is an illegal exception of statutes enacted pursuant to
the Enforcement Clause. Thinks Hans should not be overturned (other way to read
Hans is Contracts Clauseforce states to pay their debts)he disagrees w/this.
Souter, Ginsburg & Breyers dissenthe treats 11th A like legislation as if it has
legislative history. Says Hans was a bad decision & should be overruled (b/c it was a
misreading of the 11th A). Sovereign law immunity was common law immunity &
not subject to statutes. States waived some sovereignty by joining the Union. Pg.
916he accuses the ct of being like Lochner. In V, even w/o Ex Parte Young, he
would follow Union Gas.

iv) Quiet Title to Submerged Lands: State officers cannot be sued to quiet title to
submerged lands. This is because the suit would diminish, even extinguish, the states
control over its lands and waters, and this would be as intrusive as almost any retroactive

52

Downloaded From OutlineDepot.com

Kannar Federal Courts, Spring 2011

levy upon funds of the treasury (see Coeur dAlene Tribe). Itd be like suing the
sovereign.
E.

Abrogation: Suits Against States Pursuant to 5 of the Fourteenth


Amendment: Congress may authorize suits against state governments only when it is acting
pursuant to 5 of the FOURTEENTH AMENDMENT. Congress may not override the
Eleventh Amendment when acting under any other constitutional authority.
1. Reasoning Fourteenth Amendment Changed Federalism Balance: The Fourteenth
Amendment specifically was intended to limit state sovereignty, so legislation under it can
authorize suits directly against the states in federal court.
i) Fitzpatrick v. Bitzer (1976) (14th Amendment limited state sovereignty): Title VII federal court
action alleging that the Connecticuts retirement plan discriminated against male employees. Not
barred by 11th Amendment b/c the 14th Amendment was intended to limit state sovereignty and
therefore congressional legislation under the 14th Amendment can authorize suits directly against
the states in federal court.

2. Test: Intent to Abrogate The Clear Statement Rule: To override state sovereignty
pursuant to 5 of the Fourteenth Amendment, the federal law must explicitly and by clear
language indicate on its face an intent to sweep away the immunity of the States.
i) Abrogation Test: In Seminole Tribe, the Court set forth the following test for when a
federal statute purports to abrogate state sovereign immunity: Ask whether
a. Congress has UNEQUIVOCALLY EXPRESSED its INTENT to abrogate the
immunity?;
i.

Just b/c statute is passed under 14th amendment is not enough.

b. AND
c. Congress has acted PURSUANT to a VALID EXERCISE OF POWER?
i. Adopted Under Other Congressional Powers Do NOT Abrogate Sovereign
Immunity: The Supreme Court has ruled that Congress may abrogate the
Eleventh Amendment only when acting under its 5 powers and not under any
other constitutional authority.
ii. E.g., No Abrogation of Sovereign Immunity via 1983 in Quern v. Jordan (1979),
the Court found that in 1983 there was insufficient indication of an express
congressional desire to make state governments liable under that statute. (can still sue

state officers and municipalities though)


iii. Seminole Tribe v. Florida (1996) (interstate commerce clause did not abrogate
sovereign immunity, overrules Union Gas): Indian Gaming Regulatory Act provided that
an Indian tribe may conduct certain gaming activities as long as they conformed with a
valid compact between the tribe and its resident state. State had to negotiate in good
faith. Tribe could sue state in federal court. Held: No abrogation.

F.

Abrogation Limited Now, Sufficient Record Necessary: City of Borne


Limitations: In City of Borne, the Court sharply limited the scope of Congresss power under 5
of the Fourteenth Amendment.
1. Pursuant to 5, Congress may act only to prevent or remedy rights recognized by the courts
and that Congress may not create new rights or expand the scope of rights. Any law must be
narrowly tailored to solving constitutional violations; it must be congruent and
proportional to preventing and remedy the constitutional violations.
i) Congress Must Create a Sufficient Record: The court ruled that for Congress to invoke
5, it must identify conduct transgressing the Fourteenth Amendments substantive
provisions and must tailor its legislative scheme to remedying or preventing such
conduct. Thus, for 5 legislation to withstand judicial scrutiny, there must be a sufficient
record of constitutional violations.

53

Downloaded From OutlineDepot.com

Kannar Federal Courts, Spring 2011

i.

Florida Prepaid Postsecondary Ed. Expense Bd. v. College Sav. Bank (1999)
(insufficient record of patent infringement by states): A bank sued FL for patent
infringement relying on the Patent Remedy Act, which included any State and
specifically abrogated any Eleventh Amendment or other sovereign immunity defenses in
such actions. Held: There was no evidence that unremedied patent infringement by
States had become a problem of national importance, and thus the 5 remedy was not
properly tailored.
ii. Kimel v. Florida Bd. of Regents (2000) (insufficient record of age discrimination): A
group of librarians and professors at two FL state universities sued the state alleging that
the schools salary decisions disadvantaged older employees in violation of the Age
Discrimination in Employment Act. Held: Under the congruence and proportionality test
from Borne, there was no patterned of age discrimination in the state in public entities to
extend 5 as a prophylactic legislative remedy.
iii. Board of Trustees of the University of AL v. Garrett (2001) (insufficient record for
violations of ADA): Congress abrogation of state immunity from damages actions for
violation of the Americans with Disabilities Act could not be sustained as an exercise of
power under 5. Record was not strong enough to support legislation under 5.

ii) Abrogation Found Where Heightened Scrutiny: In cases where the Court has utilized
heightened scrutiny, e.g. quasi-suspect classes or due process claims, abrogation of state
sovereign immunity has withstood judicial review.
i.

Nevada Dept of Humans Resources v. Hibbs (2003) (gender discrimination): Court


upheld the abrogation of state sovereign immunity in Family and Medical Leave Act of
1993. The Court held that Congress had met its burden under 5 as it had sufficient
evidence of a pattern of constitutional violations on the part of States (gender
discrimination) and the provisions were congruent and proportional to the targeted
violations.
ii. Tennessee v. Lane (2004) (due process right to access to courts): Sovereign Immunity
abrogated when state courthouses didnt give equal to the disabled. Congress had plainly
intended to abrogate immunity in this case, and had the authority to abrogate immunity
and provide for a prophylactic remedy under 5 where there was ample evidence of
pervasive unequal treatment of the disabled where the claims were due-process access to
the courts allegations.

G.

Bankruptcy Exception under Article I: The Court carved out an exception to the
general rule of Seminole Tribe by holding that the Bankruptcy Clause was intended not just as a
grant of legislative authority to Congress, but also to authorize limited subordination of state
sovereign immunity in the bankruptcy arena.
1. Central Va. Community College v. Katz (2006) (limited abrogation of sovereign immunity in
bankruptcy): Katz, the supervisor of a bookstore that declared bankruptcy, filed a suit to collect debts
owed to the bookstore by Central Virginia Community College and several other schools operated by
the state of Virginia. The issue concerned whether the Bankruptcy Clause of the U.S. Constitution
(Article I Section 8), waived the states' sovereign immunity? Held: Yes. The Court ruled that states
cannot invoke sovereign immunity as a defense in bankruptcy proceedings. Rather than ruling on the
power of Congress under the Bankruptcy Clause to waive states' immunity, as the bankruptcy court
did, the Court held that ratification of the Clause itself involved a subordination of state sovereign
immunity. Court relied on the history and intent of Bankruptcy Clause, which indicated that "the power
to enact bankruptcy legislation was understood to carry with it the power to subordinate state
sovereignty, albeit within a limited sphere."
It was a fed. bankruptcy case states can be brought in b/c its inside bankruptcy power
(included the power to preempt what the states were doing). Bankruptcy Clause was intended
to grant legislative authority to Congress, but also to authorize limited subordination of state
sovereign immunity in bankruptcy arena.

H.

Congress May NOT Abrogate State Sovereign Immunity in State


Courts: The powers delegated to Congress under Article I of the United States Constitution do not
include the power to subject unconsenting states to private suits for damages in state courts. The states

54

Downloaded From OutlineDepot.com

Kannar Federal Courts, Spring 2011

sovereign immunity derives not from the Eleventh Amendment, but rather from the structure of the original
Constitution itself. Thus, the fact that the Eleventh Amendment is silent on the matter signifies that none of
the Founders contemplated that the Constitution might strip the States of that immunity.
1. Alden v. Maine (1999) (no abrogation of state sovereign immunity in state court): Probation officers
sued Maine, alleging that the state had violated the overtime provisions of the 1938 Fair Labor
Standards Act. Following the Court's decision in Seminole Tribe v. Florida, the probation officers' suit
was dismissed in federal district court. The officers then sued Maine again for violating the Fair Labor
Standards Act, this time in state court. The state trial court and the state supreme court both held that
Maine had sovereign immunity and could not be sued by private parties in their own court. Held:
Congress may not use its Article I powers to abrogate the states' sovereign immunity.
Pg. 928started off in fed. ct & goes to state ct after Seminole b/c Union Gas is no longer
good law. When they get to state ct, Maine comes in & claims sovereign immunity & that
Congress cant force them to get sued. S.C. upholds it 5-4.
Kennedys theory is the 11th A doesnt grant sovereign immunity from suit.
Pg 931: deeply offensive to sovereign dignity to subject them involuntary to suit. Isolate from
suit. 11th A is to be understood as an illustration. Pg. 933all the places where it doesnt
apply. Ex Parte Young is not displaced, 5 abrogation is still allowed, doesnt say anything
about fed. gov. suing to enforce Congress laws (ex. Solicitor General can sue).
It isnt constitutional, its natural law. Ct decided Seminole wrong. Multiple sovereigns at
play& subordinate sovereign will not be immune where the source of the right of action is the
sovereign that is dominant. He attacks the basic rationale that the majority gave.
2. Sovereignty & Dignity Rationale
Has a lot of potential for growth. It is extra-textual. The next big case is FMC v. South
Carolina State Ports Authority:
FMC is an Art. I agency. Claim came up against a state agency & Ct sad it would
harm the states agency. But its not a ctnot sovereign, so 11th A doesnt apply. It
is a substantive departure of sovereign immunity.
Dissent takes issue w/difference b/w state agency & ct. Its meant to protect the
states (the adjudicatory function allows states to prove that the rule doesnt apply)
dont offend the dignity.
Majority says indignity is not diminished b/c Art. I agency instead of Art. III Ct.
Pg 940how do you reconcile holding in Alden w/Testa? Sovereign immunity is a valid
excuse.
Note 955claims were made that S.C. needed to cut down on 1983 suits. Note suggests it
never was a big deal that ppl opposing 1983 made it seem
Barney v. NY (945): ct held if they violated state law TOO, it proves its not a state action.
Home Telephone v. City of LA
If claim is that its violating 14th A, then its violating state law.
S.C. to go along w/LAs argument would paralyze cts. If fed. cts had to wait until
state cts decided (could not move forward w/any action). Either way, taking away
fed power, it becomes subsidiary to the state action or not at all.
945Ct says 14th A applies to state action. (A state officer cant on 1 hand, as a
means of doing a wrong forbidden by the amendment, proceed upon the assumption
of possession of state power & at the same time, for the purpose of avoiding
application of the amendment, deny the power & accomplish the wrong. The Barney
case succumbs to Home Telephone.
3. Inherent in the Nature of a Sovereign: Although the immunity of one sovereign (e.g. a state) in the
courts of another (e.g. the United States) has often depended in part on comity or agreement, the
immunity of a sovereign in its own courts has always been understood to be within the sole control of
the sovereign itself. For example, the United States has sovereign immunity in both federal and state
courts; why should it be, then, that a state can be sued in its own court.
i) Federal Government Cannot Commandeer the State Courts: Congress cannot abrogate the
States sovereign immunity in federal court (see Seminole Tribe); were the rule to be different

55

Downloaded From OutlineDepot.com

Kannar Federal Courts, Spring 2011

here, the National Government would wield greater power in the state courts than in its own
judicial instrumentalities.
ii) Other Reasons for Sovereign Immunity: Private suits against unconsenting states especially
suits for money damages may threaten the financial integrity of the States. Furthermore, States
retain a level of dignity, which would be particularly offended if they could be dragged into their
own courts by an individual.
iii) No State Sovereign Immunity in Another States Courts: In Nevada v. Hall, the Court held that
since the Constitution did not reflect an agreement between the States to respect the sovereign
immunity of one another, one state is free to determine whether it will respect another states
sovereignty as a matter of comity.
iv) Limits to State Sovereign Immunity in State Courts: Sovereign immunity does not bar all
judicial review of state compliance with the Constitution and valid federal law. Rather, certain
limits are implicit in the constitutional principle of state sovereign immunity, including (1) consent
to a suit in a states own courts, (2) suits pursuant to the plan of the Constitution and certain
Amendments; and (3) suits pursuant to statutes passed under 5 of the Fourteenth Amendment.

SUITS AGAINST STATE OFFICIALS: FEDERAL


STATUTORY PROTECTION AGAINST STATE
OFFICIAL ACTION 42 U.S.C. 1983
A.

42 U.S.C. 1983 Generally: Section 1983 creates a cause of action against any person
who, acting under color of state law, abridges rights created by the Constitution and laws of the
United States. In other words, 1983 establishes a federal remedy, cognizable in federal court,
against state officials for violation of federal statutory rights.
1. State Judicial Remedies Need Not be Exhausted: As 1983 is supplementary to any state
remedies, a plaintiff may bring a 1983 suit to federal court even if the state provides
remedies that appear adequate to redress the injuries (see Monroe v. Pape). This does not
preclude plaintiffs from presenting their claims in state tribunals, however, as federal courts
do not have exclusive jurisdiction over 1983 claims.
i) Monroe v. Pape (1961) (1983 claims are supplementary to any possible state law claims)
Monroe sued Pape, who represented Chicago, under 1983 after thirteen police officers invaded
his home, searched and arrested him without a warrant. Pape argued that the suit was improper
because under color of state authority excludes acts of an official or policeman who can show no
authority under state law. They also argued that the police violated the laws of IL, that a simple
remedy is offered for that violation, and that, thus, IL courts are available to give Monroe full
redress. Held: An action under 1983 is supplementary to a state remedy, and the state remedy
need not be first sought and refused before the federal one is invoked. It is clear that one main
reason that 1983 was passed was to afford a right in federal courts because, by reason of
prejudice, passion, neglect, intolerance, or otherwise, state laws might not be enforced and the
claims of citizens to the enjoyment of rights, privileges, and immunities guaranteed by the
Fourteenth Amendment might be denied by the state agencies. Here, it is no answer that a state
has a law that, if enforced, would give relief.
ii) In Monroe, officers investigating a murder & ransacked the home, brought them to the station &
they were released w/o charges. They sued the city of Chicago & the officers & there was no
arrest. D officers claim as to why they cant be held liable hereb/c what we did was so
outrageous, fed. law doesnt apply, not under color of law under 1983. The ct responds by saying
a violation of 4th , can drag them into fed. ct. Ct looks at why 1983 was passed. Ct goes back to
congressional debates. Under color of & under claim of are essentially the same. Part III
complaint against City was properly dismissed (b/c 1983 never addressed municipal corps as
persons) but reversed dismissal of complaint against officers. Frankfurters dissentPs have
been denied due process & state ct will hear the case, so it shouldnt be in fed law. Overreaching
on the part of fed gov. He gives legislative history of 1983. Harlan & Stewart concurlook at

56

Downloaded From OutlineDepot.com

Kannar Federal Courts, Spring 2011

legislative history also. The record is unclear but the Frankfurter opinion is less clear than the
majority. Neither majority nor Frankfurters opinion is dispositive.
iii) Who is a person?a State is NOT a person (Will v. Michigan Dept of State Police963).
Talking about individuals like the cops in Monroe (they are potentially the subject of damage
actions). Its most likely that these type of ppl will be reimbursed by the town/county under color
of law (& collective bargaining agreements).
iv) Pg. 958 (note 4)In Monroe, Ct held municipality is not a person. This issue was brought up
again in Monell v. Dept of Soc. Services held that ct misread legislative history of 1983 & that
Congress did intend to include local govs among persons it rendered liable (states & Indian
tribes still not persons under 1983). After Monell, you could sue a city. Look at state law to
determine what is & is not part of the state. If a city can be a person, what does it mean to be
subjected or causes to be subjected mean? Causation part has some substance (ex. Buffalo officer
uses badge & does bad things. Officer is liable but PD is not). Not automatically attributable to
the municipality. The city would have to have caused it to be liable (ex. City passes ordinance &
causes violation of ppls rights).

2. Usually No Suits Against Federal Officers: Although 1983 generally does not create
liability for federal officers this type of claim is properly a Bivens action instead the lower
federal courts have held that officials may be sued under 1983 when they are engaged in a
conspiracy with state officials to deprive constitutional rights.
3. States and State Agencies Cannot Be Defendants in 1983 BUT can be used against a
State Official for Prospective Relief using Ex parte Young: Neither a state nor a state
official acting in official capacity is a person within the meaning of 1983, at least when
sued for retrospective relief (Querin v. Jordan). However, if sue a state official for
prospective relief, then under Ex parte Young, they are a person (Will v. Michigan)
B. Prerequisites for 1983 Claim: A plaintiff must show BOTH:
1. DEPRIVATION of CONSTITUTIONAL OR FEDERAL STATUTORY RIGHT caused
by the exercise of a rule of conduct imposed by the State or by a person for whom the state is
responsible
2. AND that the party charged with the deprivation is a person who may FAIRLY be SAID TO
BE A STATE ACTOR (American Manufacturers v. Sullivan)
C. Proving Violation of a Federal Statutory Right: 1983 creates a cause of action
for violations of all federal statutes that create private rights, but may not be used to enforce
statutes that explicitly or implicitly preclude 1983 litigation. TEST: The courts must ask two
specific, interrelated questions
1. Has the federal statute in question CREATED A PRIVATE RIGHT within the meaning of
1983?
i) A plaintiff in a 1983 case must show Congressional intent to create a private right under
the federal statute. Anything short of an UNAMBIGUOUSLY CONFERRED RIGHT
to support a 1983 cause of action is not sufficient.
a. Gonzaga University v. Doe (2002) (no private right unambiguously created when enacted
under federal spending power): Plaintiff brought 1983 suit claiming defendants had
disclosed records in violation of his rights under Family Educational Rights and Privacy Act.
Held: There was no private right created under FERPA. The Act had been enacted under the
federal spending power, and that the sole remedy prescribed by FERPA was the withholding
of federal funds. Issue: whether a statute provides a cause of action.
b. Congress creates a right & conditions for that right: this is where an analogy to Seminole
comes in (Rehnquist pointed out procedure was very complicated. Could file Ex Parte Young
cause of action). Even if notion would be a statute that creates rights (& if statute ended
there, you would have a 1983 right). If Congress creates a remedial apparatus Congress
defines the substantive right.
c. 972Relationship b/w common law torts & constitutional torts in actions against state &
local officials: (1) what does Ps case have to look like; (2) Baker v. McCollan:8days before

57

Downloaded From OutlineDepot.com

Kannar Federal Courts, Spring 2011

he was released (it was his brother). No one was malicious (no mens rea to violate rights).
He did not suffer a deprivation of liberty under 14th A.
d. Parratt v. Taylor
If warden was purposely w/holding mail then it would be a 1983 violation. But what
if they truck breaks down (no mens rea). Marshall asks what if they mail truck
breaks down every day (what about subjective intent).
Inmate orders hobby materials in the mail. He was in solitary& someone else signs
for it & it gets lost. He sues under 1983 for being deprived of his mail.
Note on 972: there is a question of state of mind requirement under 1983. There was
no intentionality in depriving D of his mail. The Ct on pg 974 notes that the state has
laws for tortuous losses at the hand of the state.
974 (part II): 1983 doesnt limit itself to purely intentional act. D claimed he had a
right to his property, someone acting under color of law deprived him of this right
it violated his due process.
Justice Powelldeprivation under Baker different rights may have different
intentionality requirements. Deprivation connotes intent.
Rehnquists majority opiniongoes to talk about whether the state has an adequate
remedy. Not every deprivation is a constitutional problem. The question is whether
the state violated due process (violate due process if STATE does not provide a
remedy). Remedies state has created are adequate & there is no violation.
2. 982--Hudson v. Palmer: if intentional deprivation by unauthorized prison actionmight be a
procedural right. Daniels v. Williams: completely negligent behavior. Ct. said no constitutional
violation b/c mere lack of due care by a state officer cant constitute a deprivation of liberty or property
under 14th A.
3. Zinermonsigned himself into mental hospital & when released, sued the hospital1983 action b/c
he was deprived of liberty w/o due process. He claimed Ds should have known he was incompetent to
give consent. 3 ways to think of due process protections: (1) substantive rightsgoes to incorporated
bill of rights claims; (2) substantive rights (ex. Roe v. Wade) no procedures that can fix it. Bona fide
due process claim; (3) guarantee of fair procedure. The Ct held petitioners couldnt escape 1983
liability by characterizing their conduct in random, unauthorized violation of FL law. Pg 996in
the real world, is all of this a fairytale?Most of what were talking about is theoretical. 998there
are some officers that have absolute immunity (ex. Judges)b/c if judges can be sued, theyll be sued
all the time.

4. Has the SCHEME of REMEDIES created by Congress in the federal statute EXPLICITLY
or IMPLICITLY PRECLUDED a private remedy under 1983?
i) Rebuttable Presumption Test: An elaborate enforcement provision or an express,
private means of redress under a statute indicates that Congress did not intend to allow
for the more expansive remedy of 1983. However, presumption can be overcome by
textual indication, express or implied, that remedy is meant to complement rather than
supplement action under 1983.
a. Middlesex County Sewerage Auth. v. National Sea Clammers Assn (1981)
(comprehensive enforcement provision precluded 1983 remedy): 1983 action brought by
commercial fishermen under the statute containing elaborate enforcement provisions.
Held: Congress intended, in providing comprehensive enforcement mechanisms, not only to
foreclose implied private actions, but also to supplant any remedy that would be available
under 1983.
b. City of Rancho Palos Verdes v. Abrams (2005) (provision of private remedy precluded
1983 action): Involved an attempt to use 1983 as a means of enforcing Telecommunications
Act. Act authorized a private remedy for violation. Held: Private remedy superseded any
remedy under 1983. Provision of express, private means of redress in the statute itself is
ordinarily an indication that Congress did not intend to leave open a more expansive remedy
under 1983. This inference can be overcome only by textual indication, express or implied,
that the remedy is meant to complement, rather than supplant 1983.

58

Downloaded From OutlineDepot.com

Kannar Federal Courts, Spring 2011

D.

The Meaning of Under Color of State Law:


1. Actions taken by an officer in his or her official capacity are deemed to have occurred under
color of state law even if they violate state law. Includes actions under color of any statute,
ordinance, regulation, custom, or usage of any state or territory. .
i) Reasoning: if definition was narrower and only covered actions pursuant to official
policies, states could immunize their officers from 1983 liability simply by enacting
general statutes prohibiting officers from violating the Constitution or federal law..
2. Monroe v. Pape (1961): Defense argued that 1983 suit wasn't proper b/c under color of
state authority excludes acts of an official or policeman who can show no authority under
state law. Defense also argued that availability of state remedy preclude 1983 suit. Court (1)
rejected defendant's narrow interpretation of "under color of" and (2) said 1983 action is
supplementary to a state remedy, and the state remedy need not be first sought and refused
before the federal one is invoked.
3. State Action Test: The test for determining whether someone is acting under color of state
law is virtually identical to evaluating whether there is state action. Thus, when evaluating
whether an official acted under color of state law, look at the following:
i) On or off duty?
ii) Clothing/badge?
iii) Use of weapon/car provided by government?
iv) Purpose of action?
v) Statements asserting governmental authority?
a. Could be a private person if actions were under color of state law (or conspiring with
state)
b. So long as act is done while cloaked with authority of govt, it is under color of state
law, even if the action violates state law. (Monroe v. Pape)
4. Private persons can be sued under 1983 if acting under color of state law. Questions
included whether the government was involved enough and whether government provided a
mantle of authority that enhanced the actor's power. Things to look for:
i. Conspiracy w/ gov't official to do the act
ii. State delegated authority
iii. State delegated public function
iv. Gov't is entwined in the actor's management/control

E.

Official Immunity: Section 1983 is written in absolute terms: it creates liability for any

person, acting under the color of state law, who violates the Constitution and laws of the United
States. No exceptions are mentioned in the statute. However, Supreme Court has held that all
officers possess some degree of immunity from liability.
1. Justification: Need to strike a balance between providing sufficient liability to ensure
compensation and deterrence, while according immunities adequate to encourage government
employees to perform their duties.
F. Absolute Immunity: Those performing judicial, legislative, and prosecutorial functions
have absolute immunity from liability under 1983. The Court has also determined that police
officers serving as witnesses and the President of the United States also possess absolute
immunity. But only in those functions, so not in administrative functions for example.
1. Functional Test: Focus is on the function performed, rather than the title possessed. .
2. Judicial Immunity: Judges have absolute immunity to suits for monetary damages for their
judicial acts, even when there are allegations that the judges have acted maliciously.

59

Downloaded From OutlineDepot.com

Kannar Federal Courts, Spring 2011

i)

Limitations: No absolute immunity when a judge was acting in the clear absence of all
jurisdiction or was not performing a judicial act. (administrative function = no
immunity)
a. Stump v. Sparkman (1978) (judge of general jurisdiction decision was covered by absolute
immunity): Judge had approved ex parte a petition filed by parents of a fifteen year-old girl to
have her sterilized without her knowledge. The Court ruled that the judge was absolutely
immune since he presided over a court of general jurisdiction, he had not acted outside his
jurisdiction, and he did not lose his immunity simply because no state statute specifically
authorized his conduct. Held judge was immune. Judge did not act wholly outside his
jurisdiction. Prosecutors have immunity similar to judges (but there is a limitation). Absolute
immunity of legislators. Speech & Debate Clause (pre-1st A protection for legislators).
b. Gravel v. USGravel was a senator from Alaska. Gravel got his hands on Pentagon Papers
& holds a hearing & releases the papers while newspapers are enjoined from releasing them.
He gets sued & he claims speech & debate clause. Ct extends this immunity to his aides.
c. Forrester v. White (1988) (judges firing of employee was not covered by absolute
immunity b/c administrative act): A state judge was sued by a probation officer who alleged
that the judge had dismissed her on account of her sex, in violation of the Fourteenth
Amendment. Held: The Supreme Court ruled that the judge was acting in an administrative
rather than a judicial capacity, and hence was not entitled to absolute immunity.

3. Legislative Immunity: Members of the United States Congress, and their aides, have
absolute immunity to suits for damages and prospective relief because of the Speech and
Debate Clause of Article I, 6. The Supreme Court has accorded state and local
legislators similar absolute immunity, both to suits for money damages and equitable
remedies (see Tenney v. Brandhove (1951)).
4. Prosecutorial Immunity: The prosecutorial function is accorded absolute immunity from
suits for monetary damages. Anything less than absolute immunity would lead to harassment
by unfounded litigation that would deflect prosecutors energies from his public duties and
may shade his decisions instead of exercising the independence of judgment required by his
public trusts.
i) Limitations: Absolute immunity does not protect administrative functions or
investigations.
a. Mitchell v. Forsyth (1985) (wiretapping was not part of attorney generals prosecutorial
function): A former Attorney General could claim only qualified, good faith immunity for his
decision to wiretap. Such conduct was not prosecutorial in nature.
b. Burns v. Reed (1991) (advice to police was not prosecutorial function): Plaintiff was
arrested by police and then was forced to spend four months in a psychiatric ward & undergo
lots of hypnosis. Held: Prosecutors have absolute immunity for their in-court behavior, but
not for their advice to police officers. Thus, the prosecutor, Reed, had absolute immunity for
statements at the probable cause hearing. However, he only had qualified immunity for
approving the hypnosis by the police officers.

5. President of the United States: The President of the United States has absolute immunity to
suits for money damages for acts done while carrying out the presidency. This is because the
presidents unique status under the Constitution and the singular importance of his duties
justify such protection.
6. Public Defenders dont have immunity because they cant be sued under 1983 (they are not
acting under color of state law)
G. Qualified Immunity: If officers are not performing a function accorded absolute immunity,
they are entitled to good faith immunity, aka qualified immunity. Most executive branch and
administrative officials can claim only good faith immunity.
1. Objective Good Faith Standard: Government officials performing discretionary
functions are shielded from liability for civil damages as long as their conduct does not

60

Downloaded From OutlineDepot.com

Kannar Federal Courts, Spring 2011

violate clearly established statutory or constitutional rights of which a reasonable person


would have known.
i) Harlow v. Fitzgerald (1982) (qualified immunity for aids, objective standard): Fitzgerald was
terminated from his federal job and claimed it was due to whistle-blowing and sued presidential
aides. Good faith immunity is an affirmative defense for governmental officials, but qualified
immunity is not available if the official knew or should have known that his actions were illegal.
This reliance on an objective element rather than a subjective test should avoid excessive
disruption of government and permit the resolution of many claims on summary judgment.
Fitzgerald is a civilian employee of the air force & is a well-known whistleblower. Nixon wants
him fired, Harlow fires him & Fitzgerald sues for 1st A violation.
Qualified immunityif acting outside official capacity, you can be sued.
Presidential aides argue if legislative aides get immunity & president gets absolute
immunity then they should get absolute immunity. Ct doesnt agree b/c it could extend to
all members of the cabinet. Ct says they have qualified immunity.
Subjective or objective standards would keep Ps claim going & would defeat qualified
immunity & holds officials performing discretionary functions, generally are shielded
from liability for civil damages as their conduct does not violate clearly established
statutory or constitutional rights or which a reasonable person would have known.
Concurrence: structure raises questions of its own.
Burgers dissentthought case should have gone the same way as Gravel. Its unfair that
legislative aides get more protection than presidents aides.
What does clearly established law mean?
Next, litigate to determine if something is clearly established. Wilson
v. Laynetells what it cant be. How precise does the holding have to
be?Contours of the right must be sufficiently clear that a reasonable
official would understand that he is doing violates that right. Apply a 2
step process.

2. Two Part Test: (Wilson v. Layne):


i) Whether the plaintiffs CONSTITUTIONAL/FEDERAL RIGHTS have been
VIOLATED. If Yes
ii) Whether it was a CLEARLY ESTABLISHED RIGHT that a REASONABLE
OFFICER should have known.
a. The purpose of qualified immunity is to give fair warning to government officials, it
need not be shown that the action in question had previously been held unlawful so
long as in the light of preexisting law the unlawfulness was apparent.
b. United States v. Lanier (1997) (clearly established law even in absence of Supreme Court
decision): State judge was prosecuted for sexually assaulting women while serving as a judge.
Sixth Circuit held that there was no clearly held constitutional right to not be sexually
assaulted by a judge. Held: Court reversed, stating that it is possible for a right to be clearly
established even in the absence of a Supreme Court decision so holding, as disparate
decisions in the lower courts might well preclude such a determination from ever being
presented.
c. Wilson v. Layne (1999) (no clearly established law without any controlling or persuasive
authority): A right had not been clearly established when plaintiffs had shown neither
controlling authority in the jurisdiction where the conduct occurred nor a consensus of cases
of persuasive authority such that a reasonable officer could not have believed that his actions
were lawful.
d. Anderson v. Creighton (1987) (contours of right must be sufficiently clear that officers
knows personal conduct violates): The legal rule that was allegedly violated was at too high
of a level of generality. The contours of the right must be sufficiently clear that a reasonable
official would understand that what he is doing violates that right.
e. Hope v. Pelzer (2002) (clearly established if officers given fair warning): Prisoner
handcuffed to a hitching post for disruptive conduct. Dont need prior decision on point to

61

Downloaded From OutlineDepot.com

Kannar Federal Courts, Spring 2011

show clearly established law. Officers can be held liable so long as they had fair warning
that their conduct was impermissible. In this case, the clarity of the violation was established
by a related Eighth Amendment holding by the Supreme Court, two of the Eleventh Circuits
own cases relating to hitching prisoners, a state regulation prohibiting the action, and a U.S.
DOJ report advising state department of corrections to stop using the practice.
1. But there are instances where its not completely objective. E.g. in Iqbal,

intent is required for a discrimination claim. Another example: cruel and


unusual punishment in prison: (must be malicious and for the purpose of
causing harm) This can be subject to qualified immunity defense
because intent is an element. What if official received a memo from
superior saying the activity is perfectly ok (even if memo is not a
colorable interpretation of the Constitution?)
f. Saucier v. Katz: if there is a right (decided 1st) then decided no right, havent you
violated Ashwander. (If not a clearly established right, then case goes away). Ct
concludes there was a right but not a clearly established right.
g. Pearson v. Callahan: ct thought there should be a better way to do this.
H.

Municipal Governments Sued under 1983: Municipal governments may be sued


for their unconstitutional or illegal policies. However, they may not be sued for the acts of
employees under respondeat superior theory.
1. Monell v. Department of Soc. Servs. (1978) (local governments may be sued under section 1983, no
respondeat superior): The Court held that Monroe v. Pape had misread the legislative history of 1983
and that Congress did intend to include local governments among the persons it rendered liable. In
the view of the Court, rejection of the Sherman Amendment could not justify an inference that
Congress sought to exclude municipal liability for the conduct of officials.

2. Proving an Official Municipal Policy: After Monell, there are at least five possible ways to
establish the existence of a policy or custom sufficient to impose 1983 liability on a
municipal government
i) Actions of Municipal Legislative Bodies: Actions taken by the municipals legislative
body, even a single decision, constitute official government policies.
a. E.g. a city councils firing of a governmental official without providing procedural due
process and a city councils cancellation of a concert in violation of the First Amendment
were properly the basis for 1983 liability.

ii) Agencies Exercising Delegated Authority: Official policy exists when there are actions
by municipal agencies or boards that exercise authority delegated by the municipal
legislative body.
a. E.g. In Monell, the plaintiffs challenged regulations adopted by the Department of Social
Services and the Board of Education requiring pregnant employees to take unpaid leaves of
absence. The Court found that actions of these agencies unquestionably involve official
policy.

iii) Individuals with Final Decision-Making Authority: Actions by officials with final
authority for making a decision in the municipality constitute official policy for the
purposes of 1983. Whether or not the official is responsible for establishing final
governmental policy is a question of state law, as such authority can be granted
legislatively or delegated from higher officials.
a. Pembaur v. City of Cincinnati (1986) (prosecutor had final decision authority under state
law): To serve subpoenas the county prosecutor issued instructions to go in and get the
witnesses. The police chopped down the door with an ax. Held: Based on the relevant Ohio
law, the Court concluded that the prosecutor had authority for making the final decision, and
hence, his decision constituted the Citys official policy. Pg 961single decision of high
official, who had authority under state law to decide whether officers should enter & whose

62

Downloaded From OutlineDepot.com

Kannar Federal Courts, Spring 2011

decision may fairly be said to represent official policy was an adequate basis for imposing
gov. liability under 1983.
b. McMillian v. Monroe County (1997) (sheriff is not final decisionmaker under state law):
Under Alabama law, a county sheriff is a state official, not a local official, and thus is not a
final decision-maker for the local government.
c. City of St. Louis v. Praprotnik (1988) (authority is a question of state law, not fact): The
determination of whether a person has final decision-making authority in a particular area is a
question of law for the judge to decide, not a question of fact for the jury to resolve.

iv) Policy of Inadequate Training, Supervision, or Screening: A government policy of


inadequate training, supervision, or screening is also a way of demonstrating an official
policy. However, only where a failure to train, supervise, or screen reflects deliberate or
conscious indifference to the fact that a violation of federal rights is a highly
predictable consequence of the failure will a municipality be liable.
a. City of Canton v. Harris (1989) (failure to train must amount to deliberate indifference):
Police failed to treat medical ailment b/c of poor training. Held: The inadequacy of police
training may serve as the basis for section 1983 liability only where the failure to train
amounts to deliberate indifference to the rights of person with whom the police come into
contact.
Presents the possibility of solving OShea problem by a newly conceived cause of
action. Duty to run dept/system systematically. Must adequately trained/failure to
supervise or train can provide cause of action against entity & supervisor.
b. Connick v. Thompson (not in the book)whether you can prove a case of deliberate
indifference by showing 1 particular egregious action. DA w/held exculpatory evidence & D
sued (1983 action). Majority says a cause of action hasnt been established b/c it was a single
instance. Cant make a deliberate indifference case based on 1 instance.
c. Board of County Commissioners v. Brown (1997) (a single hiring decision is not official
policy): The plaintiff sued county, b/c deputy arrested her with excessive force and county
hired the deputy without adequate review of his background, which included a conviction for
assault and battery. Held: A single instance of inadequate screening was not sufficient to
prove that the municipality caused the injuries. The plaintiff must establish deliberate
indifference on the part of the policymaking representative of the municipality, not merely to
the risk of any constitutional injury but of the particular injury suffered by the plaintiff.
P must establish deliberate indifference (how to show they knew about it & didnt
change. Ex file complaints & nothing is done). Have to be on notice of
particular/specific issue.

v) Custom: Municipal governments can be sued for their customs that cause constitutional
violations, even though such a custom has not received formal approval from the official
legislative body.
a. Unlike a policy, which comes into the existence because of the top-down affirmative
decision of a policymaker, a custom develops from the bottom-up. Thus, the liability
of the municipality for customary constitution violations derives not from its creation
of the custom, but from its tolerance or acquiescence of it.
3. No Qualified Immunity for Municipalities: There is no qualified immunity for local
governments, and as a result, municipalities will be liable under 1983 even when their
constitutional violations are a result of actions taken in good faith.
i) Owen v. City of Independence (1980) (no qualified immunity for local government): A city
council fired the police chief without providing him any procedural due process protections. The
city claimed immunity because its actions were done in good faith. Held: The fact that the city
officials acted in good faith did not protect the municipal government from liability under 1983.
This is so even if the officials would themselves be entitled to such an immunity in a 1983 action
against them in their personal capacity.
For individual Dsshould they have known actions would violate citizens rights
Get some immunity in gray areas. Not chilled.

63

Downloaded From OutlineDepot.com

Kannar Federal Courts, Spring 2011

Held a municipality sued under Monell for violations committed by its officials does not
have a qualified immunity from damages liability under 1983, even if it can show that the
officials would themselves be entitled to such an immunity in a 1983 action against them
in their personal capacity.
4. Hard cases: determining what is attributable to the entity. When pattern & practice is so
unconstitutional for so long attribute to entitydifficult to determine.

5. No Punitive Damages against Municipalities: Municipal governments have immunity to


claims for punitive damages. There was an absence of municipal liability for punitive
damages at common law when 1983 was adopted. Also, punitives exist to punish and its
unfair to punish citys taxpayers because of an officials wrongdoing.
Suits vs. Officials Personal Capacity:
The official, not the govt pays:
Judgment
Atty fees
So state sovereign immunity does not apply
Even tho, in practice, govt may indemnify
But official immunity doctrines limit relief
Qualified Immunity
Complete Immunity
Supervisory liability (suind a supervisor for misdeeds of underling) Does it survivor after
Iqbal?
Circuit Courts said if you put policy in place that show you were deliberately indifferent
to violation of Const, you can be held liable for supervisory liability.
But, in Iqbal, guy claimed he was being discriminated against based on religion,
but the officials being sued were fed officials, so case is under Bivens, not 1983.
Court says supervisory liability doesnt exist since there is no respondeat superior
liability for Bivens or 1983 claims. Officials can only be liable for THEIR OWN
conduct, not the conduct of others.
But does this eliminate supervisory liability? Supervisory liability
involves actions by the supervisor which play a role in the constitutional violation of
the lower official.
Effects: maybe just cutting back on Bivens since SCOTUS doesnt like
extending it, but Court analyses 1983 and Bivens claims under same standards.
Could read causes to be subjected in 1983 to mean there is supervisory
liability. But SL wasnt briefed by parties. Also, discrimination charge required
intent, so maybe that was a problem for finding SL.
Damages award would come from govt
The rules re suits against govt apply
But P can get equitable relief vs state employee in official capacity
Under Young
I.

State Waiver or Consent: A state can waive its immunity and consent to be sued in
federal court.
1. Explicit Waiver by Statute: To be effective, a state must expressly to agree to be sued in
federal court.
i) A states consent to be sued in its own state courts is not sufficient to constitute a waiver
of its Eleventh Amendment sovereign immunity (see Smith v. Reeves).
ii) A general waiver of a states sovereign immunity is not enough.
iii) Present in field regulated by Congress not enough: FL Prepaid Postsecondary Expense
Bd.: Florida Prepaid, a state agency, copied College Banks patented system. College Bank sued

64

Downloaded From OutlineDepot.com

Kannar Federal Courts, Spring 2011

for patent infringement. Held: The state did not constructively waive its sovereign immunity b/c
consent cannot be based upon the states mere presence in a field subject to congressional
regulation (e.g. patent law).

2. Constructive Waivers for accepting Federal Funds Usually NOT Allowed: Based the
states acceptance of federal funds for a particular purpose, on its engaging in federallyregulated activity, or on its conduct in ongoing litigation. However, after some years of
uncertainty in the law, it appears that constructive waivers are disfavored and will rarely be
found.
3. Constructive Waiver When State Voluntarily Chooses to Enter Field where Congress
has abrogated sovereign immunity:
i) If constructive waiver ever will exist, it will be in a situation in which Congress indicates
a clear intent to make state liable in federal court if they engage in a particular activity,
and
a. The congressional desire to make states liable must be in unmistakable language in
the statute itself and
i. The state voluntarily chooses to engage in that conduct.
a. It must be in an area where the state realistically could choose not to
engage in the activity.
4. Removal/Appearance Waiver: Waiver by Participation in Judicial Proceedings:
Although it remains unsettled whether, generally, a states choice to remove a case to federal
court constitutions a waiver, the Court has ruled that a states choice to remove a case from
state court to federal court constitutions a waiver when there are state law claims and the
state had explicitly waived its immunity as to those claims in state court.
i) Lapides v. Board of Regents (2002) (waiver by participating in judicial proceedings in federal
court): Suit by a professor in GA state university filed in state court against university and its
officials. GA waived sovereign immunity for this type of claim in state court, so the state removed
the case from state to federal court and then moved to dismiss based on the Eleventh Amendment.
Held: The Court ruled that the states choice to remove the case to federal court was a waiver of
its sovereign immunity. It would seem anomalous or inconsistent for a state both (1) to invoke
federal jurisdiction, thereby contending that the judicial power of the United States extends to the
case at hand, and (2) to claim Eleventh Amendment immunity, thereby denying that the judicial
power of the United States extends to the case at hand.

5. Counterclaims: circuit splits on whether state immunity holds up if state sues you and you
have counterclaims.
i) Compulsory counterclaims: seems like state shouldnt be able to bring suit that would
compel a counterclaim and be immune to that counterclaim

Judicial Federalism
A. The Anti-Injunction Act: A fed. ct cant stop a proceeding in state ct unless the fed ct is (1) expressly
authorized by fed. statute; (2) necessarily trying to aid the district cts jurisdiction; or (3) this injunction is
necessary to protect & effectuate the district cts judgment.
B. In Atlantic Coast Line Railroad Co. v. Brotherhood, the Supreme Court noted that the anti-injunction
statute is an absolute prohibition against enjoining state court proceedings unless the injunction falls within
one of the three statutory exceptions to 2283.
1. It was a labor dispute. It went to fed. ct to get injunction, but it was denied, so they went to state ct &
nothing happened. Another suit was brought to S.C. & Ct allowed picketing. Under the new
precedent, they went back to state Ct to remove injunction, they wouldnt, district ct enjoined the
states injunction & case gets to S.C. Ct looks to see if district ct applied an exception (either order is
not clearly expressed the 1st district ct determine it didnt have power to issue an injunction in labor
dispute b/c of Norris LaGuardia Act.
2. 1st prong of argument: b/c district ct dismissed the 1st caseit was in aid of its jurisdiction

65

Downloaded From OutlineDepot.com

Kannar Federal Courts, Spring 2011

3. 2nd prongonce the decision in Jacksonville Terminal was announced, district ct was free to enjoin the

C.

D.
E.

F.
G.

state ct. Claim that the ct was in aid of its jurisdiction. But ct says there was no judgment to protect
b/c there was no judgment.
4. Justice Brennan dissented and argued that the federal rule of labor management relations would be
displaced if there was no ability of the federal courts to enjoin state courts. In 1967 case, the district ct
said that it was a judgment on the merits & had right to picket.
Mitchum v. Foster (1025): The very purpose of 1983 was to interpose the federal courts between the
states and the people, as guardians of the peoples federal rights. Congress plainly authorized the federal
courts to issue injunctions in 1983 actions, by expressly authorizing suit in equity as one of the means
of redress. The Court has held that the test for determining whether something is an express exception to
the Anti-Injunction Act is whether an act of Congress, clearly creating a federal right or remedy
enforceable in a federal court of equity, could be given its intended scope only by the stay of a state court
proceeding. This amounts to two requirements: (1) a federal right in equity and (2) this right or remedy
must be frustrated if a federal court were not empowered to enjoin a state court proceeding. 2283 subject
to practical concerns. Notwithstanding the language of the statute, the statute is deemed to be exceptions
have been discovered/relied uponremoval, interpleader, habeas, etc. Mitchum authorizes P to go to fed ct
if they feel theyre getting the raw end of the deal.
Think of in rem as in aid of it jurisdiction to enjoin a state ct. Could re-litigate instate ct to enjoin. Superior
gov interestwhether 1983 expressly authorizes exception of the anti-injunction act?YES!
Expressly authorized doesnt mean: (1) fed law doesnt have to contain express reference to the statute; (2)
fed law doesnt have to expressly authorize an injunction of a state ct proceeding in order to qualify as an
exception & (3) in order to qualify as an expressly authorized exception to the anti-injunction statute, an
Act of Congress must have created a specific & uniquely fed right or remedy, enforceable in fed ct of
equity. 1983 is expressly authorized b/c of its nature & history. Legislative historywhat it was meant to
do, is deemed to expressly authorized.
Notes: Removal & in rem are easy parts of in aid of its jurisdiction. If its removed to fed ct & pending
review can put injunction on state case. Relitigation exception (1037): permits a fed ct to enjoin a state ct
to respect preclusive effect of a fed. judgment.
Chick Kam Choo v. Exxon Corp. (1988) (claim must have been decided on merits in federal court): A
federal district court in Texas had dismissed plaintiffs wrongful death action. P filed in state court,
asserting claims under Texas law and under Singapore law. Held: The Court ruled that 2283 did not
preclude the injunction insofar as it barred relitigation of the Texas law claim, which the federal court had
previously held to lack merit when it held that Singapore law applied. But the Court overturned the
injunction insofar as it barred state court litigation of the claim based on the law of Singapore: because
federal and state forum non conveniens might differ, and it had not been decided in state court. District ct
dismissed Ps wrongful death action (should apply case in Singapore). They file in state ct & Ds go to
district ct to enjoin state at action. S.C. relitigation exception did not apply to everything. Seem to be 2
sets of claims: Texas & Singapore claims. Okay to enjoin state cts from deciding state claims but not
Singapore claims.

i)

Timing Limitation Need to Get Injunction Before State Rules on Res Judicata: Can
only use relitigation exception when state court has not ruled on res judicata. Once the
res judicata issue is raised in state court and decided, then the federal court must accept
the states determination that there is no preclusion.
a. Parsons Steel v. First Alabama Bank (1986) (injunction must be issued before state court
rules on res judicata): Federal court decided first, ruling in favor of defendant. Plaintiff then
pursued state court suit. Defendant raised res judicata as defense, but state court held that
earlier federal court decision did not preclude state proceedings. Held: Federal court could
not enjoin state court. Because the state court had already ruled on the merits of the res
judicata issue. Once the state court has finally rejected a claim of res judicata, federal courts
must turn to state law to determine the preclusive effect of the state courts decision.
b. Parsons (1037)fed action comes to judgment 1st & bank wins. Bank asserts res judicata
defenses in state ct it was rejected & 4 mil verdict against the bank. Bank goes to fed ct to
get an injunction against state ct. S.C. using the full faith & credit clause. Overturned the
injunction b/c relitigation exception was limited to those situations in which the state ct has

66

Downloaded From OutlineDepot.com

Kannar Federal Courts, Spring 2011

not yet ruled on the merits of the res judicata issue. Must make appropriate motions in state ct
before you go running to fed ct.

c. Increases Amount of Federal Court Injunctions: This rule creates a strong


incentive to not litigate the preclusion issue in state court. Rather, many litigants will
immediately seek federal court injunctions even in many instances where the state
court would have dismissed the case anyway on preclusion grounds.
i. Criticism: This seems inconsistent with the Supreme Courts general preference
that matters be litigated in state courts where possible and its general desire to
avoid injunctions of state judicial proceedings.
ii. Declaratory judgments (1039): its not that youre asking to enjoin state
proceedings. But think of artful pleading (like an injunction).
H. Judicially-Developed Limitations on Federal Courts Jurisdiction: Doctrines of Equity, Comity &
Federalism. When is it appropriate to issue an injunction (injunctions are discretionary no in equity).
Though they have power to do something, they dont (abstention). Question (1050)when should fed cts
defer to the state system? When does a P have to go to the state system (also like what is seen in ripeness)
but now it is exhaustion. From cts perspective, it looks like ripeness. Have to exhaust state administrative
remedies.
Patsy Case: 1983gender & race discrimination. She went to fed ct. Dont have to exhaust
state remedies for a 1983 claim b/c states are potential evil-doers in eyes of Congress that
enacted 1983 (this is based on legislative history). Exceptions to the Patsy Rule: (1) actions by
prisoners & (2) plain, adequate & complete tax remedies (Real Estate Assn v. McNary).

ABSTENTION & JUDICIALLY-DEVELOPED LIMITATIONS


ON FEDERAL JURISDICTION
A. Generally: The central issue of this section is whether, and if so in what circumstances, it is
appropriate for federal courts to abstain from entertaining actions that fall within the literal
terms of congressional grants of jurisdiction.
1. Reasoning For: Desires to avoid premature constitutional determinations (if state court can
make decision w/o implicating Constitution), to defer to state tribunals on questions of state
law, to avoid duplicative proceedings, and to interfere as little as possible with state
proceedings.
i) Against: Competing impulses are to uphold a litigants choice of a federal forum, to
respect the policies of the jurisdictional grants, and to vindicate federal rights without
undue delay.
B. Exhaustion of Nonjudicial Remedies: Federal courts will normally abstain from
hearing federal claims if the plaintiff has not exhausted legislative and/or administrative state or
federal remedies.
1. Must Exhaust Administrative Remedies: A federal court will not entertain an action against
a state officer if the plaintiff has failed to exhaust remedies before a state administrative
agency.
i) This is meant to avoid premature interruption of agency procedures, to permit proper
factual development, to take advantage of the agencys expertise, to give the agency the
chance to correct its own errors, and to promote efficiency in both the courts and admin.
a. Exception Section 1983 Claims: The Supreme Court has ruled that the exhaustion
of state administrative remedies is not required in actions under 1983 (1983 was
meant to put the federal courts btw the people and the state court in order to protect
their federal rights)
i. Limitation Actions by Prisoners: The Prison Litigation Reform Act of 1995
requires the exhaustion of such administrative remedies as are available prisoner
to the filing of federal suits by prisoners challenging prison conditions under
1983 or any other Federal law.

67

Downloaded From OutlineDepot.com

Kannar Federal Courts, Spring 2011

ii. Limitation Tax Remedies: Federal courts are required to decline jurisdiction

in suits seeking damages remedy for state taxation whenever the state provides a
plain, adequate, and complete remedy (Real Estate Assn v. McNary (1981).
C. Abstention When a Post-deprivation Remedy : Adequate post-deprivation judicial
remedies can sometimes provide all the process that is constitutionally due, and thus eliminate the
basis for a federal suit under the Due Process clause (Parratt v. Taylor)
D. Younger Abstention: Equitable Restraint from Enjoining State
Criminal Prosecutions (now extended to more situations) : Federal courts are
generally prohibited from interfering with pending state criminal prosecutions, even if there is an
allegation of a constitutional violation and even though all jurisdictional and justiciability
requirements are met. Equitable principle based on comity (respect for state court proceedings)
and Federalism (protect federal rights but in ways that dont unduly interfere w/ state activities)
Doesnt prevent the race to the courthouse (get injunction before proceedings begin), P must also
exhaust appeals b/c the proceedings are still pending.
1. Normally, to get an injunction against state court proceeding, party must show: great
and immediate IRREPARABLE INJURY and LACK OF ADEQUATE REMEDY:
Normally Fed Courts will not enjoin state court proceedings.
i) When D can raise Constitutional/federal/etc defense, thats an adequate remedy, no
injunction: Younger v. Harris (1971) The plaintiff got injunction against state criminal
prosecution on the grounds that the Act he was prosecuted under violated the First and Fourteenth
Amendments. The Supreme Court reversed b/c federal courts cannot enjoin pending state court
proceedings except under special circumstances. The defendant could raise his constitutional
claims as a defense to the state court prosecution. Hence, there was a preexisting remedy that
made the injunction unnecessary, and out of commiserations of equity and comity, the injunction
was denied.
a. State courts are presumed to be as competent as federal courts in deciding federal

claims or defenses.
b. There is little reason to grant the federal injunction if the state proceeding is actually
pending, since the federal defense can be presented to the state court with reasonable
promptness (i.e. federal defense will be decided timely by a competent forum).
2. Exceptions to Younger: Three exceptions to Younger, whereby a federal court CAN issue an
INJUNCTION to cease pending state criminal proceedings
i) Bad Faith Harassment: If the state prosecution is brought in bad faith or for the
purposes of harassment. The definition of a bad faith prosecution is when a prosecution
has been brought without reasonable expectation of obtaining a valid conviction.
a. Dombrowski v. Pfister (1965) : If prosecutors repeatedly arrest and indict, but dismiss
charges before trial for harassment purposes, the individual has no opportunity to assert his or
her constitutional claims. So its an exception to Younger.

ii) Flagrantly and Patently Unconstitutional Statute: Injunctions would be appropriate if


there was a statute that is flagrantly and patently violative of express constitutional
provisions in every clause, sentence and paragraph, and in whatever manner and against
whomever an effort might be made to apply it. There is not a single instance where the
Court has applied this.
iii) Other Unusual Circumstances i.e. Bias of State Tribunal: The Court has held that
where the state tribunal before which the federal claim or defense would be asserted is
biased, a federal court may issue an injunction to block the criminal prosecution (Gibson
v. Berryhill (1973)).
3. Declaratory Judgments are Generally Prohibited just like Injunctions: In Samuels v.
Mackell (1971), the Court held that the Younger doctrine applies not only to injunctive but
also to declaratory relief against a pending state criminal proceedings. This is because a
declaratory judgment would result in precisely the same interference with state proceedings

68

Downloaded From OutlineDepot.com

Kannar Federal Courts, Spring 2011

as an injunction because a declaratory judgment could be enforced, if necessary, with a


subsequent injunction.
i) If the proceedings arent pending, could be allowed.
E. Pullman & Related Doctrines: substantive area of law made up of judge-made (S.C. made) doctrines as a
matter of deference, they are ordered.
F. Railroad Commissioner of Texas v. Pullman
1. the railroad commission enacted a law saying that no sleeping car could be operated unless it was
continuously in the charge of an employee having the rank and position of a Pullman conductor. There
are three requirements here: (1) unclear state law, (2) federal constitutional question; and (3) adequate
state remedy (today, you can just use a declaratory judgment action. Pullman abstention is usually not
discretionary. Once these three factors are met, the federal court should Pullman abstain. Usually,
Pullman abstention cases involve injunctions against state officers. Frankfurters opinion points to the
idea that Pullman is just about equity cases.
2. RR Comm. was making an orderif you have a train w/only 1 sleeping car, the position of overseeing
that car must be a conductor (a white person) & theres a 14th A challenge to it, commerce clause issue,
question if commission had power under Texas Law to give order.
3. Ct says the Texas law is genuinely unclear (whether commissioner has such authority). If ct were to
decide if commissioner has authority the equal protection claim will go away. Lower ct should stay
proceeding until state supreme ct has made a ruling about Texas state law (stay of fed equal protection
question).
4. It flies in the face of Marshall in Cohens v. VA (abdications of judicial duty. Not authorized not to
decide something). All equitable judgments are discretionary (Frankfurter in Pullman).
a. Costs of going forward in Frankfurters generic friction b/w fed &state law. If fed ct decides
what state law is & when state ct decides the issue differently, state interpretation will stand.
b. Note 7 (1068)problem of delayb/c state issue have to defer adjudication of the claim.
Pullman is inconsistent w/Siler.
c. Abstention is a live doctrine. Abstention got extended from equitable cases to damages cases.
Statute of limitations is1 reason you would prefer a stay instead of dismissal or remand.
Would extending Pullman to diversity jurisdiction be good or reasonable? (1065).No, still
will have the same problem b/c have to decide state law in a diversity case. Under Warren Ct,
there was a decline in abstention.
d. Fed. ct doesnt have discretion NOT to decide fed statutory question, dont wait for the states.
e. 1 requirement is that state law is unclear, but question of how unclear is unclear. Novelty of
state law is a factor. It would really make constitutional question go away. Remember, its
equity discretion (1069 n 8). Pg 1072 n5in modern times, thanks to ALI, all states except
Missouri & N.C. permitted fed cts to certify uncertain state law issues to state supreme ct for
authoritative resolution. Its easier to stay the case. Certification casescertification offers a
more expeditious method of obtaining state ct resolution of state issues than would classic
Pullman Abstention.
5. Frankfurter: unclear question of Texas Law (about commissioners power) & high ct of Texas should
decide. If it is decided that commissioner overstepped law, state could dispose of the case w/o
discrimination issue (fed constitutional law)
G. A Pullman abstained Plaintiff, though, Can Still Have his Day in Federal Court by Instructing the
State Court Not to Rule on the Federal Issues:
1. England v. Louisiana (1071).
a. the Court held that abstention merely postpones federal court rulings on constitutional issues;
it does not abdicate the federal courts role, though. The decision in England is a non-parity
decision. The Court is saying that there is a primacy of the federal judiciary in deciding
questions of federal law. Thus, there is something special about federal court vindication of
federal rights.
b. England procedure is well entrenched in Pullman doctrine. A party is bound by state ct
determination ONLY if the party did in fact elect to seek a complete & final adjudication.

69

Downloaded From OutlineDepot.com

Kannar Federal Courts, Spring 2011

H. Pullman abstention also applies in 1983 cases, even though 1983 does not have a state exhaustion

I.

J.

K.

L.

M.

N.

principle: In Wisconsin v. Constantineau, the Court held that a 1983 case can be abstained when there is
an unclear state law issue.
1. Here, the majority, though, decided not to abstain since there was no unclear state law issue.
2. Justice Burger dissented and argued that state courts should be accorded the opportunity to address the
constitutionality of a state statute, whether the claim is either state or federal.
3. The important thing about this case is that the plaintiff could have raised a state constitutional claim,
but didnt. As such, someone cannot be abstained on an issue he did not raise.
When a Plaintiff is Pullman abstained, the District Court Usually Retains Jurisdiction Until the State
Court Decides the Unclear State Law Issue: However, in Harris County Commissioners v. Moore, the
Supreme Court held that a district court should dismiss jurisdiction, so that a Texas court could hear the
case (since Texas had a statute saying that a Texas court could not hear the case if a federal court had
concurrent jurisdiction).
1. The holding in Moore very much disrespects states rights. There is no regard for those rights, because
the Court is just side-stepping the Texas statute.
Sometimes the Court abstains, even when no one argues this issue and when the district court fails to
recognize the issue. In Kaiser Steel Corp. v. W.S. Ranch. Co., for example, the Court held that abstention in
a water right suit was appropriate, even though the Court never mentions what type of abstention this is.
The motivating factors were that the state had a pending action and water right usage was so novel at the
time in New Mexico.
Why We Dont Have Many Pullman Abstention Cases Now: (1) The unclear state issues are certified to
state courts; (2) there are justiciability problems; (3) Lots of the cases are subsumed under the Younger
doctrine; (4) The Eleventh Amendment forbids the federal courts from enjoining a state officer for
violations of state law.
Why Pullman Abstention is Good: (1) Avoids unnecessary tentative constitutional holdings; (2) Respect
for federalism and letting state courts decide state issues (this isnt the thrust of Frankfurters opinion in
Pullman, though); (3) avoids unnecessary holdings that are overruled by the state court; (4) Friction
between the states and the federal government might result; (5) Congress has never curbed judicial
abstention practices; (6) Abstention can be likened to certiorari, which the court grants or denies; (7)
Abstention is also like standing where sometimes, for prudential reasons, the Court will either grant or deny
standing
Why Pullman Abstention is Bad: (1) Lets courts decide when they will abstain on issues, rather than
Congress this might implicate a separation of powers problem; (2) This doesnt necessarily guarantee that
the highest state court will give an answer (the state Supreme Court could deny cert on this issue, for
example); (3) Really discourages people from going to federal court, since a trial involving both state and
federal claims will likely be abstained; (4) If we want to respect states rights, just use certification (its
quicker and you get a definitive answer from the State Supreme Court); (5) Court shouldnt be able to duck
the hard issues
2 more kinds of abstention
1. Thibodaux: how is this a diversity case? Its a Florida corporation. As pleaded, the case included no
fed question. Sees as if it has something to do w/diversity of eminent domain. When there is an
unclear state law and interference with an important state policy (such as eminent domain), the
Court might abstain.
a. In Louisiana Power & Light Co. v. City of Thibodeaux, the Court abstained from hearing an
eminent domain case since the case would have involved the hazards of serious disruption by
federal courts of state government. It is possible that Thibodeaux abstention only applies to
eminent domain cases (but this was not the logic in Allegheny County v. Frank Mashuda,
another eminent domain case where there was no abstention but there was also no unclear
state law). Thibodeaux abstention has not been widely applied, so there is little to make of
this case. However, when a plaintiff is Thibodeaux abstained, he can return to the federal
court after the state law decides the unclear state law.
i. In Quackenbush v. Allstate, a majority of the Court held that when someone is abstained,
the district court stays the proceedings, but does not dismiss the case. Furthermore,
abstention is not appropriate in damage actions (it is appropriate only in equitable cases).

70

Downloaded From OutlineDepot.com

Kannar Federal Courts, Spring 2011

2. Burford: decided in 1943, it was applied in 1951 & disappears from view until it is mentioned in 1989
in NOPSI case. What is it? See pg 1078. A lot of precedent for its unavailability. The lower cts that
have used it havent used it very much. When the state has a complex administrative scheme with
review concentrated in one court and state agency action is implicated, the district court should
abstain. Most people think that Burford requires some unclear state law. Burford can occur in either a
diversity case or a federal question case
a. In Burford v. Sun Oil Co., the Sun Oil Company wanted an injunction in federal court that
would prevent the railroad commission from granting a drilling permit. The Court holds that
abstention is appropriate since questions of regulation so clearly [involve] basic problems of
Texas policy that equitable discretion should be exercised to give Texas courts the first
opportunity to consider them. In Burford abstained cases, you never get back into federal
court for review of federal issues the state administrative scheme is the sole source of
review.
O. Parallel Proceedings
1. Colorado River Water Conservation District v. US
a. Facts: McCarren Amendment doesnt talk about fora. Colorado divides water into 7different
districts. 1st, US brought suit in district ct in Colorado, but that ct was 300 miles from
Division 7 (all of the Ds). Gov. asserts its rights & rights on Indian tribes. One of the Ds
wants to join US & files suit in state ct. There was a motion to stay fed proceedings (district
ct allowed the stay but 10th Cir ruled that there was no doctrine of abstention to allow the stay
& dissolves the stay. This is what goes up to the S.C.
b. Issue: whether the case can go forward in fed ct, if district ct had the power to allow the stay.
c. Its not plainly a Pullman abstention (Pullman is not available). Ct said it doesnt fall into
Burford or Thubodaux. Part C: sound judicial administration. Different b/c concurrent
jurisdiction & fed jurisdiction are important. Cts have an ordinary duty (Art. III) to decide
cases, but when things come up, the ct abstains (pg 1134 footnote6in rem jurisdiction,
havent had sufficient proceedings on the fed case).
2. Moses Cone (1136): district ct stays the suit. Ct of appeals tells the district ct to go forward. The
district ct thought they were following Colorado River & the S.C. told them to go forward.
3. Wilton v. Seven Falls (1138): all that had been asked for was a declaratory judgment.
P. Basic doctrine of EQUITABLE RESTRAINT: equitynot enjoining a criminal prosecution.
1. Dombrowski v. Pfister
a. the Court held that a district court can enjoin a statute that banned subversive activities and
Communist Propaganda. This was because the state forum was inadequate to hear whether
the statute was permissible since the statute was overly broad and vague. The Court was
skeptical of whether the states criminal prosecution will assure adequate vindication of
constitutional rights. This, in Justice Brennans idea, was an exception to the general
premise that state criminal statutes must be challenged in state court. Even when the Court
strikes down a statute as being overbroad, the state gets the chance to narrow the construction
of the statute and can apply the newly construed statute in a narrow fashion as long as there
were no problems with adequate notice
b. The ct issued an injunction . The statutes were being misapplied. The Ct held that abstention
doctrine is inappropriate for cases like this where statutes are justifiably attacked on their fact
abridging free expression, or as applied for purpose of discouraging protected activities.
Q. Younger v. Harris
1. Some ppl are being prosecuted for being a progressive labor party (Syndicalism Act). 2 of the ppl
were not indicted (standing, ripeness issue), but Harris was indicted. Harris wants to enjoin the state
from applying the statute against him. Harris has standing /c hes being prosecuted.
2. Black says that theres no proof if Harris goes through state ct proceedings that he will be irreparably
harmed. Fed. cts should not enjoin state criminal proceedings (deferred b/c of comity & federalism).
3. The critique is that Younger undermines civil rights
4. But then theres this problemDombrowski, which was decided 6 yrs ago. The Ct highlights the facts
(bad-faith prosecutions) of Dombrowski & limits it to unusual situations (bad-faith or harassment).

71

Downloaded From OutlineDepot.com

Kannar Federal Courts, Spring 2011

a. Samuels v. Mackell (a district court cant grant a declaratory judgment since the same
equitable principles relevant to the propriety of an injunction must be taken into consideration
by federal district courts in determining whether to issue a declaratory judgment.)
b. Younger Abstention Only applies to Pending State Criminal Prosecutions
i.
In Steffel v. Thompson, a man was threatened with prosecution for distribution of
handbills, although the state never charged him. The man was able to bring this
declaratory judgment challenge to the statute in federal court since there was no pending
state criminal proceeding. The one man that was arrested & the other petitioner wanted
to bring a suit pursuant to 1983 claiming officers infringed on their 1st A rights. The
demonstrator satisfies the justiciablility requirements. The petitioner wanted a
declaratory judgment under 1st & 14th A.
ii. Justice Brennan (majority): says that this is NOT Younger (ongoing state proceedings).
Here, petitioner was not charged, there was no state proceeding; therefore, considerations
of equity, comity & federalism have little vitality.
iii. The Ct holds that even though theres no state proceeding, declaratory judgment is NOT
precluded. This is why the declaratory judgment act was enacted.
iv. 1105regardless of whether injunctive relief may be appropriate, fed. declaratory relief
is not precluded when no state prosecution is pending & a fed P demonstrates a genuine
threat of enforcement of a disputed state criminal statute, whether an attack is made on
the constitutionality of the statute on its face or as applied.
c. Younger abstention was applied to a non-criminal nuisance proceeding in Huffman v. Pursue
(1113). Justice Rehnquist wrote that a public nuisance case was very similar to a criminal
proceeding (even though there was no habeas review in a nuisance case). Justice Brennan
dissented again, arguing against Younger extension to non-criminal cases. Its a state CIVIL
proceeding & Younger applies to civil proceedings.
d. Hicks v. Miranda: seize the theater & ppl sue the officials to get their movies. Like Steffel (no
criminal charges when ppl file fed charges against officials). A 3 judge ct convenes. The
criminal complaint is amended & ppl are added as Ds in state criminal proceedings. The Ct
held where state criminal proceedings have begun against federal Ps after fed complaint was
filed but before any proceedings of substance on the merits have taken place in the fed ct, the
principles of Younger should apply in full force. Justice Stewart dissents: this does not
eliminate the race to the ct house, it allows the state to take a short-cut& make it to the finish
line 1st.
i. The Hicks rule was extended in Doran v. Salem Inn (1118), where the Court held that the
plaintiffs quickness to the federal court was irrelevant since the federal court proceeding
was still in an embryonic stage and no contested matter had been decided. In Doran,
the Court did affirm the practice of a district court granting a preliminary injunction (an
injunction where the plaintiff will suffer irreparable injury and is likely to prevail on the
merits.
ii. A suit is still pending in state court even if the final judgment of the trial court was
entered, but the state appellate proceedings were not complete. Huffman v. Pursue
(1121). Thus, there must be an exhaustion of state remedies before you can get to federal
court. It begins to cross the linethe line b/w civil & criminal is not as clear.
iii. However, if you go to federal court to seek injunctive relief against future state
prosecutions, you can seemingly bring a constitutional challenge to the statute. See
Wooley v. Maynard. There, a man challenged the constitutionality of a license plate
statute and sought to only enjoin future criminal enforcement, not his past prosecution.
5. Younger Abstention Also Applies in Some Civil Proceedings: Federal Courts will abstain in
enforcement proceedings (i.e., where the federal plaintiff is pushed into a state defensive posture
a. In Trainor v. Hernandez, the Supreme Court held that the Younger doctrine applies to both
criminal and quasi-criminal proceedings. Here, the Illinois Department of Public Aid brought
a civil enforcement action against the Hernandez family seeking return of fraudulently
concealed assets. Though this was not a criminal prosecution, the state defendant could not
avail himself of federal declaratory judgment relief. Justice Brennans dissent argued that
there should be no Younger abstention in non-criminal proceedings. Furthermore, he argued

72

Downloaded From OutlineDepot.com

Kannar Federal Courts, Spring 2011

that this was a flagrant and patent violation of express constitutional prohibitions, such that it
should qualify as a Younger abstention.
6. Cases where the state is not a party
a. Pennzoil (1124)Younger is applied b/c its a contempt issue
7. In certain kinds of administrative proceedings (attorney discipline, etc) neither are cts, but grows out of
Pennzoil & involves state adjudicative functions.
8. NOPSI casethere was a backing off some of the later stuff (trying to back off wholesale abstention if
the state is a party).

FEDERAL COMMON LAW & IMPLIED RIGHTS OF


ACTION/SUITS AGAINST FEDERAL OFFICIALS
A.

Presumption Against Federal Common Law: There has been a strong presumption

against the federal courts fashioning common law to decide cases. The Rules of Decision Act,
part of the Judiciary Act of 1789 and which remains largely unchanged today, by its very terms
seems to deny the existence of federal common laws and indicates that in the absence of positive
federal law, federal courts must apply state law.
1. In overruling Swift, the Court in Erie flatly declared that there is no federal general common law.
B. Limited Scope of Federal Common Law: Despite these declarations, federal courts
have fashioned common law in limited circumstances.
1. Can be Overruled by Federal Statute: Almost all federal common law can be overruled by
federal statute. Federal common law is judge-made, developed in the absence of clear
legislative command, and is generally non-constitutional in nature.
i) But, when common law is inferred from the Constitution, it cannot be overruled by
federal statute See Bivens v. Six Unknown Named Federal Narcotics Agents
2. Developed out of Necessity: Federal common law has developed out of necessity. In some
instances, there are gaps in the law; the application of statutory and constitutional provision
often requires the development of legal rules.
C. Federal Common Law to Protect Federal Interests: In many instances, federal
common law has been created to protect the federal governments interests. Two-part inquiry in
deciding whether to create federal law to safeguard federal interests
1. First, the Court considers whether a federal interest JUSTIFIES creating federal common
law.
i) No clear criteria exist to guide this determination. The Court often looks to whether the
underlying purpose of a constitutional or statutory provision warrants the development of
federal common law.
a. Clearfield Trust Co. v. United States (1943) (uniformity interests of regulating U.S.
currency warrants development of federal common law): The question was whether the
federal governments delay in notifying Clearfield Trust of the theft and forgery of the first
check (issued by the govt) precluded the government from recovering. The lower court
applied PA law and ruled against the U.S. Held: The Supreme Court held that the federal
court should have fashioned federal common law. All the rights and duties of the United
States concerning commercial power which it issues are governed by federal law in no way
dependant on the laws of PA or any other state. Uniformity interests require federal law, and
thus in absence of an applicable Act of Congress, it is for the federal courts to fashion the
governing rule of law according to their own standards.

2. Second, if federal law is developed, the Court decides its CONTENT; specifically, the Court
determines whether to copy existing state law principles or to formulate new rules.
i) The court can base the federal law on already existing state law principles i.e.
incorporating or borrowing state law as the federal rule of decision or it can create a
new legal rule to serve the federal governments interests.

73

Downloaded From OutlineDepot.com

Kannar Federal Courts, Spring 2011

D.

Balancing Test: In deciding whether to incorporate state law or to fashion new federal law,
the Court balances the need for federal uniformity (whether state law would frustrate objectives
of federal program) and/or a need for special rules to protect federal interests against the
disruption that will come from creating new legal rules.
1. Kimbell Foods (1979) (no need for uniformity thus state law could be adopted): The issue in the case
was whether the federal governments liens take priority over private liens when the federal
government seeks recovery on defaulted federal loans. Because there was no need of uniformity, the
prudent course was to adopt the readymade body of state law as the federal rule of decision until
Congress strikes a different accommodation.

2. Federal Law for Tort Claims Involving the Federal Government: In tort cases, the Court
has developed federal common law to protect the interests of the United States.
a. United States v. Standard Oil (1947) (developed federal common law to preclude state tort
law): After a Standard Oil truck injured a solder, the government sued the company. The
issue came down in final consequence to a question of federal fiscal policy. Whatever the
merits of the policy, its conversion into law is a proper subject not for the court but rather for
Congress.

3. Federal Common Law in Cases Involving Private Parties: In some instances, the Court
has been wiling to develop a federal common law defense in suits between private parties in
order to protect federal interests.
i) Boyle v. United Technologies Corp. (1988) (created common law shielding contractors from
liability to protect federal interest in obtaining equipment for the military (military contractor
defense)): The issue was whether a contractor could be held liable under state tort law for injuries
caused by design defects in products supplied to the military. Held: There is a uniquely federal
interest in obtaining equipment for the military and that the application of state tort law would
impair this federal activity. Thus, the Court chose to fashion federal common law and declared
that a contractor is not liable where the United States approved precise specifications for the
equipment, the equipment met those specifications, and the supplier warned the United States bout
the dangers in the use of the equipment known to supplier but not to the United States.

4. Will preclude state law if following federal law forces person to break state law (WDAY): Radio station sued for libel after political candidate responded to opponent on air (station was
required by federal law to carry the response). Court said it would be improper to hold them libel
under state law for conduct that was required by federal law, itd be too perverse.

5. Foreign Relations: Federal common law can govern questions of foreign relations when
there is no applicable federal statute. It is obvious that the need for a single federal rule is
particularly great when the foreign relations of the United States are at stake.
i) Banco Nacional de Cuba v. Sabbatino (1964) (act of state doctrine decided by federal common
law): An assignee of contract rights held by the Cuban government sued on the contract in the
United States. The Supreme Court held that the contours of the act of state doctrine, which
governed the case, were to be determined by federal common law.

6. Other Interests: Federal common law has also been formulated to protect other interest such
as when the United States is a party to the lawsuit (see Clearfield Trust), interstate disputes
(e.g. allocation of water or interstate pollution), regulation of banking, and admiralty.
E. Federal Common Law Inferred from Federal Statutory Provisions: If
necessary to effectuate the intent behind a federal statute.
1. Lawmaking Authority Implied From a Statutory Grant of Jurisdiction: Congress
sometimes intends the federal courts to develop a body of federal common law rules under
grants of jurisdiction. In such a case, Congress will provide a broad jurisdictional grant with
the expectation that the federal judiciary will develop specific standards to effectuate the
purpose of the statute.
i) Textile Workers Union v. Lincoln Mills of America (1957) (federal substantive law developed
from broad statutory grant under Taft-Hartley Act): An employer sued a union for an injunction
to enforce an arbitration agreement. Taft-Hartley Act grants the federal courts jurisdiction to

74

Downloaded From OutlineDepot.com

Kannar Federal Courts, Spring 2011

decide disputes under labor-management contracts in industries that affect interstate commerce but
didnt enact any substantive principles for the federal courts to use in deciding such cases. Held:
Federal jurisdiction was upheld on the grounds that Congress intended for the federal courts to
develop a body of common law principles to resolve labor-management disputes.

2. Other Statutes Interpreted as Authorizing Federal Common Law: The Supreme Court
has also approved the development of federal common law under other various statutes.
However, the federal judiciary will formulate a body of common law rules only pursuant to
clear congressional intent for such action (see Texas Industries v. Radcliff Materials).
i) E.g. Employee Retirement and Income Security Act (ERISA); Sherman Antitrust Act; and the
Alien Tort Claims Act.

FEDERAL COMMON LAW RIGHTS OF ACTION


A.

Implied Private Rights of Action under Statutes: Although it is generally

reluctant to do so, the Court has created a limited number substantive causes of action in the
absence of express statutory authorization. The Court has only created such private rights of
action, though, where was necessary to effectuate Congresss intent.
1. Reasoning for Reluctance: Both separation of powers and federalism concerns. Separations
of powers is implicated because of the Courts conviction that the legislature, not the
judiciary, should authorize recovery. The absence of federal legislation also means that the
conduct is controlled by state law; the creation of federal common law means that the states
no longer have exclusive authority in the area.
B. Current Affirmative Creation Test Alexander v. Sandoval Approach: Under its
new, more restrictive approach, the Court asks whether Congress affirmatively created a private
right of action under a statute. The Court will PRESUME that NO SUCH PRIVATE RIGHT
exists unless there is evidence to suggest to the contrary. Two propositions:
1. A private right of action must be based on LEGISLATIVE INTENT, and legal context
matters only to the extent that it CLARIFIES THE TEXT of the statute
2. The EXPRESS PROVISION of one method of ENFORCING a substantive rule suggests
that Congress intended to PRECLUDE OTHER METHODS, such as an implied private
right of action.
i) Alexander v. Sandoval (2001) (no private right of action where alternative method of
enforcement provided in the statute): Concerned whether private individuals may sue to enforce
disparate-impact regulations promulgated under Title VI of the Civil Rights Act of 1964. The
plaintiff brought a class action to enjoin the Alabama Dept. of Public Safetys policy adopted
after an amendment to the AL Constitution declared English the official language of the state
of administering drivers license examinations only in English. Held: The Court found no private
right of action. The Court said that neither as originally enacted nor as later amended does Title
VI display an intent to create a freestanding private right of action to enforce regulations
promulgated pursuant to it. Furthermore, the statute provided methods for its enforcement, such
as fund cut-offs for noncompliance. As a result, the express provision of one method of enforcing
a substantive rule suggests that Congress intended to preclude other.

C. Cort v. Ash Denial Test This is No Longer really Used: Traditionally, the Court relied on a
four-part test for determining whether private rights of action should be created under federal
statutes. Under this approach, it was not necessary to show an intention to create a private
cause of action, although an explicit purpose to deny such a cause of action would be
controlling.
1. Test: In determining whether a private remedy is implicit in a statute not expressly providing
one, the Court would ask...

75

Downloaded From OutlineDepot.com

Kannar Federal Courts, Spring 2011

i)

First, the threshold question is whether the statute was enacted for the benefit of a special class of
which the plaintiff is a member. That question is answered by looking to the language of the
statute itself.
ii) Second, the analysis requires consideration of legislative history. Legislative history of a statute
that does not expressly create or deny a private remedy will typically be equally silent or
ambiguous on the question. Therefore, in a situation in which it is clear that federal law has
granted a class of persons certain rights, it is not necessary to show an intention to create a private
cause of action, although an explicit purpose to deny such a cause of action would be controlling.
iii) Third, a private remedy should not be implied if it would frustrate the underlying purpose of the
legislative scheme. However, when that remedy is necessary or at least helpful to the
accomplishment of the statutory purpose, the Court is decidedly receptive to its implication under
the statute.
iv) Fourth, the final inquiry is whether implying a federal remedy is inappropriate because the subject
matter involves an area basically of concern to the states.

BIVENS: SUITS AGAINST FEDERAL OFFICERS FOR


CONSTITUTIONAL VIOLATIONS
A.

Bivens: Implied Rights of Action Under the Constitution and Suits


against Federal Officers: No federal statute authorizes federal courts to hear suits or give

relief against federal officers who violate the Constitution of the United States, as there is no
analogous statute to 42 U.S.C. 1983 pertaining to violations of federal law by federal officials.
B. Inferred from Constitution: Supreme Court held that it would infer a cause of action
for damages directly from constitutional provisions.
C. Injunctive/Exclusionary Relief Only in Past: Prior to the Courts decision in
Bivens, although courts protected constitutional rights through injunctive relief and doctrines such
as the exclusionary rule, plaintiffs were not allowed to sue federal officers for monetary remedies
in federal court. Plaintiffs seeking such compensation were relegated to state tort law causes of
action.
i) Const generally does not expressly create remedy
a. Remedies under Constitution: Takings & Habeas
D. Monetary Relief Now Available via Bivens Claim: Money Damages available
when NO OTHER FEDERAL REMEDY is provided for the vindication of a Constitutional
right.
1. Bivens (1971) The Court determined there must be a remedy for every wrong, and laid down the rule
that it would imply a private right of action for monetary damages where no other federal remedy is
provided for the vindication of a Constitutional right. The Court stated that the presumption is that
where there is a violation of a constitutional right, the plaintiff can recover whatever he could recover
under any civil action, unless Congress has specifically curtailed that right of recovery.

E.

Exceptions to Bivens: NO Bivens cause of action if


1. There are SPECIAL FACTORS counseling hesitation and precluding Bivens remedies, or
i) Suits from Military Service: the Supreme Court has refused to permit Bivens suits
arising from military service. The Courts reasoning for this is based on the rationale that
there is a need to preserve the military hierarchy
i.

Chappell v. Wallace (1983: Discriminatory practices by superior officers. The special


nature of the military was a factor counseling hesitation against Bivens claims.
ii. United States v. Stanley (1987) Given LSD without his knowledge in an Army
experiment. Bivens remedy is not available for injures that arise out of military service.

76

Downloaded From OutlineDepot.com

Kannar Federal Courts, Spring 2011

2. Congress has specified an ALTERNATIVE REMEDY that court believes provides an


EQUALLY EFFECTIVE SUBSTITUTE.
i) Bush v. Lucas (1983) (existence of comprehensive civil service remedies precluded Bivens
claim): An aerospace engineer employed by NASA claimed that he was demoted because of his
public statements, which were critical of the agency. No Bivens claim allowed. The Court found
that the existence of comprehensive civil service remedies prevented Bush from brining a cause of
action directly under the First Amendment. Congress could indicate its intent to prevent judicial
remedies by providing a statutory remedy itself. Here, the separate remedy was adequate.
ii) Schweiker v. Chilicky (1988) (social security system remedies precluded Bivens claim): The
Court against refused to permit a Bivens action for alleged due process violation stressing that the
Social Security Act provided an elaborate administrative and judicial remedy.
iii) BUT see, Carlson (next pg), Bivens is a counterpart to remedies under FTCA.

F.

Immunity: Even if a cause of action is recognized, the defendant can still raise immunity as a

defense. The rules for immunities of federal officials are identical to those that have been
recognized for state and local officials under 1983.
G. Criticism Of Bivens: Legislative action is required before suits for money damages can be
brought in federal court for violations of constitutional rights because state law tort remedies
exist, and Congress must act if federal law is to provide an independent remedy. Separation of
powers is violated when the Court replicates through judicial action what 1983 provides against
state and local officers.
1. Rebuttal: Judicial role is to provide a remedy of violation of rights. Courts traditionally have
fashioned remedies in the absence of legislative action, including the exclusionary rule and
damage remedies under federal statutes. Protection of federal rights should not depend on the
varies of state law, and rather, federal court must safeguard and enforce constitutional rights.
H. Amendments Protected by Bivens: In Bivens cause of action for damages against
federal officers only for violation of the Fourth Amendment,
1. Now: SCOTUS recognizes cause of action for First, Fourth, Fifth, and Eighth Amendments.
i) Lower federal courts have recognized Bivens suits for violations of the Ninth and
Fourteenth Amendments as well.
I. Bivens may still be available where certain statutes exclude liability :
1. In Davis, Congress exempted its members from federal employment discrimination
legislation .
i) Bivens was still allowed b/c the Court narrowly construed the exemption as solely
removing them from liability under Title VIII and not precluding all suits for employment
discrimination. Davis v. Passman (1979) (narrow interpretation of immunity in statute): A
congressman fired his administrative assistance because he wanted the position to be filled by a
male. The Court concluded that Congress did not mean to foreclose other remedies not included
in Title VII. Federal officers could be sued for money damages for violations of the Fifth
Amendment.

2. Bivens is a Counterpart to Alternative Remedies under FTCA only (limited elsewhere):


The Court held that Bivens suits (recovery from officers) were a counterpart of the
alternative remedy of the Federal Torts Claim Act (FTCA) allowing recovery from the
government and thus Congress did not intend the federal statute to preempt the possibility
of Bivens suits.
i) Carlson v. Green (1980) (possibility of recovery under FTCA did not preclude recovery under
Bivens as well) : A mother sued federal prison officials on behalf of her deceased son, claiming
that he was the victim of gross inadequacies of medical facilities and staff, which caused his death
and constituted cruel and unusual punishment. A remedy was available under the FTCA, but no
indication that Congress intended for the FTCA to preempt Bivens suits b/c one was directed at
governmental liability and the other was directed at individual liability. Furthermore, remedies

77

Downloaded From OutlineDepot.com

Kannar Federal Courts, Spring 2011

available under the FTCA were not as effective as a Bivens suit, for damages against individual
officers would serve as a more effective deterrent to constitutional violations.

J.

Recent Limitation of Bivens Claims by the Supreme Court: In the last two
decades, however, the Supreme Court has consistently refused to expand, and indeed has
substantially limited, the availability of Bivens suits.
1. No Bivens Claim Allowed if Alternative Remedies: Congress can indicate its intent to
prevent judicial remedies by statutory language, clear legislative history, or even providing
the statutory remedy itself. Where such a remedy is already provided, no Bivens claims are
permissible.
2. No Bivens Suits Against Federal Agencies: In FDIC v. Meyer (1994), the Court held that the
Bivens remedy is available only against government officials, not against government
agencies. The Court rationalized this decision by focusing on the deterrence rationale for
Bivens actions. If litigants could bypass officers and sue federal agencies directly, the
deterrent effects of the Bivens remedy would be lost.
3. Eleventh Amendment Bars Bivens Suits Against State Governments: Generally, Bivens
suits against state governments are not allowed because the Eleventh Amendment precludes
states from being sued in federal court. The only exceptions would be consent or a suit
brought under 5 of the Fourteenth Amendment.
4. Bivens Suits Against Local Governments: Because Monells limitation on respondeat
superior liability is based on the Courts understanding of 1983s legislative history, no
inherent reason exists for precluding Bivens liability in suits brought directly under the
Constitution.
5. No Bivens Suits Against Private Entities: The Court has held that a private entity which
operates a prison cannot be sued in a Bivens action. This is because state tort law typically
provides adequate remedies.
i) Correctional Services Corp. v. Malesko (2001) (no Bivens claims against private company
contracted to provided services for the federal government) Private company operating a halfway house under a contract with the Federal Bureau of Prisons. An inmate suffered a heart attack
b/c facility refusal to allow him to use an elevator despite a serious heart condition. Held: Private
companies may not be sued under Bivens. Bivens suits are available only against individual
federal officers, not against government or private entities. Only twice has the Court expanded
Bivens holding: once in Davis (when no other remedy was available) and once in Carlson (when
no other remedy against the defendant officials was available). Here, the case is neither, and there
are alternative remedies including state tort law.

6. Floodgates/difficulty in defining COA


i) Wilkie v. Robbins (2007): Chain of events where govt harassed P to get easement. Held:
No Bivens available. First looked at alt remedies: Patchwork of alternative remedies.
Then looked at SFCH: difficulty in defining a workable COA. Would open floodgates.
Efforts to induce Robbins to grant easement were perfectly legitimate. But maybe they
could use this COA as a defense in a suit?
7. Iqbal: selectively confining certain prisoners to harsher conditions after 9/11. Not part of
actual ruling, but court said it would hesitate to find a Bivens First Amendment free exercise
claim if it had occasion to reach that claim. But it was decided on pleading grounds. No
supervisory liability anymore under Bivens. Q to class: what business did the court have in
addressing these Qs at all? Iqbal sues a bunch of high level federal defendants, lower courts
say the claim can go forward. Cant be supervisory liability for the chief. Only liable for his
or her misconduct.

SOVEREGN IMMUNITY AND 1983 CLAIMS:

78

Downloaded From OutlineDepot.com

Kannar Federal Courts, Spring 2011

Damages

State; State State official Local gov't


agency; Arm in official
of State
capacity

Local official Fed, state, or


in official
local official in
capacity
personal capacity

Sovereign
immunity

Same as for
Absolute
damages claim immunity if
vs. local gov't available.
Otherwise,
qualified
immunity.

None,
Sovereign
immunity

To state claim,
must satisfy
Monell, etc.
No qualified
immunity. No
sovereign
immunity (unless
arm of state).

Prospective Sovereign
Relief
immunity

Ex parte
Young.

Available unless it's If arm of state, No qualified


an arm of state.
use ex parte
immunity.
Young.
Re: judges,
Re: judges, see
see 1983.
Re: judges, see 1983.
1983.
Absolute
Absolute
immunity for
Absolute
immunity for
legislators
immunity for legislators.
legislators.

FEDERAL HABEAS CORPUS & THE WAR ON


TERROR
A. Habeas Corpus Generally: Habeas corpus is provided for in Article I, 9, cl.2 of the Federal
Constitution. In addition, the writ was also provided in the First Judiciary Act, allowing the
justices of the Supreme Court as well as judges of district courts the power to grant writs;
however this only applied to prisoners in federal rather than state custody. Federal prisoners
apply under 2255 (so that can be suspended?)
U.S. Citizen = Normal Habeas Statute Applies
Foreign citizen in U.S. Territory = Can Avail of Habeas Statute, see Rasul
Enemy Combatant in U.S. Territory = Habeas (Boumediene)
B. Cant hold U.S. Citizen deemed enemy combatant indefinitely w/o DP: : The Executive
Branch does not have the power to hold a U.S. citizen indefinitely even if deemed an enemy
combatant on American soil without basic due process protections enforceable through judicial
review.

79

Downloaded From OutlineDepot.com

Kannar Federal Courts, Spring 2011

1. Meaningful Opportunity to Contest Factual Basis for Detention: Plurality of SCOTUS


says Fifth Amendment due process guarantees that an American citizen held in the United
States as an enemy combatant be given a meaningful opportunity to contest the factual basis
for that detention before a neutral decisionmaker.
i) Hamdi v. Rumsfeld (2004) Hamdi, an American citizen, was arrested by the U.S. military in
Afghanistan, declared an "enemy combatant," and transferred to a military prison in Virginia. He
wanted to show that there wasnt a factual basis for his enemy combatant status. Under the
Mathews v. Eldridge balancing test, Congresss authorization of military force authorizes holding
Hamdi, but citizen detainee gets limited due process including notice of the charges, opportunity
to be heard, and some sort of legal counsel. However, the Court did not require normal procedural
protections such as placing burdens on the government or a ban on hearsay evidence. So some
due process, but not as much as in a usual civil proceeding. Might have to permit hearsay to allow
govt to show hes an enemy combatant, just let prisoner challenge the allegations. Just want to
make sure tourists etc dont get swept up as Enemy Combatants.
a. In dissent, Scalia thought Milligan meant he should be tried in State Criminal court. But then
Ex-Parte Quirin (military tribunal tried/ executed US citizen as unlawful combatant). Scalia
said that wasnt courts finest hour, distinguished it b/c they didnt dispute that they were
enemy combatants.

C. Non-Citizen Detainees have same rights: Although by its terms, the holding was limited to
citizen-detainees, the last sentence of the plurality relies on the Geneva Convention and states
that habeas corpus should be available to an alleged enemy combatant. Based on that language
and the Courts holding in Rasul that U.S. courts have jurisdiction to hear habeas petitions filed
by non-citizen Guantanamo detainees there are likely some limited due process rights owed to
non-citizens which can be fulfilled by habeas or other federal statute.
1. Textual Support in Habeas Statute: 2241 does not distinguish citizens from aliens,
there is very little reason to think that Congress intended the geographical coverage of the
statute to vary depending on the detainees citizenship (see Rasul).
2. Argument for Suspension of Writ: In an opinion by Scalia joined by Stevens, the justices
asserted that the government had only two options to detain Hamdi: either Congress must
suspend the right to habeas corpus (a power provided for under the Constitution only during
times of invasion or rebellion), which hadnt happened; or Hamdi must be tried under normal
criminal laws entitled to other U.S. Citizens.
D. Military Commissions Act of 2006 Ruled Unconstitutional Suspension of the Writ: MCA
now prevented detainees in Guantanamo Bay from challenging their detention in U.S. courts.
The DC Cir thought that there was no constitutional problem because the petitioners have no
constitutional rights being aliens held outside of the United States. Overruled by Boumediene.
1. Military Commissions Act of 2006 (MCA) 7: "(e)(1) No court . . . shall have jx to hear . . . an
appl for . . . habeas corpus filed by. . . an alien detained by the US who has been determined by the US
to have been properly detained as an EC or is awaiting such determination. "(2) Except as provided
in Ps (2) & (3) of section 1005(e) of the [DTA], no court . . . shall have jx to hear . . . any other action
against the US or its agents [wrt] any aspect of the detention, transfer, tx, trial, or conds of confinement
of an alien who is or was detained by the US & has been determined by the US to have been properly
detained as an enemy combatant or is awaiting such determination."

E. Boumediene v. Bush (2008): Aliens held at Guantanemo who have been designated Enemy
Combatants by CSRT hearings, are entitled to habeas review. MCA 7, modified habeas statute,
stripped jurisdiction & review of CSRT, so only review would be limited review in DC Cir.
Court ruled that is an Unconstitutional suspension of the writ (Suspension Clause). Cong did not
provide adequate alternative in DTA/MCA. . Also, there has been enough delay here to allow for
review. US has de facto sovereignty over guantanamo. Q of adequacy of procedure and nature of
CSRT proc look @ history of habeas (always had it), de facto sov in Guant, adversarial process
didnt really exist here (no advocate, limited ability to rebut/enter later-discovered evidence or be
present, only a few claims could be raised). Dont really say how much review must be provided

80

Downloaded From OutlineDepot.com

Kannar Federal Courts, Spring 2011

balance btwn original process and subseq review that follows (habeas can be narrowed if initial
review is more thorough)
1. Separation of Powers/Susp. CL: Under DTA DC Cir could decide whether CSRT
determinations were correct procedurally, review questions of law to extent laws/constitution
were applicable to CSRT proceedings.
i) Dont want Congress/President to say what the law is and determine courts jurisdiction
by being able to move bases around strategically in order to avoid U.S. law.(shouldnt
allow legislature/executive to switch the right on and off) Suspension Cl has a separation
of powers fn to protect people from detention by exec.
2. Talked about history of habeas. At minimum, Sup Cl provides floor that protects writ as it
existed in 1789 (dont answer Q whether Sup Cl has expanded along with Habeas over time)
i) DCA violated the minimum b/c there has been no trial, and yet the executive is confining
these people.
ii) Alienage didnt bar habeas relief in the past. Also, havent seen situations like
Guantanemo where US has pseudo sovereignty, but Cuba maintains ultimate sovereignty.
Had to take functional approach, not formalistic that would depend on who has
sovereignty.
iii) Particular type of evidence that wouldnt be available to detainee under DTA: evidence
that wasnt available at CSRT hearing, but was found later. That makes the statute way
too limited.
a. To be allowable, relief needed would: opportunity for release if the govt cant
establish that CSRT was rendered in compliance with applicable legal standards.
o Nature of CSRT procedure: actual procedure is lacking, detainee cant challenge
evidence, doesnt get adequate counsel.
Due process reqs fair notice, opp to dispute charges, neutral decision maker,
few others, but things like hearsay might be ok (from Hamdi, need to help
out govt a little)
Hamdi left 2231 intact, didnt concern suspension clause, here Congress has
actually suspended the writ.
Facts on the ground, that same flexible review they required in Hamdi might
not apply anymore, 2 justices switched sides b/c detainees might not actually
be getting proper review on the ground, so they might have to give more
prophylactic protections.
Even if the review is broadened, there would still be instances where it
wouldnt be enough (later exculpatory evidence) This is the big reason they
overturned the statute.
So, statute could be valid in some applications (like where there was no after
discovered evidence), so why not say let those go, and when there is new
evidence, allow habeas. Court says that would add another layer of
complexity. Easier to just invalidate the jurisdiction stripping and say the
CSRT procedures are inadequate.
o Chief Justice says that they jumped the gun, didnt have to invalidate everything.
Maybe Congress wanted to centralize review in DC Cir. May need special
accommodations for classified info.
o Scalias dissent: habeas review never for aliens held abroad. Majority takes
separation of powers into effect when evaluating suspension clause (sep of powers
requires some kind of habeas review, even tho never find anything exactly on point,
its a structural Q and courts need to protect liberty). Scalia says shouldnt use sep or
powers to expand scope of suspension clause, only applies to specific powers, should
extrapolate to lots of other issues.

81

Downloaded From OutlineDepot.com

Kannar Federal Courts, Spring 2011

o
o
o

Thinks this will lead more Americans being killed.


How does this relate to Alden (state sovereign immunity): states wouldnt have
ratified Const if they didnt get sovereign immunity, probably didnt care about extraterritorial habeas review for aliens.
They didnt say habeas expanded over time, just that at its core, Sup Cl was all about
protection from detention by executive (sep of powers)
Detainee cases are an extended view of the courts view on the judiciarys role.
Congress essentially forced the courts to decide whether habeas reached aliens held
at Guantanamo, and court says yes.

Suspension Cl raises questions about whether fed habeas jx is required and whether it limits
congs auth to withdraw habeas jx Ex parte Bollman (1807): habeas is not inherent but must be
conferred by statute.
INS v. St. Cyr (2001): Suspension Cl restricts congs power to preclude review of the legality of
federal executive detentions at abs minimum, Suspension Cl protects writ as it existed in 1789
Boumediene v. Bush (2008): congressional preclusion of review invalid b/c it violated the right to
habeas (or an adequate substitute) guaranteed by the Suspension Cl.
F. Territorial Jurisdiction in Habeas: 2241(a) vests authority to grant the writ in the Supreme
Court and district courts, but only within their respective jurisdictions.
1. Custodian Jurisdiction Rule (Issuing Court must have jurisdiction over custodian): The
Court now only requires that the issuing court have jurisdiction over the custodian (used to be
over petitioner)
i) Johnson v. Eisentrager (1950): Held that district courts lack jurisdiction to issue a writ of habeas
corpus on the applications of foreign citizens held abroad. (foreign war criminals held abroad)
Unclear if decided on the merits or saying court didnt have jx.
ii) Braden v. 30th Judicial Cir. Ct. (1973) (established custodial jurisdiction rule): recognized the
territorial jurisdiction of a district court to entertain a petition from a prisoner physically confined
in another state. The Supreme Court concluded that 2241(a) requires only that the court have
jurisdiction over the custodian.

2. If Petitioner is Located at Military Base in a Foreign Country: Normal Habeas if U. S.


exercises complete control over a military brig located on foreign soil, it is irrelevant that
the ultimate sovereignty remained foreign.
a. Rasul v. Bush (2004) (non-citizen aliens detained at the U.S. Naval Base at Guantanamo
Bay, Cuba, may avail themselves at the time of the writ of habeas corpus) The Court
found that the degree of control exercised by the United States over the Guantanamo Bay base
was sufficient to trigger the application of habeas corpus rights. Further, the Court wrote that
the right to habeas corpus is not dependent on citizenship status.

G. The Proper Respondent: Habeas can only be filed against the person directly responsible for a
prisoner's confinement or, put another way, the person with the power to bring the prisoner to
court. Usually the warden of the prison where the petitioner is incarcerated.
1. Rumsfeld v. Padilla (2004) (proper respondent is warden or brig commander where being held):
Lawyer filed a habeas petition against President Bush, Secretary Rumsfeld, Melanie Marr, Commander
of the Naval Brig where Padilla was being held in S.C. The proper respondent was the commander of
the military brig in which Padilla was held. Case had to be refilled
H. US citizen can file against any high ranking military official (proper respondent rule doesnt apply)?
I. Immediate custodian rule is like venue/personal jurisdiction, so it can be waived. Also exceptions can be
made (dont want govt to just move prisoners around to avoid jurisdiction)

82

Downloaded From OutlineDepot.com

Kannar Federal Courts, Spring 2011

HABEAS CORPUS & STATE PRISONERS


GENERALLY
A.

Habeas Corpus Generally:

B.

Types of Claims in Habeas That Were Already Raised in State Court

The federal court may order the release of a state prisoner


who is held by the state in violation of the Constitution or Federal law.
1. U.S. Const. Art. I, Section 9: "The Privilege of the Writ of Habeas Corpus shall not be
suspended, unless when in Cases of Rebellion or invasion the public Safety may require it."
2. (Collateral Review): Separate Civil Lawsuit; Not Direct Review: Technically, federal
court consideration of the habeas corpus petition is not considered direct review of the state
court decision; rather, the petitioner constitutes a separate civil suit filed in federal court and
termed collateral relief.
3. Can be Utilized In Variety of Situations: Habeas is available whenever a person is in
government custody. In addition to challenging a criminal conviction, it may be used by to
challenge confinement in an institution, a deportation order, an extradition order, executive
detention, conviction in military court, or denial of parole.
1. Most Questions of Constitutional Law Can be Relitigated: Federal habeas courts can
relitigate questions of federal constitutional law that were fully and fairly litigated in state
court.
i) Examples: Proof of guilt beyond reasonable doubt, jury of peers (racial composition),
ineffective counsel, confessions admitted against Miranda.
2. Exception Fourth Amendment Exclusionary Rule Challenges NOT ALLOWED if Full
and Fair Hearing:
i) Claims that a state court improperly failed to exclude evidence as being the product of an
illegal search or seizure cannot be relitigated on habeas corpus if the state court provided
a full and fair hearing.
a. Stone v. Powell (1976) No habeas on the ground that the evidence obtained in an
unconstitutional search or seizure was introduced at his trial.
i. Not guilt related: Exclusionary rule claims do not relate to the accuracy of the factfinding process.
ii. Exclusionary rule exists to deter illegal police practices. This deterrence would only be
increased marginally if at all if allowed on habeas, while there would be substantial
costs for society in permitting guilty defendants to go free and in undermining respect for
criminal justice system.
1. But seeRose v. Mitchell (if racial discrim); Kimmelman v. Morrison (if given
no fair opportunity in state court)
3. No New Rules, See AEDPA and Teague

4. Habeas prisoners cannot complain about state law.


5. Guilt or Innocence Not Determinative: Traditionally, the availability of habeas review did
not depend upon a claim that the prisoner was innocent of the crime. Justice Brennan worried
in dissent that Stone would limit habeas corpus to matters relating to guilt or innocence.
Despite his fears, decisions since Stone have not adopted that rule, and habeas is not limited
to claims of innocence. However, true innocence without more is likely not sufficient
for habeas review either (see extraordinarily high threshold in Herrera).
i) Jackson v. VA d/p concerns (no reasonable juror would have found guilt BRD) allowed
innocence claim d/p violation can be const claim
ii) Herrera v. Collins (1993) (actual innocence does not guarantee habeas relief, need Const.
violation): The function of habeas review was to redress constitutional violations, not to correct
factual efforts. Review of freestanding innocence claims would disrupt the strong state interest in
finality.

83

Downloaded From OutlineDepot.com

Kannar Federal Courts, Spring 2011

a. But seeWe may assume, for the sake of argument, that in a capital case a truly persuasive
demonstration of actual innocence would warrant habeas relief if there were no state avenue
open to process such a claim. However, the petitioners showing in this case fell far short of
the extraordinarily high threshold for such an assumed right.

PREREQUISITES FOR FEDERAL HABEAS CORPUS


A.

The Custody Doctrine: Petitioners who wish to file federal petitioners pursuant to 2241
and 2254 must allege that they are in custody in violation of federal law.
1. Does not demand actual incarceration, but can be satisfied by constructive restraints on
liberty.
i) In criminal prosecutions, it is enough if petitioners are subject to bail or parole
conditions.
ii) If just a fine, no habeas claim allowed.
iii) Not Moot: If you file before getting released claiming that the conviction was erroneous,
the claim doesnt become moot..
a. But, if you are only attacking the sentence, claim become moots on release.

B.

The Exhaustion Doctrine: 28 U.S.C. 2254 (b)-(c): prisoners MUST EXHAUST ALL
STATE JUDICIAL OPPORTUNITIES to litigate federal claims before presenting those claims
to federal courts in habeas petitions. Rule rests on comity: respect for state court decisions by
fed. Jud.
1. Rationale: (1) to avoid federal interference with state processes and (2) to preserve the state
courts role in the making and enforcement of federal law.
2. Exhaustion Requirement: Prisoners must identify the substance of their federal claims for
consideration in state court and present the courts with the facts on which the claims may
exist.
i) A prisoner cannot present the state courts with one claim and then advance a different
claim in federal court.
ii) Nor is it enough merely to identify a legal claim in the abstract.
iii) Exception: Civil Rights Claims: Exhaustion under 1983 not required. Often a Q of
whether claim is properly a habeas or 1983 claim. But if complaining about prison
conditions, must exhaust administrative remedies.
3. Must Exhaust Direct AND Collateral Claims: Prisoners must also exhaust in any available
state post-conviction procedures aimed to correct fundamental errors not corrected at trial or
on direct review. This includes any and all discretionary avenues of review.
i) 2254(c): An applicant shall not be deemed to have exhausted his state remedies if he
has the right under the law of the state to raise, by an available procedure, the questions
present.
ii) OSullivan v. Boerckel (1999): The Court held that prisoners ordinarily must seek discretionary
review in the highest state court in order to satisfy the exhaustion doctrine and keep their claims
available for federal habeas.
iii) State Habeas Review Usually Not Required: Except when the federal claim could only be raised
on habeas in the state system (ineffective assistance of counsel, original counsel wouldnt raise it
at trial, will come up for the first time during habeas)

4. Claim Must have been Raised and Decided on the Merits: Pursuant to 28 U.S.C. 2264(a),
a federal court entertaining a habeas corpus petition from a prisoner on death row can
consider only claims that were previously raised and decided on the merits in state court.
5. No Ruling on Claim Required: A prisoner cannot force a state court to actually address the
merits of a claim. Thus, it is enough if the highest state court has a fair opportunity to reach

84

Downloaded From OutlineDepot.com

Kannar Federal Courts, Spring 2011

the merits, even if the court declines do so on procedural grounds or overlooks or disregards
the claim without explanation.
6. Dismiss Without Prejudice: When a petition fails to satisfy the exhaustion requirement, the
premature petition is usually dismissed without prejudice (or put on hold so to speak) in
order that a renewed writ, filed when no state court avenues for litigation remain open, may
be filed in the future.
7. Mixed Petitions Should Be Dismissed in Entirety: Prisoners sometimes file habeas
corpus petitions containing multiple claims, some of which are ready for federal adjudication
and some of which are not. These mixed petitions will be dismissed by the federal district
courts in their entirety.
i) Two Options Post-Dismissal: A prisoner who has submitted a mixed petition which was
dismissed in its entirety has two options:
a. PURSUE AVAILABLE STATE COURT LITIGATION opportunities for the
claims that are premature and then return to federal court when exhaustion doctrine is
satisfied with all claims.
b. ABANDON ANY CLAIMS that are NOT YET EXHAUSTED and proceed with
the claims the state courts have already had an opportunity to address.
ii) Alternative Habeas Court Ignores Failure and Denies Relief on the Merits:
Pursuant to 2254 (b)(2), a federal district court may ignore a prisoners failure to
exhaust, provided the court denies relief on the merits. This is sensible only when the
claims are so frivolous that there is no chance that a state court would sustain them i.e.
federal court should only leapfrog the exhaustion requirement and deny relief on the
merits when it is convinced that a claim has no merit and that it would be useless to
send prisoner to state court.
i. Allows Federal Court to Decide Meritorious Claims: (Judicial efficiency)
iii) Alternative Express Waiver by State: Under 2254(b)(3), a state may choose to
overlook a prisoners failure to satisfy the exhaustion doctrine, however this waiver must
be done expressly. States attorneys cannot forfeit or waive the exhaustion requirement
through inattention or failure to raise an objection.
iv) Alternative Habeas Statute Says No Exhaustion Doctrine for Death Row Inmates:
Under 2264(a), a district court entertaining an application from a death row prisoner
must determine at the threshold whether federal adjudication if foreclosed because a
claim was not presented in state court. If the court determines that a claim is not barred
because of procedural default, 2264(b) instructs the court to consider the claims
properly before it in light of paragraphs (a), (d) and (e) of 2254. Accordingly,
paragraphs (b)-(c) i.e. the exhaustion doctrine are not mentioned.
a. One available inference is that 2264 jettisons the exhaustion requirement in the
interest of speeding capital cases through the federal courts. If this is what 2264
means, district courts in capital cases controlled by 2264 cannot enforce the
exhaustion doctrine even if the respondent asks that the state courts be given the
chance to consider the prisoners claim.
C.

STATUTE OF LIMITATIONS ISSUES


1. Normal Filing Deadline: Under 2244(d)(1), a prisoner in state custody must be in
federal court within one year after the latest of four dates:
i) JUDGMENT BECAME FINAL by the conclusion of direct review or the expiration of
the time for seeking such review;
ii) IMPEDIMENT TO FILING an application created by State action in violation of the
Constitution or laws of the United States is removed, if the applicant was prevented from
filing by such State action;

85

Downloaded From OutlineDepot.com

Kannar Federal Courts, Spring 2011

iii) CONSTITUTIONAL RIGHT ASSERTED was INITIALLY RECOGNIZED by the


Supreme Court, if the right has been newly recognized by the Supreme Court and made
retroactively applicable to cases on collateral review; or
iv) FACTUAL PREDICATE of the claim or claims presented COULD HAVE BEEN
DISCOVERED through the exercise of due diligence.
2. Time Limit Tolled During State Post-Conviction Review (like state habeas): 2244(d)(2),
the time during which a properly filed application for State post-conviction or other collateral
review with respect to the pertinent judgment or claim is pending shall not be counted
toward any period of limitation
3. Death Penalty Cases: Under 2263(a), must be filed not later than 180 DAYS AFTER
FINAL STATE COURT AFFIRMANCE of the conviction and sentence on direct review or
the expiration of the time for seeking such review.
i) Tolling Rules for Death Penalty Cases: Under 2263(b), the time requirements
established by subsection (a) shall be tolled
a. From the date that a petition for CERTIORARI IS FILED IN THE SUPREME
COURT until the date of final disposition of the petition if a State prisoner files the
petition to secure review by the Supreme Court of the affirmance of a capital
sentence on direct review by the court of last resort of the State or other final State
court decision on direct review;
b. From the date on which the first PETITION FOR POST-CONVICTION
REVIEW or other collateral relief is filed until the final State court disposition of
such petition; and
c. During an ADDITIONAL PERIOD not to exceed 30 DAYS, if -- (A) a motion for
an extension of time is filed in the Federal district court that would have jurisdiction
over the case upon the filing of a habeas corpus application under section 2254; and
(B) a showing of good cause is made for the failure to file the habeas corpus
application within the time period established by this section.

PROCEDURAL DEFAULTS IN HABEAS CASES


A. Procedural Defaults Generally: If petitioner failed to present a federal defense in state court
because of a procedural default in that court, he will be precluded from asserting that defense on
federal habeas review.
1. Examples: failure to take timely appeal of state conviction, failure to make contemporaneous
objection to introduction of evidence, failure to timely challenge racial composition of jury.
2. Rationale: Enforces the exhaustion doctrine. If prisoners could routinely commit
procedural default in state court without suffering any consequences, they might do so
deliberately, thus satisfying the exhaustion requirement simply by eliminating any current
state court avenue for vindicating their federal claims.
i) Historical Approach The Deliberate-Bypass Standard: In Fay v. Noia, the Court
stated that federal habeas courts should decline to entertain claims that prisoners
intentionally withheld from state courts for strategic, tactical, or any other reasons that
could fairly be described as the deliberate by-passing of state procedures.
B. Modern Doctrine: Procedural default in state court forecloses federal habeas corpus,
1. Except in cases in which there is a good reason for excusing a failure to comply with state
procedural rules.
i) Wainwright v. Sykes (1977) (failure to make a timely objection under the state
contemporaneous objection rule bars federal habeas corpus review of the admitted inculpatory
evidence):

a. Rationale: The deliberate-bypass standard failed to accord sufficient respect to the


state interests served by contemporaneous objection rules, invited petitioners and
their lawyers to sandbag state courts, and generally treated state criminal trials as a

86

Downloaded From OutlineDepot.com

Kannar Federal Courts, Spring 2011

tryout on the road to federal habeas corpus rather than as the main event for the
adjudication of all issues pertaining to a criminal case.
C.

New Procedural Default Standard


1. There must be a state procedural rule requiring a prisoner to raise a federal claim in a
particular way or at a particular time.
2. The prisoner must have failed to comply with that rule, and for that reason, the state courts
must be unwilling to consider the claim.
3. The federal district court must determine whether the state courts procedural disposition of
the claim would constitute an adequate and independent state ground of decision that
would defeat jurisdiction in the supreme court if on direct review. If the state courts
procedural ground would cut off direct supreme court review, the district court typically will
refuse to entertain the claim.
4. BUT, a district court CAN consider a claim on the merits DESPITE an ADEQUATE
STATE PROCEDURAL GROUND for rejecting it if Cause & Prejudice, Actual Innocence,
or Novel Constitutional Claim
i) Cause & Prejudice: The prisoner shows both CAUSE for having failed to raise the
claim properly in state court AND ACTUAL PREJUDICE resulting from the default
OR
1.
Cause: Show that some objective factor EXTERNAL to the defense
impeded counsels efforts to comply with a states procedural rule. Thus, cause can
be shown
i. If, at the time counsel might have advanced a claim in state court, the factual or
legal basis for the claim was NOT REASONABLY AVAILABLE however,
defense attorneys have no cause for withholding a claim simply because the
existing precedents suggest that it is unlikely to succeed.
ii. If STATE AUTHORITIES INTERFERED with counsels ability to comply
with a rule, making compliance with the procedural rule impracticable;
iii. It is unlikely that counsel will say they are themselves ineffective, so Defendants
should be able to get relief for ineffective counsel: If counsels failure to follow
a procedural rule was so fundamentally incompetent and prejudicial as to
constitute INEFFECTIVE ASSISTANCE OF COUNSEL in violation of the
Fourteenth and Sixth Amendments however, this ineffectiveness must be at the
trial or appellate stages, as there is no constitutional right to counsel for prisoners
seeking state post-conviction relief. Will require filing state habeas first.
o Novelty and/or Futility Does Not Constitute Cause: The Court has held
that futility of raising an objection at trial (Engle v. Isaac (1982)) and the
novelty of a constitutional claim (Reed v. Ross (1984)) do not constitute
cause for the Sykes standard.
o Ineffective Assistance of Counsel is Cause: Although ineffective assistance
of counsel may serve as cause for the Sykes standard (Edwards v. Carpenter
(2000)), the right to counsel does not extend to postconviction proceedings
and a petitioner cannot claim constitutionally ineffective assistance in such
hearings (Coleman v. Thompson (1991)). Requires filing state habeas.
o External Impediment/Government Interference as Cause: Where the
governments withholding of information is purposeful for racist reasons
(Amadeo v. Zant (1988)) or inadvertent but the attorney relied on an official
policy (Strickler v. Green (1999)), this external impediment constitutes cause
for Sykes.

87

Downloaded From OutlineDepot.com

Kannar Federal Courts, Spring 2011

b. Prejudice: A prisoner must also show prejudice by proving that the federal error
complained of so infected the entire trial that the resulting conviction was
unconstitutional (United States v. Frady (1982)) (might not have been convicted)
ii) Actual Innocence: The prisoner demonstrates that the federal error that went uncorrected
in state court probably resulted in the CONVICTION OF ONE WHO IS ACTUALLY
INNOCENT. OR
a. ACTUAL INNOCENCE TEST: In extraordinary cases, where a constitutional
violation has probably resulted in the conviction of one who is actually innocent, a
federal court can address the merits of a claim even in the absence of a showing of
cause (Schlup v. Delo (1995); House v. Bell (2006)).
i. Standard: A prisoner who hopes to satisfy the probable innocence standard
must
1. Support his allegations of constitutional error with NEW RELIABLE
EVIDENCE whether it be exculpatory scientific evidence, trustworthy
eyewitness accounts, or critical physical evidence that was NOT
PRESENTED AT TRIAL. AND
2. On the basis of that new evidence, the prisoner must show that it is
MORE LIKELY THAN NOT that NO REASONABLE JUROR
WOULD HAVE CONVICTED HIM in light of the new evidence.
iii) Novel Constitutional Claim, but not very useful, b/c there is only a narrow gap here
when considering Teague (no collateral review of novel laws). So if its novel
constitutional claim, they are out of luck under Teague unless it falls under one of the
Teague exceptions.
5. 1983 cannot be used by prisoner to get out of prison w/o using proper habeas
procedures. Can use it to like improve prison conditions (get meds, etc), but have to
follow procedure otherwise to get out.

PRIOR STATE COURT DECISIONS OF LAW & FACTFINDING


A. Section 2254(d) & Prior State Court Decisions: 2254(d) becomes applicable to a habeas
corpus petition if a state court previously adjudicated a claim on the merits and that
adjudication resulted in a decision that was also on the merits. Therefore, 2254(d) is only
implicated if a state court previously investigated whether a claim had sufficient factual and legal
support to establish a violation of federal law and decided the claim did not.
B. No Habeas if claim was adjudicated on merits unless Contrary

to/unreasonable application of clearly est. fed law or unreasonable


determination of the facts
1. 2254(d): No habeas for any claim that was ADJUDICATED on the MERITS in State court
proceedings
i) UNLESS the adjudication of the claim resulted in a decision that was
a. CONTRARY TO, or
b. INVOLVED AN UNREASONABLE APPLICATION OF,
i. CLEARLY ESTABLISHED Federal LAW, as determined by Supreme
Court of the United States;
c. OR Resulted in a decision that was based on an UNREASONABLE
DETERMINATION of the FACTS in light of the evidence presented in the State
court proceeding.

88

Downloaded From OutlineDepot.com

Kannar Federal Courts, Spring 2011

C.

Clearly Established Federal Law?: Law must be established by the U.S. Supreme

Court at the time his state-court conviction became final. If a federal habeas court concludes that
a state court decision rejecting a prisoners claim was correct in light of the Courts holdings at
that time, that decision is dispositive and the claim must be dismissed.
D. Contrary To: if the state court arrives at a conclusion OPPOSITE to that reached by
SCOTUS on a question of law OR (ii) if state court decides a case differently than this Court has
on a set of MATERIALLY INDISTINGUISHABLE FACTS.
E. Unreasonable Application: A state-court decision involves an unreasonable application
of the Courts precedent if
1. The state court identifies the correct governing legal rule from this Courts cases but
UNREASONABLY APPLIES to the facts of the particular state prisoners case; or
2. The state court either UNREASONABLY EXTENDS a legal principle from our precedent to
a new context where it should not apply OR unreasonably REFUSES TO EXTEND that
principle to a new context where it should apply.
i) Objective Not Subjective Determination: The unreasonable application clause is
not concerned unreasonable judges; rather it is concerned with objectively unreasonable
state court decisions.
ii) Erroneous Not Enough, Must be Unreasonable Determination: 2254(d)(2) permits
habeas relief only if a state courts decision was anchored in a determination of the facts
that was not only wrong, but was also unreasonable.
F. Unreasonable Determination of the Facts: Fact Finding In State Court
Presumed Correct, Section 2254(e) :
1. Burden on applicant to rebut: 2254(e)(1), State court factual determinations presumed to
be correct.
i) The applicant can rebut the presumption of correctness by clear and convincing
evidence.
2. Usually No Evidentiary Hearing If Failed to Develop Facts, 2254(e)(2):
i) Failed To Develop: If the prisoner was himself responsible for inadequate fact-finding
in state court.
a. If the prisoner is not at fault, he gets an evidentiary hearing.
ii) Lack of Diligence Standard: A failure to develop the factual basis of a claim is not
established unless there is a lack of diligence, or some greater fault, attributable to the
prisoner or the prisoners counsel. This diligence depends on whether the prisoner
made a reasonable attempt, in light of the information available at the time, to investigate
and pursue claims in state court. It does not depend upon whether those efforts could
have been successful.
a. Michael Williams v. Taylor (2000) (2254(e)s failure to develop requirement means a
lack of diligence or some greater fault attributable to the prisoner or his counsel): The
statute does not bar the evidentiary hearing petitioner seeks on his juror bias and prosecutorial
misconduct claims. The defense counsel had no reason to know what was going on and he
could not be faulted for failing to visit the hall of records and learning the truth of the history
of the conflicted juror.
i. Davis: Cop sentenced to death, bringing up new evidence, SCOTUS refers petition to
D.Ct for fact finding.

3. Exception: If the applicant FAILED TO DEVELOP THE FACTUAL BASIS of a claim in


State court the court may hold an evidentiary hearing on the claim IF:
i) The claim relies on
a. A NEW RULE OF CONSTITUTIONAL LAW, made retroactive to cases on
collateral review by the Supreme Court, that was previously unavailable; or

89

Downloaded From OutlineDepot.com

Kannar Federal Courts, Spring 2011

b. A FACTUAL PREDICATE that could NOT have been PREVIOUSLY


DISCOVERED through the exercise of due diligence; AND
c. Innocent: The facts underlying the claim would be sufficient to establish by clear
and convincing evidence that but for constitutional error, NO REASONABLE
FACTFINDER would have found the applicant GUILTY of the underlying offense.

THE TEAGUE RETROACTIVITY DOCTRINE


A.

Retroactivity Doctrine in Habeas Corpus:


1. Teague v. Lane (1988) The case that defendant relied upon only held that the Sixth Amendment
requires that jury venire be drawn from a fair cross section of the community not that the juries
actually chosen must mirror the community and reflect various groups in the population. Accordingly,
the rule urged by the defendant would be a new constitutional rule. Further, because the absence of a
fair cross section of the jury venire does not undermine the fundamental fairness that must underlie a
conviction, the exceptions are not applicable.
2. Basically redundant anyways, since under AEDPA, can only get relief for decisions contrary to clearly
established federal review.

B.

RETROACTIVITY RULE:

In Habeas cases federal courts apply the rule of law that


prevailed when the prisoners conviction became final on direct review. it does not apply the
rule of law including new rules that prevails when the prisoner appears in federal court.
1. EXCEPTIONS:
i) Not Illegal Anymore: New rules that place certain kinds of PRIMARY, PRIVATE
INDIVIDUAL CONDUCT beyond the power of the criminal law-making authority to
proscribe i.e. the new substantive rule makes the conduct no longer criminal or
prohibits the death penalty in cases such as this.
a. Changes in Substantive Law: This is meant primarily to cover cases in which federal
substantive law develops to insulate citizens from criminal punishment at all e.g. finding a
law which made it a crime for married people to use contraceptives violative of due process
(Griswold v. Connecticut);
i. Penry v. Lynaugh (1989): The only decision finding the first exception, and presented the
claim that execution of a prisoner with the mental capacity of a seven-year-old violated
the Eighth Amendment.

ii) Fundamental Fairness: New rules without which the LIKELIHOOD OF AN


ACCURATE CONVICTION is SERIOUSLY DIMINISHED i.e. watershed rules
of criminal procedure changing bedrock procedural elements and which are necessary
for the fundamental fairness of the criminal proceeding.
a. Changes in Procedural Law: A new rule must not only seriously diminish the likelihood of
obtaining an accurate conviction and it must also alter the courts understanding of the
bedrock procedural elements essential to the fairness of a proceeding. To date, the Court
has never recognized a new rule that fit the second exception!

C.

DEFINING A NEW RULE: A case announces a new rule when it breaks new ground or
imposes a new obligation on the government. In subsequent cases, the Court has been clear that
clear breaks from precedent are not necessary, but rather gradual developments in the law over
which reasonable jurists may disagree can also produce entirely new rules.
i) Evidence of a New Rule: a new rule can be evidenced
a. Breaks New Ground: or imposes new obligation on States or Fed Govt.
b. Lack of Cited Precedent in New Rule Case: If a holding does not rely or cite any
precedent, this is good evidence that the decision constitutes a new rule. Precedent
is not cited if a case merely uses the citation c.f. indicating authority supporting in
only dictum or analogy.

90

Downloaded From OutlineDepot.com

Kannar Federal Courts, Spring 2011

c. Reasonable Jurist Reaching Different Result: If a reasonable jurist could have


reached a different result than the decision in the case, the decision likely constitutes
a new rule. (Butler)
D. Determining the Old Law: a habeas court should survey the legal landscape as it existed on the
date conviction became final to determine whether a state court considering the claim would have
felt COMPELLED BY EXISTING PRECEDENT to conclude that the rule the defendant
seeks was constitutionally required.
E. Refusing to Announce a New Rule Unless Retroactive: If no exceptions
implicating fundamental fairness concerns, the Supreme Court generally refuses to announce new
rules unless the rule would be applied retroactively to the defendant and all others similarly
situated. Accordingly, when reviewing a habeas petition, the Court will not announce a new
rule unless it fits in one of the exceptions.
1. Cant expect a state court to have followed a rule that didnt exist at time of trial.
F. State courts not bound by Teague : States can give further remedies that fed courts
would not themselves afford and could consider new rule that would not be applied retroactively
on habeas (Danforth v. Minn)
G. Notable Timing Situations
1. One-Year Statute of Limitation to file Habeas Petition under a New Rule: Under
2255, a 1-year period of limitation shall apply to a [habeas petition] under this section . . .
from . . . (3) the date on which the right asserted was initially recognized by the Supreme
Court, if that right has been newly recognized by the Supreme Court and made retroactively
applicable to cases on collateral review.
2. New Rule When Cert. is Pending (fed court will consider new rule) (Direct Review by Cert): If
the Supreme Court handed down a decision announcing a new rule after the date on which the states
highest court affirmed a prisoners conviction and sentence but before the time for seeking cert runs
out or the Supreme Court denies a petition for certiorari review, the prisoner is entitled to the benefit of
the new rule in federal court, even though the state court of last resort had no opportunity to consider it
on direct review.
3. New Rule When Case in State Court Post-Conviction (state gets to review): If the Supreme Court
announces a new rule after a prisoners conviction and sentence became final on direct appeal, but
before the highest state court considers a claim based on the new rule in state post-conviction
proceedings, the state court does have an opportunity to apply the new rule but may decline.

H. Finality: When state court appellate proceedings ended or when SCOTUS cert was denied.
1. Criticism: The major criticism of the retroactivity doctrine is that the date of finality is an
arbitrary cutoff. A prisoner may be benefited by slow trial or direct appeals process. This is
especially problematic with co-defendants convicted of the same crime.

SECOND OR SUCCESSIVE PETITIONS ABUSE OF


THE WRIT
A. Historical Approach Deliberate Bypass Standard: In the Warren Court era, the Supreme
Court generously allowed multiple federal habeas petitions. The prisoners new claim could only
be dismissed if he had abused the habeas corpus remedy by deliberately bypassing the
opportunity to raise the claim in the initial application.
1. Sanders v. United States (1963): Held: A federal court may give controlling weight to denial of a
prior application for collateral review only if that application had rejected the present claim on the
merits and if the ends of justice would not be served by reaching the merits of the subsequent
application.

91

Downloaded From OutlineDepot.com

Kannar Federal Courts, Spring 2011

i)
B.

But See McCleskey v. Zant (1991): Held: The failure to raise a claim in an earlier federal habeas
petition will be excused only by showing that (i) cause and prejudice, or (ii) that a fundamental
miscarriage of justice would result from a failure to entertain the claim.

Claims Previously Presented Will be Dismissed:

1. Under 2244(b)(1), a claim that was presented in a previous 2254 habeas petition shall be
dismissed.
i) Does Not Apply to Dismissals on Technical Grounds: If, for example, a claim was
dismissed on a prior occasion because the prisoner failed to exhaust state opportunities
for litigating the claim, when the prisoner satisfies the exhaustion doctrine 2244(b)(1)
permits another petition.
C. Claims Not Previously Presented: Under 2244(b)(2), a claim raised for the first time
in a second or successive habeas petition may be considered, IF:
1. New Constitutional Basis: The applicant shows that the claim relies on a NEW RULE OF
CONSTITUTIONAL LAW, made retroactive to cases on collateral review by Supreme
Court, that was PREVIOUSLY UNAVAILABLE; OR
2. Both:
i) New Facts: The FACTUAL PREDICATE for the claim could NOT HAVE BEEN
DISCOVERED previously through the exercise of due diligence; AND
ii) Innocence: The facts, if proven and viewed in light of the evidence as a whole, would be
sufficient to establish by clear and convincing evidence that, but for constitutional error,
NO REASONABLE FACTFINDER would have found applicant GUILTY of the
underlying offense.
D. Original Habeas Jurisdiction of Supreme Court Not Barred by
Successive Petitions: Section 2244(b) instructs for the grant or denial of authorization by
the court of appeals. In Felker v. Turpin (1996), the Court ruled that the statute did not preclude
a prisoner from filing a petition in the Supreme Court seeking original writ of habeas corpus.

92

Vous aimerez peut-être aussi